Jaypee’s Triple-A: A Treatise for NEET (Volume 2) Sakshi Arora, Taruna Mehra
INDEX
×
Chapter Notes

Save Clear


1
  • Section A
    • 1. Neonatology
    • 2. Growth and Development
    • 3. Nutrition in Health and Disease
    • 4. Fluid and Electrolytes
    • 5. Disorders of Gastrointestinal System
    • 6. Immunization, Infectious Diseases and Worm Infestation
    • 7. Hematological Disorders
    • 8. Disorders of Cardiovascular System
    • 9. Disorders of Respiratory System
    • 10. Disorders of Kidney and Urinary Tract
    • 11. Disorders of Endocrine System
    • 12. Disorders of Neuromuscular System
    • 13. Metabolic Disorders
    • 14. Genetic Disorders
    • 15. Childhood Malignancies
    • 16. Miscellaneous
  • Section B
    • Practice Questions (Comprising of Questions from Recent Exams and NEET Pattern Questions)

Section A

 
1. NEONATOLOGY
  • A. Normal New Born
  • B. Primitive Reflexes
  • C. Low Birth Weight
  • D. Respiratory Distress
  • E. Congenital Diaphragmatic Hernia
  • F. Bronchopulmonary Dysplasia
  • G. Resuscitation
  • H. Neonatal Jaundice and Kernicterus
  • I. Neonate of Diabetic Mother
  • J. Miscellaneous
 
4NEONATOLOGY (QUESTIONS)
 
A. NORMAL NEW BORN
1. The appropriate approach to a neonate presenting with vaginal bleeding on day 4 of life is: (AI 05)
  1. Administration of vitamin K
  2. Investigation bleeding for disorder
  3. No specific therapy
  4. Administration of 10 ml/kg of fresh frozen plasma over 4 hours
Ref: Ghai 8/e p172
2. Neonatal MCV: (PGI Nov 09)
  1. 90-110
  2. 110-125
  3. 125-135
Ref: Ghai 8/e p322; 7/e, p296, 298
3. A full term baby, exclusively breast fed, at the end of 1 week was passing golden yellow stools and was found to have adequate hydration with normal systemic examination. The weight of the baby was just same as it was at birth. The pediatrician should now advise: (AIIMS May 02)
  1. Give oral solution with breast feeding
  2. Start top feeding
  3. Investigate for lactic acidosis
  4. Reassure the mother that nothing is abnormal
Ref: Ghai 6/e, p4
4. Normal finding in term neonate: (PGI Dec 02, Dec 98)
  1. Erythema toxicum
  2. Epstein’s pearl
  3. Bilateral cryptorchidism
  4. Subconjunctival hemorrhages
  5. Erythema nodosum
Ref: Ghai 8/e p138
5. Most common cause of delayed puberty in males is: (AI 08)
  1. Kallamann syndrome
  2. Klinefelter syndrome
  3. Constitutional
  4. Prader-willi syndrome
Ref: Ghai 8/e p535; Oski’s 4/e, p2083
6. The pincer grasp with the index finger and the thumb is attained by an infant by the age of: (MP PG 2009)
  1. 3 months
  2. 6 months
  3. 9 months
  4. 12 months
Ref: Ghai 8/e p49-55; 7/e p28
7. Upper segment to lower segment ratio at birth is: (J & K PG 2011)
  1. 1.3
  2. 1.5
  3. 1.7
  4. 1
Ref: OP Ghai 8/e p11
8. Kangaroo mother care is necessary till baby attains a weight of: (J & K PG 2011)
  1. 1500 gms
  2. 2000 gms
  3. 2500 gms
  4. 3000 gms
Ref: OP Ghai 8/e p148
9. Gestational age of a new born baby can be assessed by which one of the following scoring system: (J & K PG 2010)
  1. Sarnat and Sarnat
  2. Downeys
  3. Silvermans
  4. New Ballards
Ref: OP Ghai 8/e p125
 
B. PRIMITIVE REFLEXES
10. A 3 month baby will have: (PGI Nov 09)
  1. Pincer grasp
  2. Head control
  3. Sitting with support
  4. 2 words with meaning
  5. Rolling over
Ref: Ghai 8/e, p49
11. Moro reflex disappears at: (AI 07; PGI June 98)
  1. 5 months
  2. 3 months
  3. 7 months
  4. 6 months
Ref: Nelson 18/e, p2439; Ghai 8/e p142
12. Persistence of Moro’s reflex is abnormal beyond the age of: (AIIMS May 07)
  1. 3rd month
  2. 4th month
  3. 5th month
  4. 6th month
Ref: Ghai 8/e p142; Nelson 18/e p2439
13. All of the following reflexes are present at birth except: (AI 07; PGI June 98)
  1. Rooting reflex
  2. Symmetrical tonic neck reflex
  3. Asymmetrical neck reflex
  4. Crossed extensor reflex
Ref: Nelson 18/e p2439; Ghai 8/e p142
14. Moro’s reflex is abnormal after: (DNB 2007)
  1. 4 weeks
  2. 10 weeks
  3. 12 weeks
  4. 20 weeks
Ref: OP Ghai peds 8/e p210
15. Persistent moro’s reflex at 12 weeks indicates: (Feb DP PGMEE 2009)
  1. Normal child
  2. Brain damage
  3. Hungry child
  4. Irritable
Ref: Ghai 8/e p143
5
16. Persistent moro’s reflex at 12 weeks indicates: (Feb DP PGMEE 2009)
  1. Normal child
  2. Brain damage
  3. Hungry child
  4. Irritable
Ref: Ghai 8/e p143
17. Which of the following is not present at birth? (Kerala PG 08)
  1. Parachute reflex
  2. Moro reflex
  3. Tonic neck reflex
  4. Rooting reflex
Ref: Ghai 8/e p133; Nelson, 17/e p1979, Table 584-2 18/e p2439, Table 594-2
 
C. LOW BIRTH WEIGHT
18. All of the following are features of prematurity in a neonate, except: (AI 06)
  1. No creases on sole
  2. Abundant lanugo
  3. Thick ear cartilage
  4. Empty scrotum
Ref: Ghai 8/e p124; 7/e p129; Nelson 18/e p703; 17/e p525
19. Small for date baby is: (PGI June 00)
  1. < 10 percentile for the gestational age
  2. < 50 percentile for gestational age
  3. < 2000 gm
  4. < 2500 gm
Ref: Ghai 8/e p155; 7/e p129
20. A women delivered a baby of 2.2 kg weight her IMP is not known. To know the maturity of baby, following are used: (PGI June 03)
  1. Sole crease
  2. Ear cartilage
  3. Breast nodule
  4. Anterior fontaneue
  5. Weight of the baby
Ref: Ghai 8/e p155
21. Full term, small-for-date babies are at high-risk of: (AI 00)
  1. Hypoglycemia
  2. Intraventricular hemorrhage
  3. Bronchopulmonary dysplasia
  4. Hyperthermia
Ref: Ghai 8/e p179; 7/e p129
22. Retinopathy of prematurity is commonly predisposed by: (AIIMS June 00)
  1. Less gestation age
  2. Low birth weight
  3. O2 toxicity
  4. Carbohydrate excess
Ref: Nelson 18/e p2423, 2603 and 17/e p2113; ROP, p709, 2598-2600; Ghai 8/e p666; 7/e p130
23. Loss of pulmonary surfactant in premature infant: (Nov 08)
  1. Pulmonary edema
  2. Collapse of alveoli
  3. Elastic recoil of lungs
  4. All of above
Ref: Nelson 17/e p575, 18/e p732; Ghai 8/e p373
24. Retrolental fibroplasias has association with: (AIIMS May 02)
  1. Prolonged labor
  2. Intrauterine infection
  3. Meconium aspiration
  4. Low birth weight
Ref: Ghai 8/e p155; Meharban Singh 6/e p404
25. True about LBW is: (DNB 2009)
  1. Less than 1800 gms
  2. Less than 2000 gms
  3. Less than 2500 gms
  4. Less than 3000 gms
Ref: OP Ghai 8/e p310
26. Low birth weight (LBW) baby is the one whose birth weight is: (MP PG 2010)
  1. Less than 1800 gms
  2. Less than 2000 gms
  3. Less than 2500 gms
  4. Less than 3000 gms
Ref: Ghai 8/e p124; Nelson’s 18/e p702
27. A very low birth weight is a newborn whose weight, in grams, is less than: (MP PG 2010)
  1. 1000
  2. 1500
  3. 2000
  4. 2500
Ref: Ghai 8/e p124; Nelson’s 18/e p702
 
D. RESPIRATORY DISTRESS
28. With reference to RDS, all of the following statements are true except: (AI 02; PGI Dec 99)
  1. Usually occurs in infants born before 34 weeks of gestation
  2. Is more common in babies born to diabetic mothers
  3. Leads to cyanosis
  4. Is treated by administering 100% oxygen
Ref: Ghai 8/e p169; 7/e p144-146, 6/e, p166-167; Nelson 18/e p731-740 and 17/e p575-578
29. Most common sign of acute hypoxia in neonates: (AI 09)
  1. Tachycardia
  2. Bradycardia
  3. Cardiac arrest tachypnea
  4. Ventricular arrhythmia
Ref: Nelson 18/e p718, 719; Ghai 8/e p166
6
30. Nile blue sulfate test is done to detect: (AIIMS Nov 06)
  1. Maturity of kidney
  2. Maturity of liver
  3. Maturity of lungs
  4. Maturity of skin
Ref: Meharban Singh 6/e, p42-43; Dutta p420
31. Neonatal apnea is seen in all except: (Nov 08)
  1. Prematurity
  2. Hyperglycemia
  3. Hypoglycemia
  4. Hypercalcemia
  5. Hyperthermia
Ref: OP Ghai 8/e p164; 7/e p146
32. Which of the following are signs of neonatal respiratory distress syndrome: (PGI June 01)
  1. Intercostal retraction
  2. RR > 60/min
  3. Absence of cyanosis
  4. PH < 7.2
  5. A linear streak on CXR
Ref: Ghai 8/e p137; 7/e p144-146; Nelson 18/e p723; 17/e p575-577
33. Neonate at 2 hours age develops moderate respiratory distress, in terms of increased respiratory rate (70/m), chest indrawing and grunting. Which is the best management: (AIIMS Nov 09)
  1. Surfactant therapy and mechanical ventilation
  1. Intubation and mechanical ventilation
  2. Facemask with CPAP
  3. 100% O2 and and head box
Ref: Ghai 8/e p169
34. A 3 kg term baby delivered by cesarian section develops respiratory distress soon after birth. The liquor was meconium stained. Breathing rate is 90/ minute. Correct statements: (PGI Dec 04)
  1. Transient tachypnea of newborn
  2. Meconium aspiration syndrome
  3. Reticulonodular shadows in X-ray chest
  4. Surfactant production
  5. Oral feeding started early
Ref: Ghai 8/e p170; Nelson 18/e p742-743 and 17/e p583-584
35. Respiratory distress in newborn is defined when: (PGI June 04)
  1. Respiration rate >60/min
  2. Intercostal recession
  3. Aspiration> 20 ml
  4. Hypoxemia
  5. Reticulonodular shadow in CXR
Ref: Ghai 8/e p169; 7/e p144-147; Nelson 18/e p723 and 17/e p575-577
36. Newborn baby of term gestation, developed respiratory depression after 6 hours. What will suggest HMD?
  1. Receipt of antenatal steroids
  1. Air bronchogram in CXR
  2. Term gestation
  3. Onset after 6 hours
Ref: Ghai 8/e p169; Nelson’s, 18/e p731-5
37. True about RDS are all except:
  1. Develops 6 hours after birth
  2. Air bronchogram seen
  3. Cyanosis seen
  4. Prenatal steroids given
Ref: Ghai 8/e p169; Nelson’s, 18/e p731-5
38. A child 30 weeks preterm presents with moderately severe respiratory distress with grunting after 2 hours. Respiratory management.
  1. Nasal CPAP
  2. Surfactant with mechanical ventilation
  3. Intubation
  4. Warm oxygen
Ref: Nelsons. 18/e p731-5, OP Ghai 8/e p169; 7/e p144
39. In a postoperative ward newborn developed respiratory depression in ward. It can be caused by?
  1. Opioid
  2. Propofol
  3. Diazepam
  4. Ketamin
Ref: Care of newborn Meharban singh, 6/e p79; 8/e p168
40. A neonate with scaphoid abdomen with respiratory distress has: (Karnataka PG 2010)
  1. Congenital pyloric stenosis
  2. Volvulus
  3. Congenitae Diaphragmatic hernia
  4. Choanal atresia
Ref: Ghai 8/e p138, 178
 
E. CONGENITAL DIAPHRAGMATIC HERNIA
41. A child presented with severe respiratory distress two days after birth. On examination he was observed to have a scaphoid abdomen and decreased breath sounds on the left side. He was managed by prompt endotracheal intubation. After ET tube placement the maximal cardiac impulse shifted further to the right side. What should be the next step in management? (AI 09)
  1. Confirm the position of endotracheal tube by chest X-ray
  2. Remove tube and reattempt intubation
  3. Nasogastric tube insertion and decompress the bowel
  4. Chest X-ray to confirm diagnosis
Ref: Ghai 8/e p138, 178; 7/e p153; Nelson 18/e p731, 740
7
42. A child presented with respiratory distress was brought to emergency with bag and mask ventilation. Now child is intubated. Chest X-ray shows right-sided deviation of mediastinum with scaphoid abdomen. His pulse rate is increased. What is the next step? (AIIMS Nov 07)
  1. Endotracheal intubation
  2. Put a nasogastric tube
  3. Surgery
  4. End tidal CO2 to confirm intubation
Ref: Ghai 8/e p178; 7/e p153
43. A neonate having congenital diaphragmatic hernia developed respiratory distress. Breath sounds were decreased on the left side. After bag and mask ventilation, ET tube was put and the maximal cardiac impulse shifted to the right side. What should be the next step in management: (AI 08)
  1. Confirm the position of endotracheal tube by X-ray chest
  1. Remove tube and reattempt intubation
  2. Nasogastric tube insertion
  3. Chest X-ray
Ref: Nelson 18/e p748; Ghai 8/e p178
44. A Newborn weighing 1000 g is born at gestational age of 30 weeks with respiratory distress after 2-3 hours of birth. What are the diagnostic possibilities: (PGI June 01)
  1. Diaphragmatic hernia
  2. Cong. bronchopulmonary cysts
  3. Bronchopulmonary dysplasia
  4. HMD
  5. Pulmonary haemorrhage
Ref: Ghai 8/e p179; 7/e p153; Nelson 18/e p731-740 and 17/e p575–578
45. Newborn with APGAR score of 2 at 1 min. and 6 at 5 min. has respiratory distress and mediastinal shift diagnosis is: (PGI Dec 00)
  1. Congenital adenomatoid lung disease
  2. Pneumothorax
  3. Diaphragmatic hernia
  4. Transient tachypnea of newborn
  5. HMD
Ref: Nelson 18/e p731-740; 17/e, 575–578; Ghai 8/e p176
46. A newborn baby had normal APGAR score at birth and developed excessive frothing and choking on attempted feeds. The investigation of choice is: (AIIMS May 03)
  1. Esophagoscopy
  2. Bronchoscopy
  3. MRI chest
  4. X-ray chest and abdomen with the red rubber catheter passed per orally into esophagus
Ref: Ghai 8/e p178-179; 7/e p151; Nelson 18/e p1541 and 17/e p1219-1220
47. A new born baby has been referred to the casualty as a case of congenital diaphragmatic hernia. The first clinical intervention is to: (AIIMS May 03)
  1. Insert a central venous pressure line
  2. Bag and mask ventilation
  3. Insert a nasogastric tube
  4. Ventilate with high frequency ventilator
Ref: Nelson 18/e p746-749; 17/e p1353-1355; Schwartz 7/e p1720; Ghai 8/e p179
 
F. BRONCHOPULMONARY DYSPLASIA
48. A male born at term after an uncomplicated pregnancy, labor and delivery develops severe respiratory distress within a few hours of birth. Results of routine culture were negative. The chest roengogram reveals a normal heart shadow and fine reticulonodular infiltrates radiating from the hilum. ECHO findings reveal no abnormality. Family history reveals similar clinical course and death of a male female sibling at 1 month and 2 months of age respectively. The most likely diagnosis is: (AI 08)
  1. Neonatal alveolar proteinosisy
  2. Total anomalous pulmonary venous circulation (TAPVC)
  3. Meconium aspiration syndrome
  4. Diffuse herpes simplex infection
Ref: Nelson 18/e p1820-1821; Ghai 8/e p176
49. A newborn female child, weight 3.5 kg, delivered by uncomplicated delivery, developed respiratory distress immediately after birth. On chest X-ray ground glass appearance was seen. Baby put on mechanical ventilation and was give surfactant but condition of baby deteriorates and increasing hypoxemia was present. A full term female sibling died within a week with the same complaints. ECHO is normal. Usual cultures are negative. Your diagnosis is: (AIIMS Nov 08)
  1. Total anomalous pulmonary vein connection
  2. Meconium aspiration syndrome
  3. Neonatal pulmonary alveolar proteinosis
  4. Disseminated HSV infection
Ref: Nelson 18/e p1821-1822; Ghai 8/e p168; 176
50. 3.5 kg term male baby, born of uncomplicated pregnancy, developed respiratory distress at birth, did not responded to surfactant, ECHO finding revealed nothing abnormal, X-ray showed ground glass appearance and culture negative. APGARS 4 and 5 at 1 and 5 minutes. History of one month female sibling died before. What is the diagnosis? (AIIMS June 08)
  1. TAPVC
  2. Meconium aspiration
  3. Neonatal pulmonary alveolar proteinosis
  4. Diffuse herpes simplex infection
Ref: Nelson 18/e p2185, Harrison 16/e p1696; Gahi 8/e p172
 
8G. RESUSCITATION
51. A 5-year-old child is rushed to casualty reportedly electrocuted while playing in a park. The child is apneic and is ventilated with bag and mask. There are bums on each hand. What will be the next step in the management: (AIIMS Nov. 05, 04)
  1. Check pulses
  2. Start chest compressions
  3. Intubate
  4. Check oxygen saturation
Ref: Nelson 18/e p395; 17/e p281, 286–288; Ghai 8/e p125
52. A 6 months old child having severe dehydration comes to the casualty with weak pulse and unrecordable BP Repeated attempt in gaining IV access has failed. The next best step is: (AIIMS May 01)
  1. Try again
  2. Jugular vein catheterization
  3. Intraosseous IV fluids
  4. Venesection
Ref: Bailey and Love 24/e p290; Ghai 8/e p714
53. A child presented in the casualty with fever, unconscious and papilloedema. What next to be done: (PGI Nov 08)
  1. Intubation
  2. Oxygenation
  3. CT scan
  4. Lumber puncture
Ref: Ghai 8/e p710; 7/e p535-536
54. Best artery to palpate for pulse in infants is: (PGI Dec 00)
  1. Femoral artery
  2. Radial artery
  3. Carotid artery
  4. Brachial artery
Ref: Morgan’s Anesthesia 3/e p936; Ghai p710; 712
55. Which among the following is a helpful characteristic in neonatal resuscitation: (Kerala PG 10)
  1. APGAR Score
  2. Colour
  3. Cry
  4. Heart rate
Ref: Ghai 8/e p126; Nelson 18/e p679
56. A neonate on bag and mask ventilation in cardio pulmonary resuscitation. The heart rate is 60-100 per munute. Next treatment is: (UP PG 2010)
  1. Continue ventilation and start adranaline
  2. Continue ventilation and start chest compressions
  3. Continue chest compressions and ventilation
  4. Stop chest compressions and continue ventilation
Ref: OP Ghai 8/e p714; 7/e p99
57. A ratio of lung inflation to chest compression in cardiopulmonary resuscitation is: (UP PG 2010)
  1. 1:5
  2. 2:5
  3. 1:15
  4. 2:15
Ref: Ghai 8/e p714; 7/e p691-92; Nelson 18/e p389
58. The following modes of ventilation may be used for weaning of patients from mechanical ventilation except: (UP PG 2010)
  1. Pressure support ventilation (PSV)
  2. Control mechanical ventilation (CMV)
  3. Assist control ventilation (ACV)
  4. Synchronized intermittent mandatory ventilation (SIMV)
Ref: Ghai 7/e p701-03; A yadav 4/e p205
59. The first medication that may be required in the resuscitation of baby with prolonged birth asphyxia is: (J & K PG 2010)
  1. Sodium bicarbonate
  2. Adrenalin in 1:10000 dilution
  3. Calcium gluconate
  4. 25% dextrose
Ref: OP Ghai 8/e p166, 126
 
H. NEONATAL JAUNDICE AND KERNICTERUS
60. Drugs that can be used in kernicterus: (PGI June 09)
  1. Barbiturates
  2. Benzodiazepines
  3. Phenytoin
  4. Chlorpromazine
  5. Carbamazepine
Ref: Ghai 8/e p172; 7/e p150
61. A child has bilirubin of 4 mg. Conjugated bilirubin and alkaline phosphatase are normal, bile salts and bile in urine are absent. However urobilinogen in urine is raised. What is the likely diagnosis: (AIIMS Nov 01)
  1. Obstructive jaundice
  2. Rotor’s syndrome
  3. Biliary cholestasis
  4. Hemolytic jaundice
Ref: Chatterjee Shinde 4/e p593; Chandrasoma 3/e p635; Ghai 8/e p172-175
62. Conjugated hyperbilirubinemia in infancy seen in: (PGI Dec 04)
  1. Choledochal cyst
  2. Extra hepatic biliary atresia
  3. Crigler - Najjar disease
  4. Gilbert disease
Ref: Already explained above; Ghai 8/e p173
63. Conjugated hyperbilirubinemia in infancy is seen in: (PGI Dec 06)
  1. Gilbert syndrome
  2. Crigler-Najjar syndrome
  3. Dubin Johnson syndrome
  4. Rotor syndrome
  5. Neonatal hepatitis
Ref: Already explained above; Ghai 8/e p173
64. A term neonate with unconjugated hyperbilirubinemia of 18 mg/dl on 20 days. All are common causes except: (AIIMS May 07)
  1. Breast milk jaundice
  2. Congenital cholangiopathy
  3. G 6PD deficiency
  4. Hypothyroidism
Ref: Ghai 8/e p173-174; 7/e p147-151
9
65. Late onset hemorrhagic disease of newborn is characterized by all of the following features except: (AI 06)
  1. Usually occurs in cow-milk-fed babies
  2. Onset occurs at 4-12 weeks of age
  3. Intracranial hemorrhage can occur
  4. Intramuscular vitamin K prophylaxis at birth has a protective role
Ref: Nelson 18/e p773-774; 17/e p606–607; Ghai 8/e p148
66. Which of the following is the principal mode of heat exchange in an infant incubator? (AI 06)
  1. Radiation
  2. Evaporation
  3. Convection
  4. Conduction
Ref: Ghai 8/e p173; 7/e p150
67. In unconjugated hyperbilirubinemia, the risk of kernicterus increases with the use of: (AI 05)
  1. Ceftriaxone
  2. Phenobarbitone
  3. Ampicillin
  4. Sulfonamide
Ref: Harrsion 16/e p1818; KDT 5/e, p644; Ghai 8/e p173
68. What should be measured in a newborn who presents with hyperbilirubinemia: (AI 00)
  1. Total and direct bilirubin
  2. Total bilirubin only
  3. Direct bilirubin only
  4. Conjugated bilirubin only
Ref: Meharban Singh 6/e p241; Ghai 8/e p172
69. Unconjugated hyperbilirubinemia in neonate is seen in all of the following except: (AI 98)
  1. Physiological jaundice
  2. Dubin Johnson syndrome
  3. Hypothyroidism
  4. Hemolytic anemia
Ref: Ghai 8/e p172; Nelson 18/e p1677; 17/e p592-596, 1321
70. Which mechanism in phototherapy is chiefly responsible for reduction in serum bilirubin? (AIIMS May 05)
  1. Photo-oxidation
  2. Photo-isomerization
  3. Structural isomerization
  4. Conjugation
Ref: Ghai 8/e p172; 7/e p150, 6/e p172-173; Nelson 18/e p762-764; 17/e p597-598
71. A full term, 80 hours old new born baby develops jaundice, what should be the minimum level of serum bilirubin to start phototherapy: (AIIMS June 99)
  1. 20 mg%
  2. 12.5 mg%
  3. 18 mg%
  4. 15 mg%
Ref: Ghai 8/e p173; Nelson 18/e p762-763; 17/e p597–598; Meharban Singh 6/e p252
72. A term baby developed jaundice on 3rd day up to the thigh with normal stool and urine. Mother’s blood group is ‘O’ -ve and that of baby’s ‘A’ +ve. The cause of jaundice is: (PGI Dec 04; Dec 02)
  1. Rh incompatibility
  2. Physiological jaundice
  3. Extrahepatic biliary atresia
  4. Sepsis
  5. Glucose-6 phosphate dehydrogenase deficiency
Ref: Ghai 8/e p173; 7/e, 1147; Nelson 18/e p760-761 and 17/e p594-595
73. True about physiological jaundice in neonate: (PGI Dec 00)
  1. Occurs in first 6 hours of delivery
  2. Unconjugated hyperbilirubinemia
  3. Neurological equel are common
  4. Best treated by phototherapy
  5. Starts on 2nd day of life
Ref: Ghai 8/e p172
74. Bronze baby syndrome is due to: (PGI Dec 98)
  1. Phototherapy
  2. Wilson disease
  3. Chloramphenicol toxicity
  4. Hemochromatosis
Ref: Nelson 18/e p762, 763 and 17/e p598; Ghai 8/ep172; 7/e p150
75. The late features of kernicterus include all except: (DP PGMEE 2009)
  1. Hypotonia
  2. Sensorineural hearing loss
  3. Choreoathetosis
  4. Upward gaze palsy
Ref: Ghai 8/e p174
76. In a neonate, jaundice appears for the first time in the 2nd week. The following is not a cause: (DP PGMEE 2010)
  1. Galactosemia
  2. Rh incompatibility
  3. Hypothyroidism
  4. Breast milk jaundice
Ref: Ghai 8/e p172; Nelson 18/e p758
 
I. NEONATE OF DIABETIC MOTHER
77. Which of the following malformation in a newborn is specific for maternal insulin dependent diabetes mellitus? (AI 06)
  1. Transposition of great arteries
  2. Caudal regression
  3. Holoprosencephaly
  4. Meningomyelocele
Ref: Read below Ghai 8/e p179; 7/e p156
78. Infants of diabetic mother have the folio wings: (PGI June 03)
  1. Macrosomia
  2. Neural tube defect
  3. Hyperglycemia
  4. Hypocalcemia
Ref: Nelson 18/e p783, 784; Dutta-Obs 5/e, p 303, 304; Ghai 8/e p179; 7/e p156
10
79. Long-term complication of infants born to IDDM mother all except: (AI 95, AIIMS 98)
  1. DM
  2. Ketotic hypoglycemia
  3. Obesity
  4. Blindness
Ref: Ghai 8/e p179; 7/e p156
80. All of the following are the complications in the new born of a diabetic mother except: (AIMS May 06)
  1. Hyperbilirubinemia
  2. Hyperglycemia
  3. Hypocalcemia
  4. Hypomagnesemia
Ref: Ghai 8/e p179; 7/e p156, 6/e p180; Nelson Pediatrics 17/e p613-614
81. Macrosomia is seen in: (PGI Nov 09)
  1. GDM
  2. Maternal obesity
  3. Maternal hypothyroidism
  4. Neonatal Hyperglycemia
  5. Neonatal Hypoglycemia
Ref: Dutta Obs 6/e p287; Ghai 8/e p178, 179
82. All of the following therapies may be required in a 1 hour old infant with severe birth asphyxia except: (AI 05)
  1. Glucose
  2. Dexamethasone
  3. Calcium gluconate
  4. Normal saline
Ref: Meharban Singh 6/e p106; Ghai 8/e p166
83. The most common congenital anomaly in baby born to IDDM mother is: (AIIMS Nov 06)
  1. NTD
  2. Cardiovascular anomalies
  3. GIT anomalies
  4. Pulmonary anomalies
Ref: Ghai 8/e p396; Nelson 18/e p618, 619 and 17/e p614
84. A 3.5 kg baby born to diabetic mother develops seizures at 16 hours. The most likely cause is: (DNB 2010)
  1. Hypoglycemia
  2. Hypoxia/Respiratory distress syndrome
  3. Hypomagnesumia
  4. Hypocalcemia
Ref: Ghai 8/e p210; Nelson Essentials of Pediatrics 4/e p762-766, 783, 237, 238
85. All can be seen in infant of diabetic mother except: (Kerala PG 08)
  1. Hyperbilirubinaemia
  2. Polycythaemia
  3. Hyperglycaemia
  4. Hypocalcaemia
Ref: Ghai 8/e p182; Dutta 6/e p287
 
J. MISCELLANEOUS
86. Hypothermia in neonate is characterized by: (PGI Dec 02)
  1. Hyperactivity
  2. Hypoglycemia
  3. Apnea
  4. ↓ed urinary output
Ref: Ghai 8/e p143; 7/e p118, 115
87. The different manifestations of hypothermia are: [PGI June 06]
  1. Apnea
  2. Hypoglycemia
  3. Hyperglycemia
  4. Tachycardia
  5. Hypoxia
Ref: Ghai 8/e p143; 7/e, p115-118
88. Neonate with recurrent infection and abscess is diagnosed as kostmann syndrome (severe congenital neutropenia). What is the treatment: (AIIMS Nov 09)
  1. Anti-thymocyte globulin + cyclosporine
  2. Anti-thymocyte globulin + cyclosporine + gm-csf
  3. G-csf
  4. Gm-csf
Ref: Ghai 8/e p357; Current Diagnosis and treatment-Pediatrics 19/e p827
89. True about neonatal sepsis: (PGI June 03)
  1. Meningitis commonly occur lately
  2. Jaundice predisposes
  3. Fever
  4. Jaundice is a common feature
Ref: Ghai 8/e p163; 7/e p136-137
90. Transient tachypnea of new born (TTN) is commonly seen in which of the following situations: (AIIMS May 02)
  1. Term delivery requiring forceps
  2. Term requiring ventouse
  3. Elective cesarean section
  4. Normal vaginal delivery
Ref: Nelson 18/e p741; 17/e p583; Ghai 8/e p168; 7/e p146
11
91. Characteristics radiological feature of transient tachypnea of newborn is: (AIIMS May 05)
  1. Reticulogranular appearance
  2. Low volume lungs
  3. Prominent horizontal fissure
  4. Air bronchogram
Ref: Ghai 8/e p168
92. A nonventilated preterm baby in incubator is under observation. Which is the best way to monitor the baby’s breathing and detect apnea? (AI 07)
  1. Infrared throraric movement study
  2. Capnography
  3. Nasal digital temperature monitoring
  4. Impedence technique
Ref: Care of the newborn by Meharban Singh 6/e p30, 280; Ghai 8/e p180
93. Treatment for breath holding spells in a child is: (PGI Dec 01)
  1. Give extra care and love to the child
  2. Inflicting painful stimulus at the beginning of the attack
  3. Do not give attention to the child
  4. Fulfill all the wishes of the child to prevent the attack
  5. Low dose barbiturates
Ref: Nelson 18/e p131 and 17/e p88
94. True about B/L CDH: (PGI June 09)
  1. Exaggerated lordosis
  2. B/L genu valgum
  3. Waddling gait
  4. Stenton’s line brocken
  5. Short stature
Ref: Maheshwari 4/e p212, 213; 3/e p201, 202; Ghai 8/e p284
95. A 4 kg baby born to a diabetic mother found lethargic which of the folio whig is to be done: (PGI Dec 03)
  1. Reasses the baby again after 2 hours
  2. Give 10% dextrose IV
  3. Start oral feeding
  4. Give injection insulin
Ref: Ghai 8/e p179; 7/e p157
96. Administration of glucose solution is prescribed for all of the following situations except: (AIIMS May 06)
  1. Neonates
  2. Child of a diabetic mother
  3. History of unconsciousness
  4. History of hypoglycemia
  5. Uncorrected cases may develop plagiocephaly
Ref: Ghai 8/e p179
97. All of the following groups of newborns are at an increased risk of hypoglycemia except: (AIIMS Nov 02)
  1. Birth asphyxia
  2. Respiratory distress syndrome
  3. Maternal diabetes
  4. Post-term infant
Ref: Ghai 8/e p179; Nelson 18/e p785and 17/e p506-508
98. A male child of 3.8 kg born to a diabetic mother, developded seizures 16 hours after birth. What is the probable cause: (AIIMS Nov 09)
  1. Hypoglycemia
  2. Hypocalcemia
  3. Birth asphyxia
  4. Intracranial haemorrhage
Ref: Ghai 8/e p179; 7/e p156
99. A newborn child RR 86 per minute. No nasal flaring, no lower chest or xiphoid retraction, no grunt. Abdomen lagged behind chest in movement. Silverman’s score is:
  1. 1
  2. 2
  3. 3
  4. 4
Ref: Care of Newborn Meharban singh, 6/e p262; Ghai 8/e p174
100. In a preterm with PDA least likely is:
  1. Bounding pulses
  2. NEC
  3. CO2 washout
  4. Pulmonary hemorrhage
Ref: Ghai 8/e p402; Nelson. Textbook of pediatrics, 18/e p737, Neo Reviews 2010; 11: 495-502
101. Lines of blaschko are:
  1. Lymphatics
  2. Blood vessel
  3. Nerves
  4. Lines of development
Ref: Harper, John. Textbood of Pediatric Dermatology, p691; Internet
102. A neonate delivered at 38 weeks of gestation, birth weight of 2.2 kg develops intolerance to feeds on 2nd day. Physical examination reveals no abnormalities. Sepsis screen in negative. What is the next step in management?
  1. Wait and watch
  2. Do a 2nd sepsis screen
  3. Give prophylactic antibiotics
  4. X-ray abdomen
Ref: Ghai 8/e p163
12
103. Not a component of APGAR score is? (DNB 2010)
  1. Muscle tone
  2. Color of body
  3. Heart rate
  4. Respiratory rate
Ref: OP Ghai 8/e p137
104. The parameters used in APGAR score include all of the following except: (MP PG 2010)
  1. Respiratory rate
  2. Heart rate
  3. Muscle tone
  4. Reflex stimulation
Ref: Ghai 8/e p126; Nelson’s 18/e p679
105. Which of the following is not true about late onset Hemorrhagic disease of newborn (HDN)? (AI 2006, MHPGM-CET 2010)
  1. Begins between 2-7 days of life
  2. Intracranial Hemorrhage is common
  3. Bilary atresia can predispose
  4. Warfarin therapy is associated
Ref: Care of Newborn by Maherbansingh 6/e p336; Ghai 8/e p137
106. Bronchiolitis obliterans is caused by: (Kerala PG 08)
  1. Respiratory syncytial virus
  2. Adeno virus
  3. H. influenza
  4. Mycoplasma
Ref: Ghai 8/e p381; Nelson textbook of Pediatrics, 17/e p1422; 18/e p1781
 
Ans.
1. c. No specific therapy
2. a. 90-110
3. d. Reassure the...
4. a, b and d
5. c. Constitutional
6. c. 9 months
7. c. 1.7
8. c. 2500 gms
9. d. New Ballards
10. b. Head control
11. d. 6 months
12. d. 6th month
13. b. Symmetrical tonic...
14. d. 20 weeks
15. a. Normal child
16. a. Normal child
17. a. Parachute reflex
18. c. Thick ear cartilage
19. a. < 10 percentile for...
20. a, b and c
21. a. Hypoglycemia
22. a. Less gestation age
23. All of above
24. d. Low birth weight
25. c. Less than 2500 gms
26. c. Less than 2500 gms
27. b. 1500
28. d. Is treated by...
29. b. Bradycardia
30. c. Maturity of lungs
31. b, d and e
32. a, b and d
33. a. Surfactant therapy...
34. a and b
35. a and b
36. b. Air bronchogram...
37. a. Develops 6 hours...
38. a. Nasal CPAP
39. a. Opioid
40. c. Congenitae...
41. c. Nasogastric tube...
42. b. Put a nasogastric...
43. c. Nasogastric tube...
44. a, b and d
45. a, b and c
46. d. X-ray chest...
47. c. Insert a nasogastric...
48. a. Neonatal alveolar...
49. c. Neonatal...
50. c. Neonatal pulmonary...
51. a. Check pulses
52. c. Intraosseous IV fluids
53. All of these
54. b. Radial artery
55. d. Heart rate
56. d. Stop chest...
57. d. 2:15
58. b. Control mechanical...
59. b. Adrenalin in 1:1000...
60. a. Barbiturates
61. d. Hemolytic jaundice
62. a and b
63. c, d and e
64. b. Congenital...
65. a. Usually occurs...
66. c. Convection
67. a and d.
68. a. Total and direct
69. b. Dubin Johnson...
70. c. Structural...
71. a. 20 mg%
72. b. Physiological...
73. b. Unconjugated...
74. a. Phototherapy
75. a. Hypotonia
76. b. Rh incompatibility
77. b. Caudal regression
78. a, b and d
79. b. Ketotic ...
80. b. Hyperglycemia
81. a. GDM
82. b. Dexamethasone
83. b. Cardiovascular...
84. a. Hypoglycemia
85. c. Hyperglycaemia
86. b, c and d
87. All of the above
88. c. G-csf
89. a, c and d
90. c. Elective cesarean...
91. c. Prominent hori...
92. d. Impedence technique
93. b and c
94. All of these
95. b and c
96. c. History of...
97. d. Post-term infant
98. b. Hypocalcemia
99. a. 1
100. C. CO2 washout
101. D. Lines of...
102. b. Do a 2nd sepsis...
103. d. Respiratory rate
104. a. Respiratory rate
105. a. Begins between 2-7...
106. a and b
 
132. GROWTH AND DEVELOPMENT
  • A. Normal Growth and Developmental Milestones
  • B. Adolescent
  • C. Miscellaneous
 
14GROWTH AND DEVELOPMENT (QUESTIONS)
 
A. NORMAL GROWTH AND DEVELOPMENTAL MILESTONES
1. True about head circumference measurement: (PGI Dec 04)
  1. Measured in supraorbital ridge
  2. Measures hydrocephalus/microcephaly
  3. Serial measurement is useful
  4. Helps in measurement of neurological development
  5. Paediatric intelligence
Ref: Ghai 8/e p19, 27; 7/e p15-17, 6/e p4; Nelson 18/e p54 and 17/e p1975; Manual of Neonatal Care 5/e, p50
2. A neonate is able to: (PGI Dec 00)
  1. Fix his gaze at a object 8 to 12 inches apart
  2. Focus on bright object
  3. Lift his head and chest on elbow
  4. Roll from side to side
Ref: Ghai 8/e p24; 7/e p18-19; Nelson 18/e p44 and 17/e p39
3. A 6-year-old child has an IQ 50. Which of the following tasks the child can do: (AI 07)
  1. Draw a triangle
  2. Recognize colors
  3. Ride a bicycle
  4. Read a sentence
Ref: Ghai 8/e p42; 7/e p562
4. A child has started mouthing objects, shows likes and dislikes has and not yet developed stanger anxiety. The age of child is: (AI 07)
  1. 3 months
  2. 5 months
  3. 7 months
  4. 9 months
Ref: Ghai 8/e p42; 7/e p30; Nelson 18/e p47 and 17/e p36
5. A two-month-old child is able to: (AI 04)
  1. Show a positive parachute protective reflex
  2. Hold head steady in seated position
  3. Lift head and chest off a flat surface with extended elbows
  4. Sustain head level with the body when placed in ventral suspension
Ref: Ghai 8/e p24; 7/e p23-28; Nelson 18/e p44 and 17/e p34, 1979
6. A female child has recently learned to eat with spoon without spilling; to dress and undress herself with supervision: and to understand that she is a girl. These skills are FIRST mastered between the ages of: (AIIMS Nov 05)
  1. 2 and 3 years
  2. 3 and 4 years
  3. 4 and 5 years
  4. 5 and 6 years
Ref: Ghai 8/e p42; 7/e p29-30
7. A normally developing 10-month-old child should be able to do all of the following except: (AIIMS Nov 05; AI 06)
  1. Stand alone
  2. Play peek a boo
  3. Pick up a pellet with thumb and index finger
  4. Build a tower of 3-4 cubes
Ref: Ghai 8/e p49, 50; 7/e p28-29
8. A child climbs with alternate steps, builds a tower of 8-9 cubes, tells 1 but not his name and cannot say his age and sex the probable age is: (AIIMS May 01)
  1. 36 months
  2. 24 months
  3. 30 months
  4. 48 months
Ref: Ghai 8/e p50; 7/e p30-31; Nelson 18/e p44 and 17/e p38-39
9. Developmental examination should be further evaluated in child of 12 weeks if the child: (AIIMS June 00; AI 02)
  1. Does not vocalize
  2. Does not babble
  3. Does not change bright red ring from one hand to other even if given in hand
  4. Does not hold head at 90°
Ref: Ghai 8/e p49; 7/e p26-29; Nelson 18/e p4417/e p39
10. 10-month-old child can not do: (PGI Dec 05; June 04)
  1. Change cube from one hand to another
  2. Can build a cube of 6
  3. Can pull and stand
  4. Can talk a sentence of 4 to 5 words
  5. Pincer grasp
Ref: Ghai 8/e p44; 7/e p26-29; Nelson 18/e p44 and 17/e p39
11. A 3-month-old can do: (PGI June 05; Dec 01)
  1. Social smile
  2. Can sit without support
  3. Transfer objects from right to left
  4. Hold his neck
  5. Can change position from prone to supine
Ref: Ghai 8/e p42; 7/e p26-29, 6/e p44; Nelson 18/e p44 and 17/e p39
12. 2-year-old child can do: (PGI Dec 04)
  1. Ride tricycles
  2. Climb up and down stairs with one each time
  3. Knows sex and age
  4. Handles spoon well
  5. Can read story with picture
Ref: Ghai 8/e p45; 7/e p26-29; Nelson 18/e p44 and 17/e p39
15
13. An 18-month-old infant can do all except: (PGI Dec 03)
  1. Climbing upstairs
  2. Can follow mother’s activities
  3. Can turn 2-3 pages at a time
  4. Can say two or three words
  5. Can make tower of 8 cubes
Ref: Ghai 8/e p44; 7/e p26-29; Nelson 18/e p44 and 17/e p 39
14. Which of the following cannot be done by 3-year-old child: (PGI Dec 02)
  1. Draw a triangle
  2. Draw a circle
  3. Arrange 9 cubes
  4. Go up and down stains
  5. Stand on one foot for 5 second
Ref: Ghai 8/e p44; 7/e p26-29; Nelson 18/e p44 and 17/e p39
15. Which of the following is the best indicator of longterm nutritional status of a child? (AIIMS May 2010)
  1. Weight
  2. Mid arm circumference
  3. Rate of weight gain and linear growth
  4. Head circumference
Ref: Ghai 8/e p21; 7/e p5, 6
16. All are seen in fetal alcohol syndrome except: (AIIMS Nov 2009)
  1. Microcephaly
  2. Poor coordination
  3. Overgrowth of body
  4. Flat face
Ref: Internet
17. A normal child develops the ability to use 10 words with meaning at the age of: (Karnatka PG 2011)
  1. 12 months
  2. 15 months
  3. 18 months
  4. 24 months
Ref: OP Ghai 8/e p55; 7/e p28
18. The weight of the child at 3 years is usually ______ times the birth weight. (Karnataka PG 2011)
  1. 3
  2. 4
  3. 5
  4. 7
Ref: Ghai 8/e p13; Child development: Birth to adolesence, Dr Rajesh Dixit, 2006 p38; Health, Safety and Nutrition for the young child, Lynn R Marotz, p26
19. Birth length is double at the age of: (UP PG 09)
  1. 1 year
  2. 2 years
  3. 3 years
  4. 4 years
Ref: Ghai 8/e p11; 7/e p6
20. X-ray of which of the following would be most appropriate for determining bone age at 7 months? (MHPGM-CET 2010)
  1. Shoulder
  2. Medial end of clavicle
  3. Wrist
  4. Knee
Ref: OP Ghai 8/e p11; 7/e p4
21. Child begins to sit with support, able to transfer objects from one hand to another hand and speak monosyllabic babbles at the age of: (MHPGM-CET 2010)
  1. 3 months
  2. 6 months
  3. 9 months
  4. 12 months
Ref: Ghai 8/ep49-55; Nelson paediatrics 18/e p44 Table 8-1
22. A boy can climb up and down stairs, One step at a time, Jump, build a tower of 10 cubes, Open a door, use 3 words in a sentences. What is his minimum age?
  1. 20 months
  2. 24 months
  3. 30 months
  4. 36 months
Ref: Nelson 18/e p45, Ghai 8/e p49-55
23. The milestones achieved at 13 months in children are all except: (DP PGMEE 2009)
  1. Index finger approach
  2. Walking
  3. Casting
  4. Single words
Ref: Ghai 8/e p49-55, Nelson 18/e p43-54
24. A 5-year-old child is assessed to have developmental age of one year. His developmental quotient would be: (DP PGMEE 2010)
  1. 100
  2. 80
  3. 60
  4. 20
Ref: Ghai 8/e p54
25. A female child has recently learned to eat with spoon without spilling; to dress and undress herself with supervision: and to understand that she is girl. These skills are FIRST mastered between the ages: (Feb DP PGMEE 2009)
  1. 2 and 3 years
  2. 3 and 4 years
  3. 4 and 5 years
  4. 5 and 6 years
Ref: Ghai 8/e p49-55
26. What milestone of 6 month old child can do? (Raj PGI 2008)
  1. Pick up of object between index finger and thumb
  2. Prone to supine
  3. Standing by holding the furniture
  4. Crawling
Ref: Ghai 8/e p49-55
16
27. Expected three times birth weight of baby at what age: (Raj PG 2009)
  1. 1 year
  2. 2 year
  3. 3 year
  4. 5 year (Ref: Ghai 8/e p13)
28. Membraneous gap seen between fetal skull bone is known as: (Kerala PG 10)
  1. Fontanelle
  2. Suture
  3. Wormian bone
  4. Craniosynostosis
Ref: Ghai 8/e p11; DC Dutta p84, Nelson 18/e p677
29. Which among the following is order of development is not characteristic: (Kerala PG 10)
  1. Caudal-Cranial
  2. Cranial -Caudal
  3. Generalised-Specific
  4. Distal-Proximal
Ref: OP Ghai 8/42
30. What is the age of child can walks up and down stairs is: (UP PG 2010)
  1. 2 years
  2. 3 years
  3. 5 years
  4. 6 years
Ref: Ghai 8/e p49-55; 7/e p27
31. The height of an average Indian child is 100 cms by this age: (J & K PG 2010)
  1. 2 ½ years
  2. 4½ years
  3. 8½ years
  4. 6½ years
Ref: OP Ghai 8/e p13
 
B. ADOLESCENT
32. The following are characteristics of autism except: (AI 06)
  1. Onset after 6 years of age
  2. Repetitive behavior
  3. Delayed language development
  4. Severe deficit in social interaction
Ref: Ghai 8/e p65; 7/e p40; Nelson 18/e p133 and 17/e p93
33. A 10-year-old child is always restless inattentive to study and always wants to play outside. Parents are extremely distressed, what would you advise? (AIlMS 08)
  1. It’s a normal behavior
  2. Behavior therapy
  3. It’s a serious illness requires medical treatment
  4. Needs change in environment
Ref: Ghai 8/e p63; 7/e p39
34. WHO defines adolescent age between: (AI 05)
  1. 10-19 years
  2. 10-14 years
  3. 10-25 years
  4. 9-14 years
Ref: Ghai 8/e p63
35. First sign of puberty in girls: (PGI Dec 99, AI 08)
  1. Puberchy
  2. Thelarchy
  3. Growth spurt
  4. Menarche
Ref: Ghai 8/e p64, 65; 7/e, p44
36. A 13-year-old boy has bilateral gynecomastia. His height is 148 cm, weight 58 kg; the sexual maturity rating is stage 2. The gynecomastia is most likely due to: (AIIMS Nov 04)
  1. Prolactinoma
  2. Testicular tumor
  3. Pubertal gynecomastia
  4. Chronic liver disease
Ref: Nelson 18/e p60-62; Ghai 8/e p65
37. Which of the following is the first sign of puberty in girls? (MHPGM-CET 2010)
  1. Pubarche
  2. Thelarche
  3. Menarche
  4. Growth spurt
Ref: OP Ghai 8/e p531-63; 7/e 498
 
C. MISCELLANEOUS
38. Which of the following is true about eruption ofteeth: (PGI Dec 05)
  1. Premolar appear in primary dentition
  2. Incisors appear first in secondary dentition
  3. 3rd molar is last to develop
  4. Hypothyroidism delays dentition
  5. Canines is last to appear in primary dentition
Ref: Ghai 8/e p12; CPDT 18/e p452; Nelson 18/e p47, 73
39. True about dentition: (PGI June 04)
  1. Hypothyroidism causes delayed dentition
  2. Premolar is not seen in primary dentition
  3. 3rd molar is the last to appear in secondary dentition
  4. Canine is the first in primary dentition
  5. Incisor is the first in secondary dentition
Ref: Ghai 8/e p12, 13; Nelson 18/e p57, 73, 74 and 17/e, p1204-1206
40. What is meant by “Suppositions” child: (PGI Nov 08)
  1. Second born of a twin pregnancy
  2. Child born out of wed lock
  3. Child rear up by a women and she claims the child
  4. Heterozygous mother and father
  5. Homozygous mother and father
Ref: Parikh 6/e p5.27
41. Which of the following is not true about the autistic specific disorders? (AI 2010)
  1. Impaired communication
  2. Impaired imagination
  3. Language development is delayed
  4. Vision problems
Ref: Nelson’s Pediatrics 17/e p93; Ahuja Psychiatry 6/e p263; OP Ghai 7/e p40
17
42. Stork bite lesion seen:
  1. Sturge weber syndrome
  2. KJippel fei! syndrome
  3. Biue rubber bleb nevus syndrome
  4. Macular staining of infants
  5. Craniofacial nevus
Ref: Nelson 18/e p2662
43. Aspergers syndrome is a: (JIPMER 2002)
  1. Developmental delay
  2. Neuromuscular disease
  3. Degenerative disorder
  4. Metabolic disorders
Ref: Ghai 8/e p45; Nelson 18/e p136
44. Pincer grasp is achieved at: (Karnatka PG 2010)
  1. 5 months
  2. 7 months
  3. 9 months
  4. 1 year
Ref: OP Ghai 8/e p53; 7/e p153
45. A child is below third percentile for height. Hia growth velocity is normal, but cgronogical age is more than skeletal age. The most likely diagnosis is: (DP PGMEE 2010)
  1. Constitutional delay in growth
  2. Genetic short stature
  3. Primordial dwarfism
  4. Hypopituitarism
Ref: Ghai 8/e p37
46. Perinatal Period corresponds to: (J & K PG 2011)
  1. 28 weeks of gestation to 07 days after birth
  2. Period of Labour to new born period
  3. Third trimester of pregnancy to new born period
  4. 36 weeks of gestation to 3 days after birth
Ref: Ghai 8/e p125
 
Ans.
1. a, b, c and d
2. b. Focus on bright...
3. b. Recognize colors
4. b. 5 months
5. d. Sustain head...
6. a. 2 and 3 years
7. d. Build a tower...
8. c. 30 months
9. a and d
10. b and d
11. a and d
12. d. Handles spoon well
13. a and e
14. a. Draw a triangle
15. c. Rate of weight...
16. c. Overgrowth of body
17. c. 18 months
18. c. 5
19. d. 4 years
20. a. Shoulder
21. b. 6 months
22. d. 36 months
23. c. Casting
24. d. 20
25. b. 3 and 4 years
26. b. Prone to supine
27. a. 1 year
28. a. Fontanelle
29. a. Caudal-Cranial
30. a. 2 years
31. b. 4½ years
32. a. Onset after...
33. b and d
34. a. 10-19 years
35. b. Thelarchy
36. c. Pubertal...
37. b. Thelarche
38. b, c and d
39. a, b, c and e
40. c. Child rear up...
41. d. Vision problems
42. e. Craniofacial nevus
43. a. Developmental delay
44. c. 9 months
45. a. Constitutional...
46. a. 28 weeks of gestation...
 
183. NUTRITION IN HEALTH AND DISEASE
  • A. Breast Milk/Breast Feeding
  • B. Nutritional Status Indictors
  • C. Protein Energy Malnutrition
  • D. Rickets
  • E. Vitamins and their Deficiencies
  • F. Miscellaneous
 
19NUTRITION IN HEALTH AND DISEASE (QUESTIONS)
 
A. BREAST MILK/BREAST FEEDING
1. The protective effects of breast milk are known to be associated with: (AI 05)
  1. IgM antibodies
  2. Lysozyme
  3. Mast cells
  4. IgA antibodies
Ref: Ghai 8/e p150; 7/e p123; Nelson 18/e p215 and 17/e p158
2. Breast milk storage in a refrigerator is upto: (PGI Dec 98)
  1. 4 hours
  2. 8 hours
  3. 12 hours
  4. 24 hours
Ref: Ghai 8/e p152
3. Exclusive breast feeding may be associated with the following except: (AI 98)
  1. Hemolysis due to Vit K deficiency
  2. Evening colic
  3. Golden color stool
  4. Prolongation of physiological jaundice
Ref: Ghai 8/e p152; Nelson 18/e p217 and 17/e p158, 164
4. Exclusive breast feeding is at least till: (AI 98)
  1. 4 months
  2. 6 months
  3. 8 months
  4. 10 months
Ref: Ghai 8/e p150; 7/e p122
5. Recommendation of WHO for baby friendly hospital is: (AIIMS June 98)
  1. With hold breast feed for 1 day after child birth
  2. Only breast feed, no other feed
  3. For 24 hours baby is kept with the mother
  4. Within 4 hours of birth, breast feed is started
Ref: Meharban Singh 6/e p167; Ghai 8/e p155
6. True about mother’s milk: (PGI Nov 09)
  1. Best even in preterm baby
  2. 30% energy from protein 10%
  3. Lactoferrin promotes lactobacilli in baby’s gut
  4. Prevents allergies in baby
  5. Prevents asthma
Ref: Ghai 8/e p150; 7/e p123
7. The current recommendation for breast feeding is that: (AI 04)
  1. Exclusive breast feeding should be continued till 6 month of age followed by supplementation with additional foods
  2. Exclusive breast feeding should be continued till 4 month of age followed by supplementation with additional foods
  3. Colostorum is the most suitable food for a newborn baby but it is best avoided in first two days
  4. The baby should be allowed to breast feed till one year of age
Ref: Ghai 8/e p152
8. Benifits of breast milk are: (PGI June 08)
  1. Better nutrition
  2. Less infection
  3. More diarrhea
  4. Less allergy
  5. High sodium contents
Ref: Ghai 8/e p150
9. True about cow’s milk are all except: (AIIMS May 07)
  1. Cow’s milk contains 80% whey protein not casein
  2. Cow milk has less carbohydrate than mothers milk
  3. Has more K4 and Na+ than infant formula feeds
  4. Has more protein than breast milk
Ref: Nutrition and child development Flizabeth 2/e p18; Ghai 8/e p148
10. Infant can breathe while sucking breast milk because of:
  1. Small wide tongue
  2. High place larynx
  3. Small pharynx
  4. Short soft palate
Ref: Ghai 8/e p374
11. Adequate breast feeding in baby indicates: (MP PG 2008)
  1. Baby sleep 2 hour after feeds
  2. Does not cry frequently
  3. Urine 5-7 times/day
  4. Stool 4 times/day
Ref: Ghai 8/e p151; 7/e p452
12. A drug excreted in breast milk is harmful to infant. But it is us, in some situations like: (MP PG 2008)
  1. Breast carcinoma
  2. HIV therapy
  3. Methotrexate therapy
  4. Morphine therapy
Ref: Ghai 8/e p183; 7/e p159
13. After premature delivery, mother’s milk is low in: (MHPGM-CET 2010)
  1. Lactose
  2. Fat
  3. Protein
  4. Sodium
Ref: OP Ghai 8/e p151; 7/e p131, 134
14. The Baby-friendly Hospital Initiative (BFHI) is not an initiative: (AP 2010)
  1. Every 4 hourly breast-feeding
  2. Give breast-feeding to infants with within 4 hrs of caesarean
  3. Initiate Breast-feeding within a half-hour of birth
  4. Encourage breast-feeding on demand
Ref: Ghai 8/e p151
15. The protective effects of breast milk are known to be associated with: (Feb DP PGMEE 2009)
  1. IgM antibodies
  2. Lysozyme
  3. Mast cells
  4. IgA antibodies
Ref: Ghai 8/e p150
20
16. Which among the following is the best method of breast feeding in an infant of HIV mother india: (Kerala PG 10)
  1. Breast feed till 6 months and abrupt weaning
  2. Breast feeding for one year
  3. Completely avoid breast feeding as there is risk of transmission
  4. Exclusive breast feeding as long as possible
(Ref: Ghai 8/e p183; Nelson 18/e p215, 1429)
17. Which major nutrient is richly present in mothers hind milk: (J and K PG 2010)
  1. Fat
  2. Protein
  3. Carbohydrate
  4. Water
Ref: OP Ghai 8/e p151
 
B. NUTRITIONAL STATUS INDICATORS
18. Nutrition in community is assessed by all except: (AI 09)
  1. Hb < 11.5 gm% in 3rd trimester pregnancy
  2. 1-4 yr mortality rate
  3. Ht and wt of children
  4. <BW < 2500 gm
Ref: Nelson, 18/e p229-232; PSM p236
19. Best indicator for nutritional status for a child is: (AIIMS Nov 06)
  1. Mid arm circumference
  2. Head circumference
  3. Rate of increase of height and weight
  4. Chest circumference
Ref: Ghai 8/e p32; 7/e p6-7; Nelson 18/e p225 and 17/e p170
20. Acute malnutrition is manifested by: (PGI June 05)
  1. Weight for age
  2. Weight for height
  3. Age for height
  4. Broca’s index
  5. Ponderal index
Ref: Ghai 8/e p110; 7/e p62; Park 18/e p402; 19/e, p343
21. The following statement about Gomez classification is false: (Al 08)
  1. Based on height retardation
  2. Based on 50th percentile Boston standards
  3. Between 75 and 89% implies in malnutrition
  4. This classification has progn and stip value for hospitalization children
Ref: Ghai 8/e p102
22. The most common cause of short stature is: (Al 08)
  1. Constitutional
  2. Systemic diseases
  3. Hypothyroidism
  4. Growth hormone deficiency
Ref: Ghai 8/e p12
23. The most common cause of short stature is: (AI 07, 08)
  1. Constitutional delay
  2. Growth hormone deficiency
  3. Cretinism
  4. Achondroplasia
Ref: Ghai 8/e p22; Nelson 18/e p1851 and 17/e p1851
24. Short stature is seen in: (PGI June 02)
  1. Maternal deprivation syndrome
  2. Hypothyroidism
  3. Bulimia
  4. Paternal smoking
  5. IUGR
Ref: Ghai 8/e p21
25. Common to both acute and chronic malnutrition is: (AIIMS May 07)
  1. Weight for age
  2. Weight for height
  3. Height for age
  4. BMI
Ref: Indian Academy of Pediatrics 3/e p127; Ghai 8/e p102
26. Deficit in weight for height in a 3 years old child indicates: (AIIMS Nov 05)
  1. Acute malnutrition
  2. Chronic malnutrition
  3. Concomitant acute and chronic
  4. Under weight
Ref: See above explanation; Ghai 8/e p98
27. Which of the following is the best indicator of long term nutritional status: (Al 07)
  1. Mid arm circumference
  2. Height for age
  3. Weight for age
  4. Weight for height
Ref: Ghai 8/e p97, Park 19/e, p434
28. Chronic malnutrition is judged by: (DNB 2009)
  1. Weight for age
  2. Height for age
  3. Weight for height
  4. BMI
Ref: Ghai 8/e p254; Essentials of Paediatric Nelson 4/e p71-75, 88, 68
 
C. PROTEIN ENERGY MALNUTRITION
29. Kwashiorkar is characterized by all of the following features except: (AI 99)
  1. Edema
  2. Patchy depigmentation of hair
  3. Fatty liver
  4. Fatty infiltration of pancreas
Ref: Ghai 8/e p104; 7/e p66-67; Robbin’s 7/e p448-449
30. Caloric supplementation required for a severely malnourished child (per kg-body weight) is: (AI 99)
  1. 100 cal/kg
  2. 125 cal/kg
  3. 150 cal/kg
  4. 175 cal/kg
Ref: Ghai 8/e p106; 7/e p70-71
31. The important fatty acid present in breast milk, which is important for growth is: (AIIMS Nov 06)
  1. Docosahexaenoic acid
  2. Palmitic acid
  3. Linoleic acid
  4. Linolenic acid
Ref: Ghai 8/e p98; Nelson 18/e p215 and 17/e p154
21
32. All of the following conditions are observed in Marasmus, except: (AIIMS May 05)
  1. Hepatomegaly
  2. Muscle wasting
  3. Low insulin levels
  4. Extreme weakness
Ref: Ghai 8/e p102; 7/e p66-67
33. A child is suffering from severe PEM. Calories to be given per kg of body weight to regain weight: (AIIMS June 99)
  1. 175-200 Kcal
  2. 150 Kcal
  3. 400 Kcal
  4. 100 Kcal
Ref: Ghai 8/e p108; 7/e p70
34. Protein efficiency ratio is: (AIIMS June 98)
  1. Biological value X Digestibility coefficient
  2. Nitrogen absorbed/Nitrogen intake
  3. Weight gain per unit of protein consumed
  4. Energy from protein/total energy from diet
Ref: Ghai 8/e p108; PSM by Deepak Mishra 3/e p305
35. Acute complications of PEM: (PGI June 06; Dec 02)
  1. Hypothermia
  2. Hypoglycemia
  3. Hypokalemia
  4. Hyponatremia
  5. Eosinophilia
Ref: Ghai 8/e p102; 7/e p68-70
36. Acute complications of PEM: (PGI June 05)
  1. Hypothermia
  2. Hypoglycemia
  3. Hypokalemia
  4. Hypermagnesemia
  5. Eosinophilia
Ref: Ghai 8/e p102; 7/e p68-70
37. Kwashiorkor is characterized by all except: (UP PG 09)
  1. Alertness
  2. Edema
  3. Flag sign
  4. Hepatomegaly
Ref: Ghai 8/e p99; 7/e p67
38. In Kwashiorkor, the letter ‘K’ is post-fixed to denote: (MHPGM-CET 2006, 2010)
  1. Weight for height
  2. Skin change
  3. Edema
  4. Muscle wasting
Ref: OP Ghai Pediatric 8/e p99
39. The skin changes seen in protein energy malnutrition can be due to deficiency all of the following nutrients except: (DP PGMEE 2009)
  1. Zinc
  2. Tryptophan
  3. Essential fatty acids
  4. Pyridoxine
Ref: Ghai 8/e p102
40. Mostly death in PEM is due to all except: (Feb DP PGMEE 2009)
  1. Hypothermia
  2. CCF
  3. Worm infestation
  4. Electrolyte imbalance
Ref: Ghai 8/e p102
41. All of the following conditions are observed in marasmus, except: (Feb DP PGMEE 2009)
  1. Hepatomegaly-Seen in Kwashiorkor
  2. Muscle wasting
  3. Low insulin levels
  4. Extreme weakness
Ref: Ghai 8/e p99
 
D. RICKETS
42. In Rickets seen: (PGI Dec 06)
  1. ↑ ALP
  2. ↓ ALP
  3. Hypo PO–2 4 in blood
  4. Hyper PO–2 4 in blood
  5. Hyperphosphaturia
Ref: Robbin’s 7/e p453; Ghai 8/e p113
43. Rickets in infant present as all except: (AIIMS May 07)
  1. Craniotabes
  2. Widened Fontanelle
  3. Rachitic Rosary
  4. Bow legs
Ref: Ghai 8/e p113; 7/e p82
44. A 2 years child with Vit. D resistant rickets was evaluated. His lab investigation reveal. Serum calcium - 9 mg/dl Serum phosphate - 2.4 mg/dl Serum alkaline phosphate - 1041 IU Parathormone level-59 units The likely diagnosis is: (AIIMS May 02)
  1. Distal renal tubular acidosis
  2. Vit. D dependent rickets
  3. Hypophosphetemic rickets
  4. Hyperparathyroidism
Ref: Ghai 8/e p113-117; 7/e p83; Nelson 17/e p2345
45. Basanti a 7 year old girl, presents with recent onset of genu valgum and difficulty in walking. On X-ray examination there is metaphyseal widening and osteoporosis. Investigations showed a serum calcium of 9 mg/dl. Serum phosphorus of 2.5mg/dl and alkaline phosphatase of 30Ka units, the possible cause is: (AIIMS Nov 00)
  1. Nutritional rickets
  2. Hypophosphatemic rickets
  3. Azotemic renal dystrophy
  4. Primary Hyperparathyroidism
Ref: Ghai 8/e p115; 7/e p83; Nelson 18/e p254 and 17/e p2345
46. True about vit - D deficiency rickets: (PGI Dec 04)
  1. Vit. D3 given at a dose of 50-150 ing/dl
  2. X-ray knee joint is diagnostic
  3. Rickety rosary is tender
  4. Increased chances of respiratory tract infection
  5. Hyponatremia
Ref: Ghai 8/e p113; 7/e p82-84
47. True about nutritional rickets: (PGI Dec 03)
  1. Craniotabes
  2. Multiple fracture
  3. Widening of wrist
  4. Phosphate in serum
  5. Growth retardation
Ref: Ghai 8/e p113; 7/e p82-84; Nelson 18/e p253-262 and 17/e p186-188
22
48. Familial hypoposphatemic Rickets is Characterized by: (PGI June 03)
  1. Increased forehead sweating
  2. Characteristically low Ca2+
  3. Anterior fontanelle widened
  4. Increased alk. Phosphatase
Ref: Ghai 8/e p113; 7/e p83; Nelson 18/e p254 and 17/e p2345-2346
49. Seen in rickets all except: (PGI June 98)
  1. Cupping of metaphysis
  2. Defective mineralization
  3. Epiphyseal dysgenesis
  4. Defective osteoid formation
Ref: Ghai 8/e p113, 114; 7/e p82-84; Nelson 17/e p186-188
50. All of the following are seen in rickets, except: (AIIMS May 03)
  1. Bow legs
  2. Gunstock deformity
  3. Pot belly
  4. Craniotabes
Ref: Ghai 8/e p114; Nelson 18/e p253-262 and 17/e p186-188
51. A common finding in osteomalacia is: (PGI June 99)
  1. Low serum phosphate
  2. Normal level of 1, 25 di-hydroxy vit D3
  3. Low serum calcium
  4. Increased hydroxy proline in urine
Ref: Nelson’s Essentials of Pediatrics 5/e, p152; 18/e p253-262; Ghai 8/e p113
52. A 2-year-old boy has vitamin D resistant rickets. His investigations revealed serum Calcium-9 mg/dl, Phosphate-2.4 mg/dl, alkaline phosphatase-1041IU, normal intact parathyroid hormone and bicarbonate 22 mEq/L. Which of the following is the most probable diagnosis? (AIIMS May 04; Nov 02)
  1. Distal renal tubular acidosis
  2. Hypophosphatemic rickets
  3. Vitamin D dependent rickets
  4. Hypoparathyroidism
Ref: Ghai 8/e p113; 7/e p83; Nelson 17/e p2342
53. Not seen in Rickets is: (DNB 2009)
  1. Craniotabes
  2. Bow legs
  3. Increased alkaline phosphatase
  4. Increased acid phosphatase
Ref: Op Ghai 8/ep230; Nelson Text book of Peds 18/e
54. True statement about vitamin D resistant rickets is: (DNB 2011)
  1. X linked recessive is commonest
  2. Defect in proximal tubule absorption
  3. Hyperphosphatemia
  4. No end organ resistance to 1, 25 (OH)2 D3
Ref: Ghai 8/e p238; Nelson Ped, 18/e p2720
55. 25 Hydroxylatio of Vit-D (Cholecalciferol) occurs in: (MP PG 2009)
  1. Liver
  2. Skin
  3. Kidney
  4. Intestine
Ref: Ghai 8/e p112; Nelson’s 18/e p2841
56. Earliest sign of Rickets is: (UP PG 09)
  1. Craniotabes
  2. Harrison’s groove
  3. Rachitic rosary
  4. Pigeon breast
Ref: Ghai 8/e p113; 7/e p18
 
E. VITAMINS AND THEIR DEFICIENCIES
57. About scurvy true all except: (PGI June 00)
  1. Subperiosteal hematoma with tenderness
  2. Separation of epiphysis
  3. Increased alkaline phosphatase
  4. Gingival bleeding
Ref: Ghai 8/e p110; 7/e p90-91; Nelson 18/e p243-250 and 17/e p184-186
58. Following are radiological features of scurvy except: (PGI Dec 99)
  1. Bony thickening
  2. Metaphyseal widening
  3. Metaphyseal calcification
  4. Epiphyseal separation
Ref: Ghai 8/e p112; 7/e p90-91; Nelson 18/e p243-250 and 17/e p184-186
59. All are true about scurvy except: (AIIMS May 09)
  1. Skeletal changes in adult occur with clinical deficiency of Vit.C
  2. Defective proximal calcification is the central cause for bone change
  3. Cartilaginous overgrowth results in widening of epiphyseal plate
  4. Bowing of legs
Ref: Ghai 8/e p110; 7/e p91
60. Pseudoparalysis in an infant is suggestive of: (PGI 06)
  1. Acute Rheumatic fever
  2. Vitamin B6 deficiency
  3. Vitamin E deficiency
  4. Vitamin C deficiency
Ref: Ghai 8/e p112; 7/e p91
61. Vitamin A deficiency is characterized by: (PGI Dec 04)
  1. Bitot spot
  2. Xerophthalmia
  3. Night blindness
  4. Tranta’s spot
Ref: Ghai 8/e p110, 112
62. Daily dose of vitamin A in a 6-12 months old child is: (AIMS June 97)
  1. 500 microgram
  2. 200 microgram
  3. 300 microgram
  4. 700 microgram
Ref: Ghai 8/e p120
63. The vitamin A supplement administered in “Prevention of nutritional blindness in children program” contain: (AI 03, AIIMS Nov 05)
  1. 25,000 lU/ml
  2. 1 Lakh Ill/ml
  3. 3 Lakh TU/ml
  4. 5 Lakh lU/ml
Ref: Ghai 8/e p110; IAP 1/e p417, Nutrition and child development KE Elezabeth 3/e p94
23
64. Which vitamin deficiency can result in Neonatal Seizures? (AIIMS Nov 09)
  1. Thiamine
  2. Pyridoxine
  3. Cyanocobalamin
  4. Vitamin-C
Ref: Harrison Principles of Internal Medicine, 17/e p2502; Ghai 8/e p182
65. Hypervitaminosis of which of the following will cause bony abnormalities: (PGI Dec 06)
  1. Vit A
  2. Vit D
  3. Vit C
  4. Vlt E
  5. Vit K
Ref: Ghai 8/e p118; 7/e p79, 81, 6/e p121, Nelson 18th p245
66. Large doses of vitamin K in new borne cause: (DNB 2011)
  1. Hemolysis
  2. Convulsions
  3. Decreased albumin
  4. Posterior fontanella buldging
Ref: KD Tripathi Pharmacology, 7/e p596; Gahi 8/e p410
67. Recommended daily allowance of vitamin D in children is: (MP PG 2010)
  1. 400 I.U Per day
  2. 600 I.U Per day
  3. 800 I.U Per day
  4. 1000 I.U Per day
Ref: Ghai 8/e p112; Nelson’s 18/e p257
68. The recommended oral dose of vitamin A to be given in a 10 month child with deficiency on each of day 1, 2 and 14 is: (Karnataka PG 2011)
  1. 50,000 IU
  2. 1,00,000 IU
  3. 2,00,000 IU
  4. 6,00,000 IU
Ref: OP Ghai 8/e p110; 7/e p80
69. Recommended daily dietary allowance of vitamin A for an infant is: (Karnataka PG 2010)
  1. 100-200 mg
  2. 300-400 mg
  3. 500-600 mg
  4. 750 mg
Ref: Ghai 8/e p110; Park 20/e p533, Table 8
70. Hypervitaminosis is damage to: (UP PG 09)
  1. Lysosome
  2. Mycrotubules
  3. Endoplasmic reticulum
  4. Mitochondria
Ref: Ghai 8/e p113; 7/e p80
71. Which one of the following is not a pyridoxine dependent disorder: (UPSC 07)
  1. Homocystinuria
  2. Methyl-malonic-acidemia
  3. Cystathioninuria
  4. Xanthurenic aciduria
Ref: Ghai 8/e p658; Nelson 17/e, 183
 
F. MISCELLANEOUS
72. A child with alopecia, hyperpigmentation psoriatic dermatitis in genitals and mouth and hypogonadism is likely to be suffering from: (AI 98)
  1. Cu deficiency
  2. Iron deficiency
  3. Zn deficiency
  4. Mg deficiency
Ref: Ghai 8/e p121; 7/e p92
73. Which of the following congenital malformation is seen in a child of a mother who is on treatment with oral anticoagulants: (AIIMS Nov 00)
  1. Craniofacial malformations
  2. Renal agenesis
  3. Long bone defects
  4. Chondrodysplasia punctate
Ref: Katzung 9/e, p997; Internet
74. True about Fructosan, a Prebiotic? (AI 2010)
  1. Dietary supplement containing potentially beneficial bacteria or yeast, with lactic acid bacteria
  2. Non-digestible food ingredient that beneficially affect the host by-selectively stimulating the growth of bacteria or yeast
  3. Digestible food ingredient that beneficially affect the host by selectively inhibiting the growth of bacteria
  4. Non-digestible food ingredient that selectively inhibit the growth of bacteria or yeast
75. Zinc deficiency causes: (PGI Dec 03)
  1. Sexual infant ilism
  2. Loss of libido
  3. Poor weight gain
  4. Poor wound healing
Ref: Nelson 18/e p266, Ghai 8/e p122; 7/e p92
76. Elemental iron and folic acid content of pediatric iron folic acid tablet supplied under RCH program: (AI 03)
  1. 20 mg iron and 100 microgram folic acid
  2. 40 mg iron and 100 microgram folic acid
  3. 40 mg iron and 50 microgram folic acid
  4. 60 mg iron and 100 microgram folic acid
Ref: Nutrition and child development by K.E Elizabeth 3/e, p111, 112; Ghai 8/e p123; 7/e p90
77. The protein derived from which of the following food items is considred as reference protein? (MP PG 2009)
  1. Egg
  2. Milk
  3. Meat
  4. Soya bean
Ref: Ghai 8/e p92; 7/e p57
78. The daily caloric requirement of a child weighing 15 kg is: (MP PG 2009)
  1. 750 kcal
  2. 1,000 kcal
  3. 1,250 kcal
  4. 1,500 kcal
Ref: Ghai 8/e p89
24
79. Calorie requirement per day of a child weighing 15kg would be: (MHPGM-CET 2010)
  1. 1150 kcal
  2. 1250 kcal
  3. 1450 kcal
  4. 1550 kcal
Ref: OP Ghai 8/e p88; 7/e p77
80. Which of the following is not given in the early nutritional correction of severe malnutrition: (Kerala PG 10)
  1. Zinc
  2. Iron-given after 4-6 weeks to avoid free radical injury
  3. Vit A
  4. Vitamin B Complex
Ref: Ghai 8/e p102; Nelson 18/e p232
 
Ans.
1. d. IgA antibodies
2. d. 24 hours
3. b. Evening colic
4. b. 6 months
5. b > c
6. a, c, d and e
7. a. Exclusive breast...
8. a, b and d
9. a. Cow’s milk contains..
10. b. High place larynx
11. c > a
12. b. HIV therapy
13. a. Lactose
14. a. Every 4 hourly...
15. d. IgA antibodies
16. c. Completely avoid...
17. a. Fat
18. a. Hb < 11.5 gm%...
19. c. Rate of increase...
20. b. Weight for height
21. a. Based on height...
22. a. Constitutional
23. a. Constitutional delay
24. a, b and e
25. b. Weight for height
26. a. Acute malnutrition
27. b. Height for age
28. b. Height for age
29. d. Fatty infiltration...
30. d. 175 cal/kg
31. a. Docosahexaenoic acid
32. a. Hepatomegaly
33. a. 175-200 Kcal
34. c. Weight gain per…
35. a, b, c and d
36. a, b and c
37. a. Alertness
38. c. Edema
39. d. Pyridoxine
40. c. Worm infestation
41. a. Hepatomegaly...
42. a, c and e
43. d. Bow legs
44. c. Hypophosphetemic...
45. b. Hypophosphatemic...
46. a and d
47. a, c, d and e
48. a, c and d
49. c and d
50. b. Gunstock deformity
51. a. Low serum...
52. b. Hypophosphatemic...
53. d. Increased acid...
54. b. Defect in proximal...
55. a. Liver
56. a. Craniotabes
57. c. Increased alkaline...
58. b. Metaphyseal widening
59. b. Defective proximal...
60. d. Vitamin C deficiency
61. a, b and c
62. c. 300 microgram
63. b. 1 Lakh Ill/ml
64. b. Pyridoxine
65. a and b
66. a. Hemolysis
67. a. 400 I.U Per day
68. b. 1,00,000 IU
69. b. 300-400 mg
70. a. Lysosome
71. b. Methyl-malonic-acidemia
72. c. Zn deficiency
73. d. Chondrodysplasia...
74. b. Non-digestible...
75. All of these
76. a. 20 mg iron and...
77. a. Egg
78. c. 1,250 kcal
79. b. 1250 kcal
80. b. Iron-given after...
 
254. FLUID AND ELECTROLYTES
 
26FLUID AND ELECTROLYTES (QUESTIONS)
1. The sodium content of ReSoMal (rehydration solution for malnourished children) is: (AI 06)
  1. 90 mmol/L
  2. 60 mmol/L
  3. 45 mmol/L
  4. 30 mmol/L
Ref: Ghai 6/e p109; Ghai 8/e p72, 105
2. A 5-year-old boy passed 18 loose stools in last 24 hours and vomited twice in last 4 hours. He is irritable but drinking fluids. The optional therapy for this child is: (AI 03)
  1. Intravenous fluids
  2. Oral rehydration therapy
  3. Intravenous fluid initially for 4 hours followed by oral fluids
  4. Plain water ad libitum
Ref: Ghai 8/e p70-75, 294; 7/e p264
3. Oral rehydration mixture contains glucose and sodium because both of them: (AIIMS Nov 04)
  1. Are needed to maintain the plasma osmolality
  2. Are prominent energy sources for the body
  3. Facilitate the transport of each other from the intestinal mucosa to the blood
  4. Are required for the activation of sodium potassium ATPase
Ref: Ghai 8/e p72, 294; 7/e p261-264
4. In a patient who has diarrhea and vomiting with inadequate water intake is suffering from: (AIIMS June 00)
  1. Intracellular dehydration with hypernatremia
  2. Intracellular dehydration with hyponatremia
  3. Extracellular dehydration with hyponatremia
  4. Extracellular dehydration with hypernatremia
Ref: Ghai 8/e p71, 294; 7/e p51-52
5. WHO ORS, composition are (mmol): (PGI June 04; AI 99)
  1. Glucose-111
  2. K+ - 80
  3. Na+ -20
  4. Cl- - 30
  5. Total osmolarity -311
Ref: Ghai Ghai 8/e p72, 294; 7/e p263; KDT. 5/e, p 617; PSM by Deepak Mishra 7/e p374 – 379
6. Composition of ORS, which of the following is correct: (PGI Dec 02)
  1. Na+ 90 mEq/L
  2. HCO3- 10 mEq/L
  3. K+ 20 mEq/L
  4. Cl- 5 mEq/L
  5. Osmolarity – 310
Ref: Ghai 8/e p294; 7/e p263; PSM by Deepak Mishra 7/e p374-379
7. WHO ORS contains: (PGI June 02)
  1. Sodium chloride 2.5g
  2. Potassium chloride 1.5g
  3. Glucose 20g
  4. Sucrose 10 g
  5. Potassium bicarbonate 2.5g
Ref: PSM by Deepak Mishra 7/e p374–379; Ghai 8/e p294
8. The composition of ORS recommended by WHO is: (PGI Dec 01)
  1. 3.5 g NaCl
  2. 4.5 g NaCl
  3. 2.9 g sodium-potassium citrate
  4. 2.8 g sodium bicarbonate
  5. 1.5 g potassium chloride
Ref: PSM by Deepak Mishra 7/e p374–379; Ghai p294, 295
9. True about ORS: (PGI Dec 00; Dec 06)
  1. Na+ = 90 mEq/L
  2. K+ = 30 mEq/L
  3. CI- - 20 mEq/L
  4. HCO3 = 40 mEq/L
  5. Glucose =111 gm
Ref: PSM by Deepak Mishra 7/e p374–379; Ghai 8/e p294
10. Kallu 2-year-child weighing 6.7 Kg presents in the casualty with history of vomiting and diarrhea for last 2 days. On examination skin pinch over the ant. abdominal wall go quickly to its original position. Interpretation of skin-pinch test in this child will be: (AI 02)
  1. No dehydration
  2. Some dehydration
  3. Sever dehydration
  4. Skin pinch cannot be evaluated in this child
Ref: Ghai 8/e p72; 7/e p264
11. A girl of 8 years suffering from vomiting and diarrhea for last 2 days when pinched on abdomen, skin goes within seconds, she is most likely to be suffering from: (AIIMS June 00)
  1. No dehydration
  2. Some dehydration
  3. Severe dehydration
  4. Skin turgor cannot be commented
Ref: Ghai 8/e p72; 7/e p264
12. Most sensitive indicator of intravascular volume depletion in infant is: (AIIMS May 2010)
  1. Stroke volume
  2. Heart rate
  3. Cardiac output
  4. Blood pressure
Ref: L Nelson Textbook of Pediatrics 18/e, p
13. Most dangerous dehydration is: (PGI June 98)
  1. Hyponatremic
  2. Hypernatremic
  3. Isonatremic
  4. Non-diarrheal cause
Ref: Ghai 8/e p73; 7/e p262
27
14. Hypernatremic dehydration is characterized by: (PGI Dec 03)
  1. Serum sodium > 150 mmol/L
  2. Signs of dehydration are minimal
  3. ECF volume led
  4. Rapid correction is required
  5. Shift of water from ECF to ICF
Ref: Ghai 8/e p72, 73; 7/e p261, 262
15. A child suffering from acute diarrhea is brought to the casualty and is diagnosed as having severe dehydration with pH of 7.23- Serum Na-125, Serum K-3, HCO3-16. The best IV fluid of choice is: (AIIMS May 01)
  1. 3% Saline
  2. N/3 Saline+ 10% dextrose
  3. Normal saline
  4. N/3 saline + 5% dextrose
Ref: Ghai 8/e p72; 7/e p266
16. The requirement of potassium in a child is: (AI 06)
  1. 1-2 mEq/kg/24 hours
  2. 4-7 mEq/kg
  3. 10-12 mEq/kg
  4. 13-14 mEq/kg
Ref: Nelson 18/e p279 and 17/e p203; Ghai 8/e p73
17. A child presents with diarrhea and peripheral circulatory failure. The arterial pH is 7.0, PC02 15 mm Hg, and PO2 76 mm Hg. What will be the most appropriate therapy: (AIIMS Nov 05; Nov 04; May 01)
  1. Sodium bicarbonate infusion
  2. Bolus of Ringers lactate
  3. Bolus of hydroxyethyl starch
  4. 5% Dextrose infusion
Ref: Ghai 8/e p85; 7/e p49-51, 90; Harrison 15/e, p286; 16/e p265; Nelson 18/e p301 and 17/e p299-301
18. Which of the following would be the plasma osmolality of child with plasma Na+ 125 mEq/L, glucose of 108 mg/dl, and blood urea nitrogen (BUN) of l40 mg/dl? (AIIMS May 05)
  1. 360 mOsm/kg
  2. 306 mOsm/kg
  3. 312 mOsm/kg
  4. 318 mOsm/kg
Ref: Nelson 18/e p270 and 17/e p193; Ghai 8/e p83
19. A child had repeated vomiting and developed metabolic alkalosis. The treatment given is: (AIIMS June 99)
  1. Ringer lactate
  2. IV normal saline and potassium
  3. ORS
  4. IV normal saline
Ref: Ghai 8/e p87; 7/e p50-51
20. In Pediatric advanced life support, interosseous access for drug/fluid administration is recommended for pediatric age of: (AIIMS Nov 02)
  1. < 1 year age
  2. < 5 years age
  3. < 6 years age
  4. Any age
Ref: Ghai 8/e p710; Textbook of Pediatric Advanced Life Support, Nelson 18/e p403 and 17/e p292; Bailey and Love 24/e p290
21. A breast fed child presents with hypernatremia (Serum sodium > 170m Eq/L). His urine sodium is 70 mEq/L. Which of the following is the most likely cause: (AIIMS Nov 00)
  1. Diabetes insipidus
  2. Acute necrosis
  3. Severe dehydration
  4. Excessive intake of sodium
Ref: Ghai 8/e p73; Nelson 18/e p273 and 17/e p197-198; Cloherty-Manual of Neonates Care 5/e, p104
22. Oral glucose tolerance test in children done with: (PGI Nov 08)
  1. 1.5 gm/kg glucose
  2. 1.75 gm/kg glucose
  3. 2 gm/kg glucose
  4. 2.5 gm/kg glucose
  5. 75 gm as an adult
23. An alert 6-month-old child is brought with vomiting and diarrhea. RR-45/min, HR-180/min, SBP-85 mm of Hg. xtremities are cold and mottled. Capillary refilling time is 4 secs. Diagnosis is: (AIIMS May 10)
  1. Early compensated shock due to hypovolemia
  2. Early compensated shock due to supraventricular tachycardia
  3. Late decompensated shock due to hypovolemia
  4. Late decompensated shock due to supra ventricular tachycardia
Ref: Ghai 8/e p715;1. Intermediate emergency care and transportation of the sick and injured Bv American Academy of Orthopaedic Surgeons; 2. Ghai Essential Pediatrics
24. The severely malnourished child is given: (DNB 2011)
  1. Dextrose
  2. Albumin
  3. Lactate
  4. Thiamine
Ref: OP Ghai 8/e p345; 7/e p69
25. The initial fluid of choice of the treatment of hypernatremic dehydration is: (MP PG 2009)
  1. Normal saline
  2. N/2 saline
  3. N/4 saline
  4. D/4 saline + 5% dextrose
Ref: Nelson’s 18/e p274; Ghai 8/e p73; 7/e p25, 54
26. Hyponatremia produces confusion and rstlessness at a serum level of: (MP PG 2009)
  1. < 130 Meq/L
  2. < 120 Meq/L
  3. < 100 Meg/L
  4. < 100 Meq/L
Ref: Ghai 8/e p74; 7/e p51
28
27. Sodium content in mmol/L in WHO oral rehydration solution is: (Karnataka PG 2011)
  1. 20
  2. 80
  3. 90
  4. 111
Ref: Ghai 8/e p294; 7/e p263, table 10.7.
28. Daily water requirement in child weighing 30 kgs, height 123 m and BSA of 1 m2 is: (DP PGMEE 2009)
  1. 1300 ml
  2. 1700 ml
  3. 2000 ml
  4. 2500 ml
Ref: Ghai 8/e p147
29. Factors which decrease insensible water losses are all, except: (DP PGMEE 2009)
  1. Humidified air
  2. Sedation
  3. Hypothemia
  4. Prematurity
Ref: Ghai 8/e p147; Internet
30. Hypercalciuria is said to be present in children if daily calcium excretion in urine is more than: (Kerala PG 08)
  1. 1 mg/kg
  2. 2 mg/kg
  3. 3 mg/kg
  4. 4 mg/kg
Ref: Ghai 8/e p521; Nelson, 17/e p1824, Table 5391-1, 18/e p2184, Table 547-3
31. Total body water of newborn is what % of body weight? (Kerala PG 08)
  1. 60 %
  2. 70 %
  3. 80 %
  4. 90 %
Ref: Ghai 8/e p147; Nelson, 17/e p191 Fig. 45-1, 18/e p267
32. Maintenance of fluid requirement in 23 kg girl is: (UP PG 2010)
  1. 1650-ml/day
  2. 1560-ml/day
  3. 1120-ml/day
  4. 1060-ml/day
Ref: Ghai 8/e p146; 7/e p51
33. Hyponatremia is defined as serum sodium: (J & K PG 2011)
  1. <110 mEq/L
  2. <120 mEq/L
  3. <130 mEq/L
  4. <140 mEq/L
Ref: Op Ghai 8/e p73
34. A 3 years old child presents with four days history of puffiness of face, fever and tea colored urine. During the course of his disease, he can have any of the complications except: (UPSC 06)
  1. Hypokalemia
  2. Hypertensive encephalopathy
  3. Acute renal failure
  4. Acidosis
Ref: Ghai 8/e p487; 7/e p446-447
35. Baby born at 33 weeks / 1.5 kg should be started on?
  1. Nil oral and IV fluids
  2. Oral nasogastric tube/alternate oral route
  3. IV fluids and oral feeding
  4. TPN
Ref: Nelson 18/e p706-707, O P Ghai 8/e p165; 7/e p132-33
 
 
Ans.
1. c. 45 mmol/L
2. b. Oral rehydration therapy
3. c. Facilitate the...
4. c. Extracellular dehydration...
5. a and e
6. a, c and e
7. b and c
8. a, c and e
9. a and e
10. d. Skin pinch...
11. b > c
12. b. Heart rate
13. b. Hypernatremic
14. a and b
15. c. Normal saline
16. a. 1-2 mEq/kg/24 hours
17. b. Bolus of Ringers lactate
18. b. 306 mOsm/kg
19. b. IV normal saline...
20. c. < 6 years age
21. d. Excessive intake...
22. b. 1.75 gm/kg glucose
23. a. Early compensated...
24. a. Dextrose
25. a. Normal saline
26. b. < 120 Meq/L
27. c. 90
28. b. 1700 ml
29. d. Prematurity
30. d. 4 mg/kg
31. c. 80 %
32. b. 1560-ml/day
33. c. <130 mEq/L
34. a. Hypokalemia
35. c. IV fluids and oral...
 
295. DISORDERS OF GASTROINTESTINAL SYSTEM
 
30DISORDERS OF GASTROINTESTINAL SYSTEM (QUESTIONS)
1. A newborn has dribbling after feeds. He has respiratory distress and froths at the mouth. Diagnosis is: (Al 01)
  1. Tracheoesophageal fistula
  2. Tetralogy of fallot
  3. Respiratory distress syndrome
  4. None of the above
Ref: Ghai 8/e p176; 7/e p151, Nelson 18/e p1543, 1544; Nelson 19/e p1262-1263
2. A newborn baby had normal APGAR score at birth and developed excessive frothing and choking on attempted feeds. The investigation of choice is: (AIIMS May 03)
  1. Esophagoscopy
  2. Bronchoscopy
  3. MRI chest
  4. X-ray chest and abdomen with the red rubber catheter passed per orally into esophagus
Ref: Ghai 8/e p176; 7/e p151, Nelson 18/e p1543, 1544; Nelson 19/e p1263
3. Most common biochemical abnormality in congenital hypertrophic pyloric stenosis: (AIIMS Nov 09, 06, 02)
  1. Hypokalemic metabolic alkalosis
  2. Hyperkalemic metabolic acidosis
  3. Hypokalemic metabolic acidosis
  4. Hyperkalemic metabolic alkalosis
Ref: Ghai 8/e p279; 7/e p253; Nelson 18/e p282-284, 318, 1555-1557 and 17/e p1229-1230 Nelson 19/e p1275
4. Congenital hypertrophic pyloric stenosis usually presents: (AIIMS May 04)
  1. Within 2 days after birth
  2. Around 1 week after birth
  3. Around 2 weeks after birth
  4. Around 2 months after birth
Ref: Ghai 8/e p279; 7/e p253; Nelson 18/e p318, 1555, 1556 and 17/e p1229-1230; Nelson 19/e p1275
5. A 40-year-old male presents with recurrent bouts of vomiting since 9 months because of pyloric obstruction. The compensatory biochemical change is: (AIIMS May 01)
  1. Respiratory alkalosis
  2. Respiratory acidosis
  3. Paradoxical aciduria with hyponatremia and hypochloremia
  4. Metabolic acidosis
Ref: Ghai 8/e p279; 7/e p253; Nelson 18/e p2301-2303, 275-278 and 17/e p1229-1230
6. A male infant presented with distension of abdomen shortly after birth with passing of less meconium. Subsequently a full-thickness biopsy of the rectum was performed. The rectal biopsy is likely to show: (AIIMS Nov 04, Nov 09)
  1. Fibrosis of submucosa
  2. Lack of ganglion cells
  3. Thickened muscularis propria
  4. Hyalinization of the muscular coat
Ref: Ghai 8/e p285; 7/e p256; Nelson 18/e p1565 and 17/e p1239-1240; Nelson 19/e p1285
7. Failure to pass meconium within 48 hours of birth in a newborn with no obvious external abnormality should lead to the suspicion of: (AIIMS Nov 02)
  1. Anal atresia
  2. Congenital pouch colon
  3. Congenital aganglionosis
  4. Meconium ileus
Ref: Ghai 8/e p285; 7/e p256; Nelson 18/e p1565-1567, 2564 and 17/e p1239-1240; Nelson 19/e p1285
8. Which one of the following is most suggestive of neonatal small bowel obstruction: (AI 03)
  1. Generalized abdominal distension
  2. Failure to pass meconium in the first 24 hours
  3. Bilious vomiting
  4. Refusal of feeds
Ref: Ghai 8/e p326; Nelson 19/e p1278
9. An eight-year-old boy had abdominal pain, fever with bloody diarrhea for 18 months. His height is 100 cms and weight is 14.5 kg. Stool culture was negative for known enteropathogens. The sigmoidoscopy was normal. During the same period, child had an episode of renal colic and passed urinary gravel. The mantoux test was 5 x 5 mm. The most probable diagnosis is: (AI 03)
  1. Ulcerative colitis
  2. Crohn’s disease
  3. Interstinal tuberculosis
  4. Strongyloidiasis
Ref: Ghai 8/e p304; Nelson 18/e p1580-1585 and 17/e p1248; Nelson 19/e p1300
10. In which of the following conditions the lead pipe appearance of the colon on a barium enema is seen? (AIIMS Nov 04)
  1. Amoebiasis
  2. Ulcerative colitis
  3. Tuberculosis of the colon
  4. Crohn’s involvement of the colon
Ref: Ghai 8/e p304; 7/e p277; Nelson 18/e p1580-1585 and 17/e p1248; Nelson 19/e p1301
11. Which of the followings includes inflammatory bowel disease in children: (PGI June 05)
  1. Celiac disease
  2. Tropical sprue
  3. Regional ileitis
  4. Cystic fibrosis
  5. Ulcerative colitis
Ref: Ghai 8/e p305; 7/e p277; Nelson 17/e p1248; Nelson 19/e p1300
31
12. A 12-yead-old girl has history of recurrent bulky stools and abdominal pain since 3 year of age. She has moderate pallor and her weight and height are below the 3rd percentile. Which of the following is the most appropriate investigations to make a specific diagnosis? (AllMS Nov 04)
  1. Small intestinal biopsy
  2. Badan Sadies
  3. 24 hrs fecal fat estimation
  4. Urinary d-xylose test
Ref: Ghai 8/e 301; Harrison 16/e p1770-1771; Nelson 19/e p1306
13. In celiac disease all except: (PGI June 00)
  1. Gliadin is cause
  2. Associated with HLA-B8
  3. Decreased villi to crypt ratio
  4. Increased brush border
Ref: Ghai 8/e p304, 301; 7/e p27; Robbin’s 7/e p843; Nelson 19/e p1308
14. The following cereals should be avoided in patients with celiac diseases, except: (AIIMS Nov 03)
  1. Wheat
  2. Barely
  3. Maize
  4. Rye
Ref: Ghai 8/e p304, 301; Nelson 18/e p1624; Robbins 7/e 843; Chandrasoma Taylor 3/e p591; Nelson 19/e p1310
15. The histological features of celiac disease include all of the foIlowing except: (AI02)
  1. Crypt hyperplasia
  2. Increase in thickness of the mucosa
  3. Increase in intraepithelial lymphocytes
  4. Increase in inflammatory cells in lamina propyria
Ref: Ghai 8/e p304, 301; Robbins 7/e p843; Nelson 18/e p1624; Nelson 19/e p1310
16. The most common genetic cause of liver disease in children is: (AI 02)
  1. Hemochromatosis
  2. Antitrypsin deficiency
  3. Cystic fibrosis
  4. Glycogen storage disease
Ref: Ghai 8/e p289; Robbin’s 7/e p911; Nelson 18/e, p1679 and 17/e p1323; Nelson 19/e p1393
17. Unconjugated hyperbilirubinemia seen in: (PGI Dec 08)
  1. Dubin-Johnson syndrome
  2. Rotor syndrome
  3. Gilbert’s syndrome
  4. Breast feeding
  5. Physiological jaundice
Ref: Ghai 8/e p312; 7/e p147-150; Nelson 19/e p1376
18. A child with jaundice being given food intravenously. Which of the following can occur out of it: (PGI June 08)
  1. Infection
  2. Malnutrition
  3. Diarrhea
  4. Hyperglycemia
  5. Metabolic acidosis
Ref: Ghai 7/e p147-150, 6/e p666; Nelson 19/e p1376
19. Unconjugated hyperbilirubinemia is seen in: (PGI Nov 07)
  1. Physiological jaundice
  2. Breast milk jaundice
  3. Gilbert syndrome
  4. Biliary atresia
  5. Rotor syndrome
Ref: Ghai 8/e p312, Harrison 16/e p1820;17/e p263; Nelson 19/e p1376
20. Conjugated hyperbilirubinemia in infancy seen in:
  1. Choledochal cyst [PGI Dec. 04]
  2. Extra hepatic biliary atresia
  3. Crigler-Najjar disease
  4. Gilbert disease
Ref: Ghai 8/e p315; 7/e p147–150; Nelson 19/e p1376, 1377
21. A neonate presents with jaundice and clay white stools. On liver biopsy giant cells are seen. Most likely diagnosis is: (AI 01, AIIMS Nov 01)
  1. Physiological jaundice
  2. Neonatal hepatitis with extrabiliary atresia
  3. Neonatal hepatitis with physiological jaundice
  4. Extrabiliary atresia
Ref: Nelson 19/e p874; 18/e p1671-1672
22. A neonate is being investigated for jaundice. A liver biopsy shows features of a “Giant Cell/ Neonatal hepatitis”. Which one of the following conditions usually result in this case? (AIIMS Nov 04)
  1. Congenital hepatic fibrosis
  2. Hemochromatosis
  3. Alpha-1-antitrypsin deficiency
  4. Glycogen storage disease Type 1
Ref: CPDT 18/e p644; Meharban Singh 6/e p257; Nelson 19/e p1393
23. In neonatal cholestasis, if the serum gammaglutamyl transpeptidase is more than 600IU/L the most likely diagnosis is: (AIIMS Nov 02) (AI 04, AIIMS Nov 06)
  1. Neonatal hepatitis
  2. Choledochal cyst
  3. Hypothyroidism
  4. Extrahepatic Biliary atresia
Ref: Ghai 8/e p356; Surgery of Liver, bileduct and pancreas in children by Edward Hoeard and Stringes 2/e, p112, Liver disorder in Childhood-Mowat, p83; Nelson 19/e p1385
24. In a child presenting with obstructive jaundice all are seen except: (AIIMS Nov 06)
  1. Gamma glutamyl transpeptidase
  2. Alkaline phosphatase.
  3. Glutamate dehydrogenase
  4. 5’Nucleotidase
Ref: Ghai 8/e p324
32
25. A 2-month-baby presents with history of jaundice, turmeric colored urine and pale stools since birth. Examination reveals liver span of l0 cm, firm in consistency and spleen of 3 cm. The most specific investigation for establishing the diagnosis would be: (AI 03)
  1. Liver function tests
  2. Ultrasound abdomen
  3. Preoperative cholangiogram
  4. Liver biopsy
Ref: Nelson’s 19/e p889; 18/e p1666; Meharban Singh 3/e p257
26. True about extrahepatic biliary atresia: (PGI June 03)
  1. Acholic stool
  2. Unconjugated hyperbilirubinemia
  3. Conjugated hyperbilirubinemia
  4. Absence of nucleoid in duodenum in HIDA scan
  5. Jaundice is presenting feature
Ref: Ghai 8/e p356; 7/e p293-294; Harrison 16/e p1815; Nelson 18/e p1671-1673 and 17/e p1317; Meharban Singh 6/e p257; Nelson 19/e p1385
27. A 7-year-old girl from Bihar presented with three episodes of massive hematemesis and melena. There is no history of jaundice. On examination, she had a large spleen, non-palpable liver and mild ascites. Portal vein was not visualized on ultrasonography. Liver function tests were normal and endoscopy revealed esophageal varices. The most likely diagnosis is: (AI 03)
  1. Kala azar with portal hypertension
  2. Portal hypertension of unknown etiology
  3. Chronic liver disease with portal hypertension
  4. Portal hypertension due to extrahepatic obstruction
Ref: Ghai 8/e p320; 7/e p289-290; Nelson 19/e p1415
28. Ramu, a 8 years old boy presents with upper GI bleeding. On examination, he is found to have splenomegaly; there are no signs of ascites, or hepatomegaly; esophageal varices are found on UGId. Most likely diagnosis is: (AI 01)
  1. Budd chiary syndrome
  2. Non cirrhotic portal fibrosis
  3. Cirrhosis
  4. Veno-occlusive disease
Ref: Ghai 8/e p319, 320; Harrison 16/e p1862-1863
29. Portal hypertension in children in India is commonly due to: [AIIMS Nov 03]
  1. Indian childhood cirrhosis
  2. Extrahepatic portal venous obstruction
  3. Idiopathic portal hypertension
  4. Hepatic out flow tract obstruction
Ref: Ghai 8/e p320; Nelson 18/e p1709
30. A child is brought by mother with HO massive hematemesis with HO drug intake previously with NSAIDs and on Rx. Associated with moderate splenomegaly diagnosis is: (PGI Dec 06)
  1. Esophageal varices
  2. Duodenal ulcer
  3. Drug induced gastritis
  4. Peptic ulcer
Ref: Ghai 8/e p319, 320; 7/e p290; Nelson 19/e p1376
31. In a child with active liver failure, the most important prognosis factor for death is: (AIIMS May 06)
  1. Increasing transaminases
  2. Increasing bilirubin
  3. Increasing prothrombin time
  4. Gram -negative sepsis
Ref: Ghai 8/e p322; Nelson 18/e p1705; Nelson 19/e p1376
32. A 6-month-old infant presents to the ‘diarrhea clinic’ unit with some dehydration. The most likely organism causing diarrhea is: (AIIMS Nov 03)
  1. Entamoeba histolytica
  2. Rotavirus
  3. Giardia lamblia
  4. Shigella
Ref: Ghai 8/e p297; 7/e p261; Nelson 19/e p1246
33. True about diarrhea: (PGI Dec 03)
  1. Defined as passage of 2-3 formed stool/day
  2. Blood mixed with mucous stool is defined as dysentery
  3. Rotavirus is the most common organism in children
  4. Persistent diarrhea is defined if duration is more than 21 days
Ref: Ghai 8/e p297; 7/e p261-263
34. Intractable diarrhea in children is caused by all except: (PGI June 00)
  1. Cystic fibrosis
  2. Giardiasis
  3. Secreting tumors
  4. Milk allergy
Ref: Ghai 8/e p297
35. Diarrhea in children is caused by: (PGI June 09)
  1. AIDS
  2. Nanvalle
  3. Rotavirus
  4. E . Coli
Ref: Ghai 8/e p297-299
36. A child is diagnosed to have anac gastroenteritis. The consulting pediatricion wants to send a stool sample to a lab which k 16-18 hrs. away. Which of the following medium he use to send this sample: (AIIMS Nov 2000)
  1. Charcol cotton bud
  2. Carry Blair medium
  3. Sterilized jar
  4. A medium with high CO2 content
Ref: Micro CP Baweja 4/e p210; Nelson 17/e p1274; 16/e p767
37. One of the intestinal enzymes that is generally deficient in children following an attack of severe infectious enteritis is: (AI 05)
  1. Lactase
  2. Trypsin
  3. Lipase
  4. Amylase
Ref: Ghai 8/e p298; 7/e p269-270; Nelson 18/e p1598, 1619 and 17/e p1268
33
38. A 6 months old baby with h/o bloody diarrhea of 2 days duration with abdominal distension and on examination the baby screams, diagnosis is: (PGI Dec 03)
  1. Intussusception
  2. HUS
  3. Appendicitis
  4. Ac. Enterocolitis
Ref: Ghai 8/e p287; Nelson 18/e, p1569-1570 and 17/e p1242; Nelson 19/e p1288
39. Infant with blood in stools mass in abdomen, diagnosis is: (PGI June 01)
  1. Intussusception
  2. Volvulus
  3. Idiopathic abdominal epilepsy
  4. Hirschsprung’s disease
Ref: Ghai 8/e p287; Nelson 18/e, p1569-1570 and 17/e p1235; Nelson 19/e p1289
40. Double bubble sign in children seen in all except: (PG1 Dec 04)
  1. Ladds band
  2. Annular pancreas
  3. Pancreatic pseudocyst
  4. Diaphragmatic hernia
Ref: Ghai 8/e p179; Nelson 18/e p1559; Nelson 19/e p595
41. When severe dehydration in a neonate occurs, amount of fluid replacement in 1st hour: (CUPGEE 01)
  1. 20 – 40 mg/kg
  2. 5 – 10 mg/kg
  3. 10 – 15 mg/kg
  4. 15 – 20 mg/kg
Ref: Ghai 8/e p73, 104; 7/e p264
42. 14-year-old girl with history of abdominal pain (periumbilical), postprandial, passing blood in stools, fever, weight loss since ten months. She also has episodes of passing blood in stools. What may be the likely diagnosis? (St. Johns 02)
  1. Chronic appendicitis
  2. Chronic pancreatitis
  3. Crohn’s disease
  4. Bulimia
Ref: Ghai 8/e p304; 7/e p277; Nelson 17/e p1252
43. Acquired megacolon in children most commonly due to: (SGPGI 05)
  1. Psychological problems
  2. Bad bowel habit
  3. Chaga’s disease
  4. Hirschusprung’s disease
Ref: Ghai 8/e p285; Bailey and Love 24/e p1156; Bailiy
44. X-ray detect congenital anorectal malformation at: (UP 07)
  1. Immediately after birth
  2. 24-48 hours
  3. 48-72 hours
  4. After 72 hours
Ref: OP Ghai 6/e p178
45. A young boy presents with failure to thrive. Biochemical analysis of a duodenal aspirate after a meal reveals a deficiency of enteropeptidase (enterokinase)~ the levels of which one of the following digestive enzymes would be affected? (UP SC 07)
  1. Amylase
  2. Pepsin
  3. Lactose
  4. Trypsin
Ref: Guyton 10/e p346
46. Portal hypertension in children in India is commonly due to: (AIIMS Nov 03)
  1. Indian childhood cirrhosis
  2. Extrahepatic portal venous obstruction
  3. Idiopathic portal hypertension
  4. Hepatic out flow tract obstruction 7 K
Ref: Ghai 8/e p470; 7/e p319-320; Nelson 18/e p2082-2084 and 17/e p1670-1671
47. To induce vomiting at home in a child who has ingested a poison, the recommended agent of choice would be: (COMEDK 05)
  1. Oral rehydration solution
  2. Mustard in warm water
  3. Apomorphine
  4. Syrup of ipecac
Ref: Ghai 8/e p820; KD Tripathi 5/e p600
48. Baby born at 30 weeks for 18 years old primi gravida of weight 2 kg which died after 48 hours. APGAR scores were 5 and 8 at 1 and 5 minutes. On autopsy bilateral enlarged kidney with multiple radially arranged cysts. Which of the following finding is expected to be associated with? (AIIMS June 08)
  1. Imperforate anus
  2. Hepatic cyst and fibrosis
  3. Absence of ureter
  4. Holoprosencephaly
Ref: Ghai 8/e p316; Nelson 18/e p1706-1707; Nelson 19/e p1796, 1797
49. Congenital Wilson’s disease is characterized by: (SGPGI 05)
  1. KF ring is present at birth
  2. May present as acute hepatitis
  3. Decreased Urinary copper excretion
  4. Decreased hapatic copper concentration
Ref: Ghai 7/e p641, 6/e p618; Nelson 17/e p1322
50. Childhood cholelithiasis is seen in: (AIIMS June 98)
  1. Hurler Syndrome
  2. Mucopolysaccharidosis
  3. Neimann-Pick’s disease
  4. Autoimmune hepatitis
Ref: Ghai 8/e p659-660; 7/e p639; Nelson 18/e p602 and 17/e p464; Nelson 19/e p488
51. True about Wilson’s disease? (AI 2010)
  1. Increase in urinary copper and increased serum ceruloplasmin and copper
  2. Increased serum ceruloplasmin levels with increased urinary copper
  3. Elevated hepatic copper level and increased serum ceruloplasmin levels
  4. Increase in urinary copper and decreased serum ceruloplasmin
Ref: Ghai 8/e p676; 7/e p292, 640; Harrison’s 17/e p2451; Nelson 19/e p1391
34
52. A 12 year old girl with the mood and emotional liability has a golden brown discoloration in descement membrane. Most likely diagnosis is: (AI 04)
  1. Fabry’s disease
  2. Wilson’s disease
  3. Glycogen storage disease
  4. Acute rheumatic fever
Ref: Ghai 8/e p676; 7/e p640; Nelson 17/e p1321-1322; Nelson 19/e p13913
53. Congenital Wilson’s disease is characterized by: (SGPGI 05)
  1. KF ring is present at birth
  2. May present as acute hepatitis
  3. Decreased Urinary copper excretion
  4. Decreased hepatic copper concentration
Ref: OP Ghai 8/e p676; 7/e p632, 6/e p618
54. Earliest symptoms of GERD in an infant is?
  1. Respiratory distress
  2. Upper GI bleed
  3. Regulation
  4. Obstruction
Ref: Ghai 8/e p279; Nelson 18/e p1547
55. Central dot sign seen in:
  1. Caroli disease
  2. Primary sclerosing cholangitis
  3. Liver hematoma
  4. None of above
Ref: Internet
56. One of the intestinal enzymes that is generally deficient in children following an attack of severe infectious enteritis is: (DNB 2007)
  1. Lactase
  2. Trypsin
  3. Lipase
  4. Amylase
Ref: Ghai 8/e p512; Nelson, 16/e p318
57. Most common malignancy of GIT in childhood is: (DNB 2009)
  1. Carcinoid
  2. Lymphoma
  3. Adenocarcinoma
  4. Sarcoma
Ref: 8/e p499; Nelson 18/e p1643
58. Most common cause of gastroenteritis in infant is: (DNB 2011)
  1. Salmonella
  2. Rota virus
  3. Norwalk virus
  4. Coli
Ref: Ghai 8/e p610
59. Hepatomegaly, Hypoglycemia and Ketosis in a 5 year old is a feature of: (DNB 2011)
  1. Fat storage disease
  2. Glycogen storage disease
  3. Diabetes insipidus
  4. Mucopolysachridoses
Ref: Ghai 8/e p610; Essentials of Peds, Nelson, 4/e p170, Campbell Tesxt Book Of Peds
60. A child si hepatitis B immunized the marker for seroconversior: (MP PG 2008)
  1. HBs Ag
  2. Anti HBc IgM
  3. Anti Hbs Ag
  4. Anti HBc IgC
Ref: Ghai 8/e p221; 7/e p193
61. Most common bacteria causing diarrhea in children in India is: (Karnataka PG 2011)
  1. Enterotoxigenic e. coli
  2. Enteroinvasive e. coli
  3. Enterohemorrhagie e. coil
  4. Enteropathogenic e. coil
Ref: OP Ghai 8/e p299; 7/e p261
62. Unconjugated hyperbilirubinemia of newborn is caused by all except: (Karnatka PG 2010)
  1. Criggler-Najjar syndrome
  2. Cretinism
  3. Septicemia
  4. Breast milk jaundice
Ref: Ghai 8/e p718; Robbin’s Pathology, 7/e p887, Table 18-3; Nelson’s Essentials of Pediatrics, 6/e p317; Oski’s Pediatrics, 4/e p238
63. All of the following cause jaundice at birth or within 24 hours except: (Karnataka PG 2010)
  1. Physiological jaundice
  2. Hemolytic disease of the newborn
  3. Criggler-Najjar syndrome
  4. Intrauterine infections (TORCH)
Ref: OP Ghai 8/e p172; 7/e p148
64. All are true about physiological jaundice except: (UP PG 09)
  1. 60% patients presents within 1st week
  2. Cord indirect bilirubin level is 1.3 mg/dL
  3. Resolved within one week
  4. Persist beyond 10 days of jaundice
Ref: Ghai 8/e p172; 7/e p147-48
65. Budd Chiari syndrome as it is not associated with following one: (AP 2011)
  1. Leukemia
  2. Sickle cell disease
  3. Polycythemia
  4. Congestive heart failure
Ref: Ghai 8/e p320
66. Newborn-frothing mouth – unable to passing tube – gasless abdomen: (AP 2011)
  1. Oesophageal atresia with no Fistula
  2. Oesophageal atresia with Proximal Fistula
  3. Oesophageal atresia with distal Fistula
  4. Oesophageal atresia with double Fistula
Ref: Ghai 8/e p176
35
67. In neonetes with duodenal atresia, all the following statements are TRUE, except: (AP 2012)
  1. Increased incidence is seen in Down’s syndrome
  2. Incidence of other defects is reported
  3. Prenatal diagnosis by ultrasound is not possible
  4. The operative treatment of choice is duodeno-duode-nostomy
Ref: Ghai 8/e p309
68. Regarding splenomegaly, one of the following statements is false: (AP 2012)
  1. Spleen tip may be palpable in 15 percent of normal neonates and 10 percent of normal children
  2. To become palpable below the costal margin on abdominal examination, it must be twice or thrice its normal size
  3. Splenomegaly is a characteristics finding in aplastic anemia and idiopathic thrombocytopenic purpura
  4. Childhood osteopetrosis and prehepatic portal hypertension have significantly large spleen on examination
Ref: Ghai 8/e p310
69. Which one of the following hepatitis viruses have significant prenatal transmission? (Feb DP PGMEE 2009)
  1. Hepatitis E virus
  2. Hepatitis C virus
  3. Hepatitis B virus
  4. Hepatitis A virus
Ref: Ghai 8/e p183
70. A Child is 18 month old presented with vomiting without bile the diagnosis is: (Raj PG 2008)
  1. Pyloric stenosis
  2. Duodenal atresia
  3. Congenital megacolon
  4. All of the above
Ref: Ghai 8/e p279
71. All are true of GERD except: (Kerala PG 08)
  1. Antacid
  2. Prone end elevated
  3. Head end elevated
  4. Thinning of infant feed formula
Ref: Nelson, 17/e p1223; Bailey and Love, 24/e end
72. Which of the following is not a feature of infant colic syndrome: (Kerala PG 09)
  1. Vomiting
  2. Paroxysmal Symptoms
  3. Abdominal pain
  4. Continuous severe cry
Ref: Ghai 8/e p278; Nelson 18/e p222
73. Two month old baby presented with non bilous vomiting and a palpable epigastric lump. Which among the following will be investigation of choice: (Kerala PG 10)
  1. USG abdomen
  2. X ray abdomen
  3. Upper GI Series
  4. CT abdomen
Ref: Ghai 8/e p279; Nelson 18/e p1555
74. Hutchinson teeth are seen in: (UP PG 2010)
  1. Toxoplasma
  2. Congenital syphilis
  3. Rubella
  4. CMV-infection
Ref: Nelson 18/e p1265
75. Persistent diarrhea is defined as diarrhea that begins as an acute episode and lasts for at least: (J & K PG 2011)
  1. 7 days
  2. 10 days
  3. 14 days
  4. 21 days
Ref: OP Ghai 8/e p297
76. The causes of chronic diarrhea in children may be because of etiological factors given below except: (J & K PG 2010)
  1. Giardiasis
  2. Rotavirus
  3. Lactose intolerance
  4. Celiac diseases
Ref: OP Ghai 8/e p198
 
 
Ans.
1. a. Tracheoesophageal...
2. d. X-ray chest...
3. a. Hypokalemic...
4. c. Around 2 weeks...
5. c. Paradoxical aciduria...
6. b. Lack of ganglion...
7. c. Congenital...
8. c. Bilious vomiting
9. b. Crohn’s disease
10. d. Crohn’s involvement...
11. c and e
12. a. Small intestinal...
13. b. Associated with...
14. c. Maize
15. b. Increase in thickness...
16. b. Antitrypsin deficiency
17. c, d and e
18. a, d and e
19. a, b and c
20. a and b
21. b. Neonatal hepatitis...
22. c. Alpha-1-antitrypsin...
23. d. Extrahepatic Biliary...
24. c. Glutamate...
25. c. Preoperative...
26. a, c, d and e
27. d. Portal hypertension...
28. b. Non cirrhotic...
29. b. Extrahepatic...
30. a. Esophageal varices
31. c. Increasing...
32. b. Rotavirus
33. b and c
34. b. Giardiasis
35. c and d
36. b. Carry Blair medium
37. a. Lactase
38. a. Intussusception
39. a. Intussusception
40. d. Diaphragmatic
41. a. 20 – 40 mg/kg
42. c. Crohn’s disease
43. c. Chaga’s disease
44. b. 24-48 hours
45. d. Trypsin
46. b. Extrahepatic portal...
47. d. Syrup of ipecac
48. b. Hepatic cyst...
49. b. May present as...
50. c. Neimann-Pick’s...
51. d. Increase in urinary
52. b. Wilson’s disease
53. b. May present as...
54. a. Respiratory distress
55. a. Caroli disease
56. a. Lactase
57. b. lymphoma
58. b. Rota virus
59. b. Glycogen storage...
60. c. Anti Hbs Ag
61. a. Enterotoxigenic...
62. c. Septicemia
63. a. Physiological jaundice
64. d. Persist beyond...
65. d. Congestive heart...
66. a. Oesophageal atresia...
67. c. Prenatal diagnosis...
68. c. Splenomegaly...
69. c. Hepatitis B virus
70. a. Pyloric stenosis
71. b and d
72. c. Abdominal pain
73. a. USG abdomen
74. b. Congenital syphilis
75. d. 21 days
76. b. Rotavirus
 
366. IMMUNIZATION, INFECTIOUS DISEASES AND WORM INFESTATION
 
37IMMUNIZATION, INFECTIOUS DISEASES... (QUESTIONS)
1. Which of the following is not a common manifestations of congenital Rubella: (AI 02)
  1. Deafness
  2. PDA
  3. Aortic stenosis
  4. Mental retardation
Ref: Ghai 8/e p213; Nelson 18/e p1340 and 17/e p569, 1033; Nelson 19/e p1077
2. All are true about congenital rubella except: (AIIMS Nov 08)
  1. IgG persists for more than 6 months
  2. IgM antibody is present at birth
  3. Most common anomalies are hearing and heart defects
  4. Increased congenital malformation if infection after 16 weeks
Ref: Ghai 8/e p213; Nelson 18/e p1340; Nelson 19/e p1076-1077
3. Congenital rubella manifestations are all except: (PGI June 00)
  1. Rash appears first on trunk
  2. Pre-auricular lymph nodes
  3. Arthralgia
  4. Retinopathy
Ref: Ghai 8/e p213; Nelson 18/e p1337-1371 and 17/e p1032–1034; Nelson 19/e p1076
4. Symptomatic neonatal CNS involvement is most commonly seen in which group of congenital intrauterine infections: (AI 09)
  1. CMV and Toxoplasmosis
  2. Rubella and Toxoplasmosis
  3. Rubella and HSV
  4. CMV and Syphillis
Ref: Ghai, 8/e p214; Nelson 18/e p1340; Nelson 19/e p1077
5. Congenital Rubella Syndrome in associated with anomalies? (PGI Nov 09)
  1. VSD
  2. ASD
  3. PDA
  4. Epsteni’s
  5. Myocarditis
Ref: Ghai 8/e p213; Robbins Pathology 7/e p473 Nelson 19/e p1077
6. True about chicken pox: (PGI June 01)
  1. I.P. 2-3 days
  2. Scabs are infective
  3. Centrifugal rash
  4. Rash appears on first day
  5. Rash can occur in axilla
Ref: Ghai 8/e p215; Nelson’s 18/e p1767-1369; Nelson 19/e p1105
7. M/C complication of chicken pox in children: (PGI June 00)
  1. Encephalitis
  2. Secondary bacterial infection
  3. Pneumonia
  4. Otitis media
Ref: Ghai 8/e p215; 7/e p186; Harrison 16/e p1043; Nelson 19/e p1109
8. True about measles: (PGI Dec 05)
  1. Rash appears first on leg
  2. Koplik spots are seen in retina
  3. Long-term complication follows in the form of SSPE
  4. Caused by RNA virus
  5. IP is 2 - 3 days
Ref: Ghai 8/e p213-215; 7/e p185; Harrison 16/e p485; Nelson 19/e p1072
9. SSPE (subacute sclerosing panencephalitis) is associated with: (PGI June 01)
  1. Mumps
  2. Chicken pox
  3. Herpes
  4. Measles
Ref: Ghai 8/e p214; 7/e p185; Harrison 16/e p485; Nelson 19/e p1072
10. With reference to mumps which of the following is true? (AI 06)
  1. Meningoencephalitis can precede parotitis
  2. Salivary gland involvement is limited to the parotids
  3. The patient is not infectious prior to clinical parotid enlargement
  4. Mumps orchitis frequency leads to infertility
Ref: Ghai 8/e p213; 7/e p187-188; Nelson 19/e p1080
11. Which of the following is true of mumps? (AIIMS May 05)
  1. Salivary gland involvement is limited to the parotids
  2. The patient is not infectious prior to clinical parotid enlargement
  3. Menigoencephalitis can precede parotitis
  4. Mumps orchitis frequently leads to infertility
Ref: Ghai 8/e p213; 6/e p209-210; Nelson 18/e p395-405; Nelson 19/e p1080
12. An 8-years-old female child following URTI developed maculopapular rash on the jaw spreading onto the trunk which cleared on the 3rd day without desquamation and tender post auricular and suboccipital lymphadenopathy. The diagnosis is:
  1. Kawasaki disease (AIIMS May 01)
  2. Erythema infectiosum
  3. Rubella
  4. Measles
Ref: Ghai 8/e p213, 214; Nelson 18/e p1337-1341 and 17/e p1032; Nelson 19/e p1076
13. A patient had fever and coryza for last 3 days developed maculopapular erythematous rash with which lasted for 48 hours and disappeared without leaving behind pigmentation is most commonly due to: (AIIMS June 00)
  1. Measles
  2. Typhoid
  3. Roseola infantum
  4. Fifth disease
Ref: Ghai 8/e p213, 207, 229; Nelson 19/e p1118
38
14. Child (girl) is suffering from varicella (fever rash). And child’s aunt is pregnant. When is it earliest that the child can meet her aunt: (PGI Dec 06)
  1. When the lesions have crusted
  2. Immediately
  3. Anytime as the child is aunt’s favourite
  4. After the delivery of the baby
Ref: Ghai 8/e p215; 7/e p186; Nelson 19/e p1105
15. Which of the following is true about Roseola infantum: (PGI Dec 05)
  1. Defervescence follows the rash
  2. Caused by HHV 6 and 7
  3. Slapped cheek appearance is seen
  4. Otitis media is common complication
  5. Rash appears first on face and neck
Ref: Ghai 8/e p213, 207, 229; Nelson 19/e p1117
16. Which of the following is true about erythema infectiosum: (PGI Dec 05)
  1. Slapped cheek appearance seen
  2. Caused by parvovirus
  3. Defervescence before rash
  4. Rash appears on head and neck
Ref: Ghai 8/e p213, 214; Nelson 19/e p1117
17. True about Roseola infantum: (PGM June 03)
  1. Also called 5th disease
  2. Causedby HHV 6 and 7
  3. Rash appear in trunk
  4. During defeverescence, rash appears
Ref: Ghai 8/e p213; See previous questions; Nelson 19/e p1117
18. A 3-month-old male infant developed otitis media for which he was given a course of Cotrimoxazole. A few days later, he developed extensive peeling of the skin; there were no mucosal lesions and the baby was not toxic. The most likely diagnosis is: (AIIMS May 04)
  1. Toxic epidermal necrolysis
  2. Staphylococcal scalded skin syndrome
  3. Steven Johnson syndrome
  4. Infantile pemphigus
Ref: Pasricha 4/e p162, 163, Ghai 8/e p687; Nelson 19/e p2302
19. A 10-month-old child presents with two weeks history of fever, vomiting and alteration of sensorium cranial CT scan reveals basal exudates and hydrocephalus, the most likely etiological agent is: (AI 04)
  1. Mycobacterium tuberculosis
  2. Cryptococcus neoformans
  3. Listeria monocytogenes
  4. Streptococcus pneumonia
Ref: Ghai 8/e p250; 7/e p539; Nelson 18/e p1240-1254 and 17/e p966; Nelson 19/e p1005
20. Management of a newborn when mother has active tuberculosis and is taking ATT: (AI 00)
  1. BCG + Rifampicin + INH + Breast Feeding
  2. BCG +Isolation of baby
  3. BCG +INH for 6 week + Breast Feeding
  4. BCG + INH +withhold Breast Feeding
Ref: Ghai 8/e p250; 7/e p166; Nelson 17/e p971-972; Nelson 19/e p1007, 1011
21. Drugs included in ATT for children: (PGI June 06)
  1. Streptomycin
  2. Ethionamide
  3. Ethambutol
  4. Pyrazinamide
  5. Ofloxacin
Ref: Ghai 8/e p250, 251; 7/e p210-212]
22. First line ATT in children: (PGI June 05)
  1. Streptomycin
  2. Pyrazinamide
  3. Ethionamide
  4. Ethambutol
  5. Ofloxacin
Ref: Ghai 8/e p250; 7/e p217
23. 20 years old Rajesh presence with fever and chronic cough ESR 35 mm sputum cytology is negative for AFB and tuberculin test show indurations of 19 X 23 mm. The probable diagnosis is: (AIIMS Nov 01)
  1. Fungal pneumonia
  2. Viral pneumonia
  3. Pulmonary TB
  4. Bacterial pneumonia
Ref: Ghai 8/e p250; CMDT 2002/e p310, API 6/e p244
24. Pertusis affects which age: (PGI June 2000)
  1. 2-3
  2. <5
  3. 5-7
  4. > 10 years
Ref: Ghai 8/e p240; 7/e, 221; Nelson 19/e p945
25. All of the following strategies are effective in preventing mother to child transmission of HIV, except: (AIIMS Nov 03)
  1. Zidovudine to mother and baby
  2. Vaginal cleansing before delivery
  3. Stopping breast feeding
  4. Elective cesarean section
Ref: Ghai 8/e p229; 7/e p204-207; Harrison 16/e p1082; Nelson 19/e p1159
26. All of the following methods are used for the diagnosis of HIV infection in a 2-months old child, except: (AIIMS May 03)
  1. DNA-PCR
  2. Viral culture
  3. HIV ELISA
  4. p24 antigen assay
Ref: Ghai 8/e p229; Nelson 18/e p1492, 859, 1435-1436 and 17/e p1116; Nelson 19/e p1166
27. Which drug is given to prevent HIV transmission from mother to child: (AIIMS Nov 06)
  1. Nevirapine
  2. Lamivudnie
  3. Stavudine
  4. Abacavir
Ref: Ghai 8/e p230, Nelson 17/e, p1120
39
28. Which one of the following hepatitis viruses have significant perinatal transmission: (A/ 03)
  1. Hepatitis E virus
  2. Hepatitis C virus
  3. Hepatitis B virus
  4. Hepatitis A virus
Ref: Ghai 8/e p222; Nelson 18/e p1680; Nelson 19/e p1396, 1397
29. A 45 day-old infant developed icterus and two days later symptoms and signs of acute liver failure appeared. Child was found to be positive for HBsAg. The mother was also HBsAg carrier. The mother’s hepatitis B serological profile is likely to be: (AI 03)
  1. HBsAg positive only
  2. HBsAg and HBeAg positivity
  3. HBsAg and anti-HBe antibody positivity
  4. Mother infected with mutant HBV
Ref: Ghai 8/e p223; 7/e p193-194; Harrison 16/e p1830; Nelson 18/e p1665, 1699, 1700 and 17/e p1327; Nelson 19/e p1398
30. A 2-month-old baby with acute viral hepatitis like illness slips into encephalopathy after 48 hours. The mother is a known hepatitis B carrier. Mother’s hepatitis B virus serological profile is most likely to be: (AI 03)
  1. HBsAg positive only
  2. HBsAg and HBeAg positive
  3. HBsAg and HBe antibody positive
  4. HBV DNA positive
Ref: Ghai 8/e p222; 7/e p193-194; Harrison 16/e p1830; Nelson 18/e p1665-1700 and 17/e p1327;
31. A 5-year-old boy is detected to be HBs Ag positive on two separate occasions during a screening program for hepatitis b. He is otherwise asymptomatic:
  1. Obtain HBeAg and anti-HBe levels [AI 03]
  2. Obtain anti HBs levels
  3. Repeat HBsAg
  4. Repeat another course of Hepatitis B vaccine
Ref: Ghai 8/e p221; Nelson 18/e p1685; 17/e p1328; Nelson 19/e p1399
32. A 30 years old lady delivered a healthy baby at 37 weeks of gestation. She was a known case of chronic hepatitis B infection. She was positive for HBsAG but negative for HBeAG. Which of the following is the most appropriate treatment for the baby: (AIIMS Nov 05)
  1. Both active and passive immunization soon after birth
  2. Passive immunization soon after birth and active immunization at 1 year of age
  3. Only passive immunization soon after birth
  4. Only active immunization soon after birth
Ref: Ghai 8/e p222; Nelson 18/e p1685 and 17/e p1328-1329; Nelson 19/e p1399
33. Regarding polio which is true: (AIIMS Nov 07) (May 08)
  1. Most of the cases are symptomatic
  2. Spastic paralysis is seen
  3. IM injections and increased muscular activity increases the risk of paralytic polio
  4. Pulse polio immunization is indicated in less than 3 years of age
Ref: OP Ghai 8/e p215; 7/e p190; SPM, park 19/e, p 168; Nelson 19/e p1087
34. Kenny Packs were used in the treatment of: (AIIMS 03)
  1. Poliomyelitis
  2. Muscular dystrophy
  3. Polyneuropathies
  4. Nerve injury
Ref: Ghai 8/e p215; 7/e p190
35. Acute flaccid paralysis is reported in a child aged: (AI 02)
  1. 0-3 years
  2. 0-5 years
  3. 0-15 years
  4. 0-25 years
Ref: Ghai 8/e p592; 7/e p190
36. An 8-year-old boy presented with fever and bilateral cervical lymphadenopathy with prior history of sore throat. There was no hepatomegaly. The peripheral blood smear shows > 20% lympho-plasmacyteoid cells. The most likely diagnosis is: (AI 02)
  1. Influenza
  2. Tuberculosis
  3. Infectious mononucleosis
  4. Acute lymphoblastic leukemia
Ref: Ghai 8/e p215; 7/e p187; Harrison 16/e p1045-1048; Nelson 19/e p1111
37. Which of the following does not establish a diagnosis of congenital CMV infection in a neonate? (AIIMS May 06)
  1. Urine culture of CMV
  2. IgG CMV antibodies in blood
  3. Intranuclear inclusion bodies in hepatocytes
  4. CMV viral DNA in blood by polymerase chain reaction
Ref: Ghai 8/e p272; Nelson 18/e p1378 and 17/e p1068; Harrison 16/e p1051; Nelson 19/e p1111
38. Dengue shock syndrome is characterized by the following except: (AIIMS May 05)
  1. Hepatomegaly
  2. Pleural effusion
  3. Thrombocytopenia
  4. Decreased haemoglobin
Ref: Ghai 8/e p200; 7/e p196; Nelson 18/e p1412-1414 and 17/e p1092-1093; Harrison 16/e p1173; Nelson 19/e p1148
39. After 5 days of birth, baby developed poor feeding, convulsions, fever with low protein, low sugar and high chloride (in CSF) is most likely due to: (AIIMS June 00)
  1. Listeria monocytogenes
  2. Mycoplasma pneumoniae
  3. TB
  4. Leptospira
Ref: Ghai 8/e p262; Nelson 18/e p1157-1159 and 17/e p2039; Nelson 19/e p929
40
40. True about tetanus: (PGI June 01)
  1. Tetanus bacilli can spread through blood
  2. Spread along the nerves only
  3. Requires oxygen
  4. IP is variable
  5. Causes deep wound invasion
Ref: Harrison Ghai 8/e p243; 16/e p840–841; Nelson 19/e p992
41. A child with fever and sore throat developed acute cervical lymphadenopathy most likely investigation to be done is: (AIIMS June 00)
  1. Open biopsy of node
  2. Radical neck dissection
  3. Neck X-ray
  4. Complete hemogram
Ref: Ghai 8/e p241; Nelson 18/e p917, 2093-2095, 2106 and 17/e p1062-1066, 1678; Nelson 19/e p992, 993
42. In a 6 months old baby, floppy infant syndrome is seen commonly due to infection with: (AIIMS June 00)
  1. Clostridium welchii
  2. Clostridium tetani
  3. Clostridium Botulinum
  4. Clostridium septicum
Ref: Ghai 8/e p243; Nelson 18/e p1224-1227 and 17/e p947–948; Nelson 19/e p989
43. Integrated management of childhood illness includes except: (Al 09)
  1. ARI
  2. Malaria
  3. Diarrhea
  4. TB
Ref: Ghai 8/e p751-756; 7/e p735; Nelson, 18th/e, p1613; Nelson 19/e p1334
44. Resistant plasmodium falciparum malaria in the pediatric age group should be treated by: (AI 08)
  1. Chloroquine
  2. Tetracycline
  3. Clindamycin
  4. Doxycycline
Ref: Ghai 8/e p260; Nelson’s 18/e p1421; Nelson 19/e p1203
45. A child presents with generalized petechiae. CSF shows gram negative diplococci. Treatment of choice: (PGI Nov 07)
  1. IV ceftriaxone
  2. IV cefotaxime
  3. IV penicillin G
  4. IV ampicillin
  5. IV ofloxacin
Ref: Ghai 8/e p243; 7/e p539; Harrison 16/e, 2476, 17/e, 913
46. All are true about septic shock in children except? (PGI Nov 09)
  1. 1st response is CO. due to vasodilaiaiion
  2. Hypotension is a late sign
  3. Heart rate remains same
  4. Periph vascular resistance to compensate
  5. Endotheliurn release vasodl” factors
Ref: Ghai 8/e p715; Robbins 7/e p139
47. A child presented with fever for 2 days, altered sensorium and purpuric rashes. His blood pressure is 90/60 mmHg. Treatment of choice is: (AIIMS May 02)
  1. IV Quinine
  2. IV Artesunate
  3. IV Penicillin
  4. Chloroquine
Ref: Ghai 8/e p223; 7/e p538-539; Nelson 18/e p1166, 2514-2519 and 17/e p2024; Nelson 19/e p932
48. Vaccine with best efficacy: (AI 09)
  1. TT
  2. DPT
  3. Measles
  4. Typhoid
Ref: Ghai 8/e p215; 7/e p171; Nelson 19/e p881
49. A 4-year-old child presented in OPD with vaccination history of receiving one dose of OPV and DPT at 2 months of age. Correct statements are: (PGI Dec 04)
  1. BCG should not be given
  2. DPT and OPV should be repeated.
  3. Measles should be given
  4. Hepatitis ‘B’ vaccine advised
  5. Hemophilus vaccine not given
Ref: Ghai 8/e p215; Nelson 18/e p1013, 1062 and 17/e p1178
50. Which vaccine is contraindicated in child with history of convulsions? (AIIMS June 08)
  1. DPT
  2. Measles
  3. Typhoid
  4. BCG
Ref: Ghai 8/e p205; 7/e p167-169; IAP guide book on immunization, 3/e p14
51. 18 months old child, who has received one dose of DPT and OPV at 2 months of age. What will be your next immunization plan? (AIIM Nov 07)
  1. Restart immunization schedule, as per age
  2. Measles, BCG, booster dose of DPT and OPV
  3. Measles, booster dose of DPT and OPV
  4. BCG, 2nd dose of DPT and OPV
Ref: Ghai 8/e p205; IAP guide book on immunization, 3/e p41, S.K. Mittal and Kukreja book of immunization, 2/e, p5
52. A premature baby of 34 weeks was delivered. The baby had a bullous lesion on the body and X-ray showed periostitis what will be the next diagnostic procedure: (AI 07)
  1. VDRL of mother and baby
  2. ELISA for HIV
  3. PCR for Herpes
  4. HBsAg for mother
Ref: Ghai 8/e p272; Nelson’s 18/e p1264-1265
53. Congenital syphilis can be best diagnosed by: [AI 01]
  1. IgM FTA- ABS
  2. IgG FTA-ABS
  3. VDRL
  4. TPI
Ref: Ghai 8/e p272; Harrison 16/e p982
41
54. Symptomatic neonatal CNS involvement is most commonly seen in which group of congenital intrauterine infection: (AI 08)
  1. CMV and toxoplasmosis
  2. Rubella and toxoplasmosis
  3. Rubella and HSV
  4. CMV and syphilis
Ref: Ghai 8/e p274; Neurology Valpe 3/e, p63
55. Cysticercosis present as: (PGI Dec 03)
  1. Seizures
  2. Neuropathy
  3. Encephalitis
  4. Muscular hypertrophy
Ref: Ghai 8/e p570
56. Which vaccines are not given in a 8 yrs old unimmunized child: (PGI Dec 06)
  1. Pertusis
  2. Salk vaccine
  3. Measles
  4. BCG
  5. DT
Ref: Ghai 8/e p205; See above explanation
57. The following age group is most severely affected by Rubella infection: (JIPMER 02)
  1. Females aged 25-35 years
  2. Young girls
  3. Adolescent girls
  4. Unborn child
Ref: Ghai 8/e p215; 7/e p1
58. Roseola infantum can be caused by: (Kerala 04)
  1. Herpes virus 6
  2. Parvovirus B 19
  3. Echovirus 19
  4. All of the above
Ref: Ghai 8/e p213; 7/e p186
59. The etiological agent for roseola infantum is: (JIPMER 02)
  1. Parvovirus
  2. Human herpes virus 6
  3. CMV
  4. EBV
Ref: Ghai 8/e p213, 214; 7/e p186
60. Which positive test done not necessarily indicate–clostridium botulinum HIV infection in a newborn? (Karnataka 04)
  1. ELISA for HIV 1gG antibody
  2. p24 antigen
  3. Virus culture
  4. ELISA for HIV IgM antibody
Ref: Ghai 8/e p229; 7/e p203
61. 10 years old child with 10 days continuos fever with soft, enlarged spleen, diagnosis is: (JIPMER 02)
  1. Enteric fever
  2. Malarial
  3. Hodgkins disease
  4. Meningitis
Ref: Ghai 8/e p240; 7/e p222
62. Patient was given chloroquine and doxycycline for 7 days. Patient’s fever decreases in 4 days, but, peripheral smear showed occasional gametocytes of plasmodium falciparum. This type of drug resistance is: (CMC 05)
  1. R type
  2. R2type
  3. R3 type
  4. R4 type
Ref: Ghai 8/e p260; 7/e p227
63. Which one of the following antibacterial antibiotics are not recommend for lactating mothers: (COMED 06)
  1. Cephaslosporins
  2. Anti tubercular drugs
  3. Quinolones
  4. Aminoglycoside
Ref: KD Tripathi 5/e p534; Ghai 7/e p724, 6/e p681
64. HIV associated nephropathy is severe form of: (JIPMER 04)
  1. Focal segmental glomerulosclerosis
  2. Membranous nephropathy
  3. Membrano-proliferative glomerulonephritis
  4. Focal segmental glomerulonephritis
Ref: Ghai 8/e p229; Robbin 7/e p984
65. Which one of the following is a conjugated vaccine: (UPSC 04)
  1. Hepatitis B
  2. Rubella
  3. Hemophilius influenza
  4. Pertussis
Ref: Ghai 8/e p190
66. 12.6 months old baby was brought with c/o difficulty in feeding. The child was found to be hypnotic with a week gag. The child is on breast milk and mother also gives honey to the child during periods of excessive crying. The causative agent is: (J & K 05)
  1. Gram-positive aerobic occurs
  2. Gram positive anaerobic spre-suffering bacillus
  3. Toxin produced by gram-positive anaerobic bacillus
  4. Echovirus
67. A 32 weeks premature infant, 900 gm weight on the third days. The serum bilirubin is 13 mg%. The treatment of choice is: (UP 07)
  1. Exchange transfusion
  2. Phototherapy
  3. Wait and watch therapy
  4. Pharmacologic therapy
Ref: OP Ghai 8/e p175; 7/e p191-195
68. Differential diagnosis in case of fever with vesicular rash for two days are all except: (UP 07)
  1. Candida albicans
  2. Infectious mononucleosis
  3. Klebsiella pneumonia
  4. Influenza
Ref: Ghai 8/e p216; Harrison’s 16/e, p110-114
69. For the prevention of parent to child transmission of HIV, the NACO’s recommendation is to give: (UPSC 07)
  1. Niverapine 200 mg in active labor to mother
  2. Niverapine 200 mg, four hours after rupture of membranes, to mother
  3. Niverapine 200 mg in active labor to mother and syrup niverapine 2 mglkg body weight to newborn within 72 hours of delivery
  4. Syrup niverapine 2 mg/kg body weight to newborn within 72 hours of birth
Ref: OP Ghai 8/e p229; 7/e p207
42
70. 2-years-old child with head circumference of 55 cm, is likely to have had intrauterine infection due to: (Corned 07)
  1. Rubella
  2. Toxoplasmosis
  3. Herpes
  4. Hepatitis B
Ref: Ghai 8/e p272; Nelson 17/e p1989
71. Infant with cystic fibrosis (CF) are likely to develop: (MAHE 07)
  1. Meconium ileus
  2. Loose motions
  3. Vomiting
  4. Constipation
Ref: OP Ghai 8/e p393; 7/e p369
72. Pneumatoceles often develop in children after pneumonia due to the following organism: (Corned 07)
  1. Klebsiella
  2. Streptococcus
  3. Staphylococcus aureus
  4. Hemophilus influenzae
Ref: OP Ghai 8/e p374; 7/e p352
73. An infant develops cough and fever. The X-ray examination is suggestive of bronchopneumonia. All of the following viruses can be the causative agent except: (AI 04)
  1. Parainfluenza viruses
  2. Influenza virus A
  3. Respiratory syncytial virus
  4. Mumps virus
Ref: Ghai 8/e p376; 7/e p187-188; Nelson 18/e p797-798, 394, 1795-1799 and 17/e p1433; Nelson 19/e p1475
74. 2-month-old girl present with verrucous plague on the trunk: What is your most probable diagnosis? (AIIMS Nov 08)
  1. Incontinentia pigmentosa
  2. Darier disease
  3. Cogenital nevus
  4. Ichthyosis
Ref: Ghai 8/e p675; Nelson 18/e p2681; Nelson 19/e p2052, 2053
75. Which is true about BCG?
  1. Distilled water is used as diluents
  2. Site for injection is cleaned with spirit
  3. Mantoux test positive in 6 weeks
  4. WHO recommends Danish 1331 for vaccine production
Ref: 171 Park 21/e, p176
76. Live attenuated bacterial vaccine is: (DNB 2009)
  1. Tetanus
  2. Hepatitis B
  3. Sabin
  4. BCG
Ref: OP Ghai 8/e p310
77. Congenital Rubella causes all the following features except: (DNB 2011)
  1. Cataract
  2. Deafness
  3. Cardiac defects
  4. Hydrocephalus
Ref: Ghai 8/e p210; Esssentials of Pediatrics, Nelson 4/e p242, 243
78. Vertical transmission of Toxoplasmosis is more severe in: (DNB 2011)
  1. During labour
  2. Ist trimester
  3. 2 nd trimester
  4. Third trimester
Ref: Ghai 8/e p210; Esssentials of Pediatrics, Nelson 4/e p242, 465, 466
79. Not a cause of pneumonia is: (DNB 2011)
  1. Measles
  2. Mumps
  3. RSV
  4. Influenza
Ref: Ghai 8/e p376; Nelson Ped, 18/e p2366
80. What is the minium interval required between the administration of two live vaccines? (MP PG 2010)
  1. 8 Weeks
  2. 6 Weeks
  3. 4 Week
  4. 2 Weels
Ref: Ghai 8/e p203; Nelson’s 18/e p1078
81. Oral typhoid vaccine is not recommended for children in the age group of: (MP PG 2009)
  1. < 2 years
  2. < 4 years
  3. > 6 years of age
  4. < 8 years
Ref: Ghai 8/e p197; Nelson’s 18/e p174
82. The most effective and preferred scabicidal drug for treatment of scabies in children is: (MP PG 2008)
  1. Permethrin
  2. Cromatilion
  3. Hexachlorobenzene
  4. Ivermectin
Ref: KDT’s Pharma 6/e p865; Nelson’s 18/e p2757
83. Black water fever is a complication associated with: (MP PG 2008)
  1. Filaria
  2. Leptospirosis
  3. Kalazar
  4. Malaria
Ref: Ghai 8/e p260; Nelson’s 18/e p1481; 17/e p1142
84. Typhoid Vi polysaccharide vaccine is usually administered in children above the age of: (Karnataka PG 2011)
  1. 6 months
  2. 1 year
  3. 2 years
  4. 1 year 6 months
Ref: OP Ghai 8/e p190-203; 7/e p174
85. Not immunized previously in a 8 years male child which of the following vaccine is not given to that children: (UP PG 09)
  1. Inactive polio vaccine
  2. Hepatitis B vaccine
  3. Acellular pertusis vaccine
  4. Measles vaccine
Ref: Ghai 8/e p203
86. DPT is contraindicated in: (UP PG 09)
  1. Family history of convulsion
  2. Acute respiratory tract infection
  3. Progressive neurological illness
  4. Mild diarrhea
Ref: Ghai 8/e p192; 7/e p168-69
43
87. Treatment of c. difficle infection: (MHPGM-CET 2010)
  1. Metronidazole
  2. Bacitracin
  3. Nitazoxanilide
  4. All of the above
Ref: Ghai 8/e p299; Harrison’s 17/e 820; KDT 6/e p731; Table 123-2
88. Incubation period of mumps? (MHPGM-CET 2007, 2010)
  1. 5-7 days
  2. 8-15 days
  3. 14-24 days
  4. 24-30 days
Ref: Ghai 8/e p217; Nelson paediatrics 18/e p1341
89. Single-dose mebendazole is effective in treatment of (repeat dose 14 days later): (MHPGM-CET 2008, 2010)
  1. Enterobius vermicularis
  2. Ankylostoma duodenale
  3. Ascaris lumbricoides
  4. All of the above
Ref: Nelson paediatrics 18/e p1450; KDT Pharmacology 6/e p809
90. Clinical Manifestations of Congenital Rubella Syndrome in children following Maternal Rubella include all except: (AP 2010)
  1. Nerve deafness
  2. Paychomotor retardation
  3. Patent Ductus Arteriosus
  4. Aortic stenosis
Ref: Ghai 8/e p272
91. A 6 month old infant presents with an early feature of vomiting, more severe diarrhea, mild to moderate fever and dehydration. The possible infection is due to: (AP 2010)
  1. Norwalk virus
  2. Rota virus
  3. Shigella
  4. e. coli
Ref: Ghai 8/e p299
92. Baby developed signs and symptoms of congential syphilis after two weeks. This is supported by the following law: (AP 2011)
  1. Kassowitz’s law
  2. Profeta law
  3. Colles law
  4. Didays law
Ref: Ghai 8/e p272
93. Reactive arthistis, erythema nodosum, haemolytic anaemia and dysentery occur more often in enteric infections with: (AP 2012)
  1. Entamoeba histolytica
  2. Yersinia enterocolitica
  3. Campylobacter jejuni
  4. Cryptosporidum parvum
Ref: Ghai 8/e p296
94. Which one of the following investigative features is NOT indicative of severe malaria: (AP 2012)
  1. Hypoglycemia
  2. Lactic acidosis
  3. Haemoglobinuria
  4. Macrocytic RBC’s
Ref: Ghai 8/e p260
95. Which one of the following is NOT a features of congenital rubella syndrome (AP 2012)
  1. Deafness
  2. Microphthalmia
  3. Hepatitis
  4. Cataracts
Ref: Ghai 8/e p272
96. All are true regarding typhoid vaccines except: (DP PGMEE 2009)
  1. Heat killed whole cell vaccine causes significant local pain and systemic malaise
  2. Vi polysaccharide is non-immunogenic in children less than two years
  3. Ty21 a vaccine can be given intramuscularly and subcutaneously
  4. Vi polysaccharide vaccine is to be repeated every 3 years
Ref: Ghai 8/e p197
97. The 23 valent pneumococcal vaccine is recommended in all except: (DP PGMEE 2009)
  1. Cerebrospinal fluid leak
  2. Chronic cardiac disease
  3. Children less than 2 years
  4. Nephritic syndrome
Ref: Ghai 8/e p199
98. Antibodies to one of the following infection is not transmitted to child: (DP PGMEE 2010)
  1. Measles
  2. Pertusis
  3. Diphtheria
  4. Polio
Ref: Ghai 8/e p243
99. All of the following are features of mumps except: (Feb DP PGMEE 2009)
  1. Caused by paramyxovirus
  2. Aseptic meningitis is a complication in children
  3. Orchitis is a complication in adults
  4. Incubation period is less than 2 weeks
Ref: Ghai 8/e p217
100. A child 18 month old with 2 month history of bloody Dysentery the probable cause is: (Raj PGI 2008)
  1. Adenovirus
  2. Rotavirus
  3. Campylobacter
  4. Norwalk
Ref: Ghai 8/e p296
101. Which of the following is best used in the diagnosis of congenital syphilis: (Raj PGI 2009)
  1. FTA-ABS
  2. TPHA
  3. IgM-FTAABS
  4. TPI
Ref: Ghai 8/e p272
102. Which of the following is contraindicated in a 4 year old child with immunosuppression? (Kerala PG 09)
  1. Acellular pertusis
  2. Measles
  3. Tetanus
  4. Varicella
Ref: Ghai 8/e p189; Nelson 18/e p1371
44
103. All the following regarding conjugate pneumococcal vaccine are true except: (Kerala PG 10)
  1. Provoke antibody response in 90% of infants and reduces the incidence of nasopharyngeal carriage of vaccine serotypes
  2. Enhanced responses after booster doses given at 12-15 months of age
  3. It helps to reduce the incidence of otitis media in infants
  4. Cannot be given in children < 2 years of age
Ref: Ghai 8/e p199; Nelson 18/e p1134
104. Which among the following is difference of conjugate pneu-mococcal vaccine, when compared to polysaccharide pneumococcal vaccine: (Kerala PG 10)
  1. Can be given in less than 2 years
  2. Contains more serotypes
  3. Booster is not required
  4. Incidence of invasive infections are not reduced following administration
Ref: Ghai 8/e p199; Nelson 18/e p1134
105. Which among the following is true regarding exanthem: (Kerala PG 10)
  1. Roseola infantum is fifth disease
  2. Koplick spot cannot occur before rash
  3. Forreschmier spot is petechial spot in soft palate in rubella
  4. Symptoms of vaccine (Breakthrough) chickenpox is similar to unvaccinated type
Ref: Ghai 8/ep213; Nelson 18/e p1339, 1317, 1357
 
 
Ans.
1. c. Aortic stenosis
2. d. Increased congenital...
3. a, b and c
4. c. Rubella and HSV
5. a, b and c
6. d and e
7. b. Secondary...
8. c and d
9. d. Measles
10. a. Meningoencephalitis...
11. c. Menigoencephalitis...
12. c. Rubella
13. c. Roseola infantum
14. a. When the lesions...
15. b. Caused by HHV...
16. a and b
17. b, c and d
18. b. Staphylococcal...
19. a. Mycobacterium...
20. c. BCG +INH...
21. All of these
22. a, b and d
23. c. Pulmonary TB
24. b. <5
25. b. Vaginal cleansing...
26. c. HIV ELISA
27. a. Nevirapine
28. c. Hepatitis B virus
29. b. HBsAg and HBeAg...
30. b. HBsAg and HBeAg...
31. b. Obtain anti HBs...
32. a. Both active...
33. c. IM injections...
34. a. Poliomyelitis
35. c. 0-15 years
36. c. Infectious...
37. b. IgG CMV antibodies...
38. d. Decreased haemoglobin
39. a. Listeria monocytogenes
40. b and d
41. d. Complete hemogram
42. c. Clostridium Botulinum
43. d. TB
44. c. Clindamycin
45. c. IV penicillin G
46. c and d
47. c. IV Penicillin
48. c. Measles
49. b, c and d
50. a. DPT
51. d. BCG, 2nd dose
52. a. VDRL of mother...
53. a. IgM FTA- ABS
54. c. Rubella and HSV
55. a and c
56. a, b and d
57. d. Unborn child
58. a. Herpes virus 6
59. b. Human herpes...
60. a. ELISA for HIV...
61. a. Enteric fever
62. b. R2type
63. c. Quinolones
64. a. Focal segmental...
65. c. Hemophilius influenza
66. c. Toxin produced...
67. a. Exchange transfusion
68. b. Infectious mononucleosis
69. c. Niverapine 200 mg...
70. b. Toxoplasmosis
71. a. Meconium ileus
72. a and c
73. d. Mumps virus
74. a. Incontinentia...
75. d. WHO recommends...
76. d. BCG
77. d. Hydrocephalus
78. b. Ist trimester
79. b. Mumps
80. c. 4 Week
81. c. > 6 years of age
82. a. Permethrin
83. d. Malaria
84. c. 2 years
85. c. Acellular pertusis...
86. c. Progressive...
87. d. All of the above
88. c. 14-24 days
89. a. Enterobius vermicularis
90. d. Aortic stenosis
91. b. Rota virus
92. b. Profeta law
93. c. Campylobacter jejuni
94. d. Macrocytic RBC’s
95. c. Hepatitis
96. c. Ty21 a vaccine...
97. c. Children less than...
98. b. Pertusis
99. d. Incubation period...
100. c. Campylobacter
101. c. IgM-FTAABS
102. b and d
103. d. Cannot be given...
104. a. Can be given
105. c. Forreschmier...
 
457. HEMATOLOGICAL DISORDERS
 
46HEMATOLOGICAL DISORDERS (QUESTIONS)
1. Which organ is the primary site of hematopoiesis in the fetus before midpregnancy: (AIIMS May 06)
  1. Bone
  2. Liver
  3. Spleen
  4. Lung
Ref: Ghai 8/e p623; Robbin’s 7/e p620, Nelson 18/e p1997; Nelson 19/e p1650, 1651
2. When does switchover from fetal to adult hemoglobing synthesis begin: (AIIMS Nov 04, Nov 05, May 05)
  1. 14 weeks gestation
  2. 30 weeks gestation
  3. 36 weeks gestation
  4. 7-10 days postnatal
Ref: Ghai 8/e p419; Nelson 18/e p2001; Nelson 19/e p1662
3. A 5-years-old girl came with history of progressively increasing pallor since birth and hepatosplenomegaly. Which of the following is the most relevant test for achieving diagnosis: (AI 04)
  1. Hb electrophoresis
  2. Peripheral smear examination
  3. Osmotic fragility test
  4. Bone marrow examination
Ref: Ghai 8/e p456; Nelson 18/e p2035, 2036; Nelson 19/e p1674, 1675
4. Bart’s hydrops fetalis is lethal because: (AI 05)
  1. Hb Bart’s cannot bind oxygen
  2. The excess a-globin form insoluble precipitates
  3. Hb Bart’s cannot release oxygen to fetal tissues
  4. Microcytic red cells become trapped in the placenta
Ref: Ghai 8/e p422; 7/e p307-308; Robbin’s 7/e p636; Nelson 18/e p2016, 2033-2037 and 17/e p1633; Nelson 19/e p1675
5. Diagnosis of beta thalassemia is established by: (AI 05)
  1. NESTROFT Test
  2. Hb A1, C estimation
  3. Hb electrophoresis
  4. Target cells in peripheral smear
Ref: Ghai 8/e p425; 7/e p308-309; Nelson 18/e p2033-2037 and 17/e p1633; Robbin’s 7/e p635; Nelson 19/e p1675
6. A child aged 2 years presents with nonspecific symptoms suggestive of anemia. On peripheral blood smear target cells are seen. He has hypochromic microcytic picture and Hb of 6 gm%. He also has a positive family history. Next investigation of choice is: (AI 01)
  1. Hb electrophoresis
  2. Coomb’s test
  3. Liver function tests
  4. Osmatic fragility test
Ref: Ghai 8/e p423; 7/e p308-309; Nelson 18/e p2034-2036 and 17/e p1633; Robbin’s 7/e p635; Nelson 19/e p1675
7. All of the following are true of thalassemia major, except: (AIIMS May 06)
  1. Splenomegaly
  2. Target cells on peripheral smear
  3. Microcytic hypochromic anemia
  4. Increased osmotic fragility
Ref: Ghai 8/e p423; 7/e p308-309; Nelson 19/e p1674
8. Which of the following aemoglobin (Hb. Estimation will be diagnostically helpful in a case of beta thalassemia trait? (AIIMS May 06)
  1. Hb-F
  2. Hb1-C
  3. Hb-A2
  4. Hb-H
Ref: Ghai 8/e p425; 7/e p307-308; Nelson 18/e p2060-2061; 17/e p1633; Robbin’s 7/e p635; Nelson 19/e p1676
9. A child died soon after birth. On examination there was hepatosplenomegaly and edema all over body. Most probable diagnosis in: (AIIMS May 02)
  1. β-thalassemia
  2. α-thalassemia
  3. Hereditary spherocytosis
  4. ABO incompatibility/sickle cell anemia
Ref: Ghai 8/e p423, 424; 7/e p307-309; Nelson 18/e p2032-2033 and 17/e p1633; Nelson 19/e p1676
10. Thalassemia occurs due to which mutations: (PGI Dec 2000)
  1. Missense
  2. Splicing
  3. Transition
  4. Frame-shift
  5. Truncation
Ref: Ghai 8/e p425; Harper 27/e, p415, Robbin’s 7/e, p632, Nelson 18/e p2035; Nelson 19/e p1674
11. Thalasemia trait; true about: (PGI June 04)
  1. HbF
  2. HbA2
  3. Microcytosis
  4. Severe anemia
Ref: Ghai 8/e p424, 425; Nelson 18/e p2035, 2036, Robbin’s 7/e p634-635, Harrison 17/e, p640, 641; Nelson 19/e p1676
12. The primary defet which leads to sickle cell anemia is: (AI 03)
  1. An abnormality in porphyrin part of hemoglobin
  2. Replacement of glutamate by valine in chain of HbA
  3. A nonsence mutation in the (3-chain of HbA)
  4. Substitution of valine by glutamate in the achain of HbA
Ref: Ghai 8/e p452; Nelson 18/e p2026, Robbin’s 7/e p628; Nelson 19/e p1663
13. Mutation leading to sickle cell anemia: (PGI June 01)
  1. Crossover mutation
  2. Frame shift
  3. Deletion
  4. 47Nondysfunction
  5. Pointmutation
Ref: Ghai 8/e p450, 452; Already explained in Previous Answer
14. Regarding G6PD deficiency true are: (PGI June 06)
  1. Autosomal dominant
  2. Bite cell (+)
  3. Protects against kala azar
  4. Enzyme level directly proportional to age of RBC
  5. Sex preponderance
Ref: Ghai 8/e p437; Nelson 18/e p2041; Nelson 19/e p1673
15. True about iron deficiency anemia: (PGI June 05)
  1. Microcytic hypochromic anemia
  2. Decreased TIBC
  3. Increased ferritin
  4. Bone marrow iron decreased earlier than serum iron
Ref: Robbin’s 7/e p645; Ghai 8/e p406; Nelson 19/e p1656
16. The earliest indicator of response after starting iron in a 6 years old girl with iron deficiency is: (AI 06)
  1. Increased reticulocyte count
  2. Increased aemoglobin
  3. Increased ferritin
  4. Increased serum iron
Ref: Ghai 8/e p405, 406; 7/e p300-301; Nelson 18/e p2016-2017 and 17/e p1615-1616; Nelson 19/e p1656
17. A nine months old boy of Sindhi parents presented to you with complaints of progressive lethargy, irritability and pallor since 6 months of age. Examination revealed severe pallor. Investigation showed Hb-3.8 mg%; MCV-58 fl; MCH-19.4 pg/cell. Blood film shows osmotic fragility is normal (target cells and normoblasts). X-ray skull shows expansion of erythmoid marrow. Which of the following is the most likely diagnosis? (AI 04)
  1. Iron deficiency anemia
  2. Acute lymphoblastic anemia
  3. Hemoglobin D disease
  4. Hereditary spherocytosis
Ref: Ghai 8/e p405; Nelson 18/e p1528, 2014-2017 and 17/e p1629; Nelson 19/e p1656
18. True about acute ITP: (PGI June 05)
  1. More common in female
  2. Specific anti platelet antibodies detected
  3. Viral infection predisposes as seen after vaccination
  4. 80% cases transforms to chronic
  5. More common in children
Ref: Ghai 8/e p642; Nelson 18/e p2082, 2084, 2087 and 17/e p1670-1671; Nelson 19/e p1715
19. A child underwent a tonsillectomy at 6 years of age with no complications. He underwent a preoperative screening for bleeding at the age of 12 years before an elective laparotomy, and was found to have a prolonged partial thromboplastin time but normal prothrombin time. There was no family history of bleeding. The patient is likely to have: (AIIMS Nov 04)
  1. Acquired VitK deficiency
  2. Acquired liver disease
  3. FactorXIIdeficiency
  4. Mild aemophilia A
Ref: Read below; Nelson 19/e p1698; Internet
20. A 10-years-old girl presents with swelling of one knee joint. All of the following conditions can be considered in the differential diagnosis except: (AI 03; AIIMS Nov 01)
  1. Tuberculosis
  2. Juvenile rheumatoid arthritis
  3. Hemophilia
  4. Villonodular synovitis
Ref: Ghai 8/e p462; 7/e p320-321; Robbin’s 7/e p1310; Harrison 16/e p2063; Nelson 19/e p1699
21. Characteristic lab findings of haemophilia A are: (PGI June 06)
  1. Increase PT
  2. Increase aPTT
  3. X-linked recessive
  4. Presence of 30% of factor level express the disease
  5. Increased bleeding time
Ref: Ghai 8/e p465; Nelson 18/e p2066-2069; Nelson 19/e p1700
22. Persistent bleeding from umbilical stump, the most probable diagnosis is: (AIIMS Nov 06)
  1. Factor XIII deficiency
  2. Glanzman’s thrombasthenia
  3. VWD
  4. Bernard soulier syndrome
Ref: Nelson 18/e p2060-2061 and 17/e p1661; Wintrob 11/e, p1604; Nelson 19/e p1703; Internet
23. A 15 years old female presented to the emergency department with history of recurrent epistaxis, hematuria and hematochezia. There was a history of profuse bleeding from the umbilicus stump at birth. Previous investigations revealed normal prothrombin time, activated partial thromboplastin time, thrombin time and fibrinogen levels. Her platelet counts as well as platelet function tests were normal but urea clot lysis test was positive. Which one of the following clotting factor is most likely to be deficient? (AIIMS May 06)
  1. Factor X
  2. Factor XI
  3. Factor XII
  4. Factor XIII
Ref: Ghai 7/e p317-318, 6/e p322; Nelson 18/e p2060-2061 and 17/e p1660-1661; Harrison 16/e p682-683; Nelson 19/e p1704; Internet
24. A newborn baby presented with profuse bleeding from umbilical stump after birth. Probable diagnosis is: (AIIMS May 07, Nov 06)
  1. Factor XIII deficiency
  2. VWF deficiency
  3. Factor XII deficiency
  4. Glanzmann thrombosthenia
Ref: J.b. Henry clinical diagnosis and management by laboratory methods 20/e, p650; Nelson 19/e p1704; Internet
25. Late onset hemorrhagic disease of newborn is characterized by all of the following features except: (Al 06)
  1. Usually occurs in cow-milk fed babies
  2. Onset occurs at 4-12 week of age
  3. 48Intracranial aemorrhage can occur
  4. Intramuscular vitamin K prophylaxis at birth has a protective role
Ref: Ghai 8/e p465; Nelson 18/e p773; Nelson 19/e p620, 621
26. The coagulation profile in a 13-years old girl with menorrhagia having von Willebrands disease is: (AI 05)
  1. Isolated prolonged PTT with a normal PT
  2. Isolated prolonged PT with a normal PTT
  3. Prolongation of both PT and PTT
  4. Prolongation of thrombin time
Ref: Robbin’s 7/e p654-655; Ghai 8/e p469; Nelson 18/e p2060, 2061 and 17/e p1662; Nelson 19/e p1704-1706
27. Which of the following is generally not seen in idiopathic thrombocytopenic purpura (ITP)? (AIIMS Nov 04)
  1. More common in females
  2. Petechiae, ecchymosis and bleeding
  3. Palpable splenomegaly
  4. Increased megakaryocytes in bone marrow
Ref: Robbin’s 7/e p654–655; Ghai 8/e p462; Nelson 18/e p2060, 2061 and 17/e p1662; Nelson 19/e p1715
28. Iron requirement is determined from the equation: (AMU 05)
  1. 3 x wt. (kg) x Hb deficit (gm/dl)
  2. 3.3 x wt. (kg) x Hb deficit (gm/dl)
  3. 4 x wt. (kg) x Hb deficit (gm/dl)
  4. 4.3 x wt. (kg) x Hb deficit (gm/dl)
Ref: OP Ghai 8/e p406; 7/e p301
29. Fanconi’s anemia is a: (Jipmer 03)
  1. Constitutional anemia
  2. Hemolytic anemia
  3. Iron deficiency anemia
  4. Auto - immune anemia
Ref: Nelson 17/e p1642; Internet
30. Quinine induced thrombocytopenia is: (Jipmer 04)
  1. Antibody mediate
  2. Dose related toxicity
  3. Idiosyncratic reaction
  4. Inhibits production of platelets
Ref: Harrison 16/e p674–675; Internet
31. Iron supplementation in a healthy term breast fed baby should be started at the age of: (COMED 06)
  1. 2 weeks
  2. 4 weeks
  3. 8 weeks
  4. 6 weeks
Ref: Ghai 8/e p352; Park 18/e p405
32. A four-years-old child presents with mild fever, malaise, prupura, arithritis, abdominal pain and microscopic hematuria. What would be the most likely diagnosis: (UPSC 07)
  1. Thrombasthenia
  2. Idiopathic thrombocytopenic
  3. Systemic lupus erythematosus
  4. Henoch-Schonlein purpura
Ref: OP Ghai 8/e p623
33. Which one of the following is not a feature of combined immunodeficiency in children: (UPSC 07)
  1. Decreased total serum immunoglobulin level
  2. Neutropenia and eosinophilia
  3. Defective T-cell function
  4. Recurrent pulmonary infection
Ref: O.P Ghai 7/e p57, 58, 6/e p187; Harrison 16/e p1942; Nelson 17th/e p697
34. Thrombocytopenia in a newborn baby can be caused by all except: (AIIMS Nov 99)
  1. ABO incompatibility
  2. Isoimmune thrombocytopenia
  3. Autoimmune thrombocytopenia
  4. SLE
Ref: Nelson’s Essentials of Pediatrics 5/e, p716
35. A child is brought with drowsiness, decreased deep tendon reflexes and seizures. On examination the child has a line on gums. There is history of constipation. Which will be most appropriate drug that should be used in this child: (AI 07)
  1. EDTA
  2. DMSA
  3. BAL
  4. Penicillamine
Ref: Ghai 8/e p696; KDT 6/e p645, Nelson 18/e p2916; Nelson 19/e p2452
36. Treatment of choice for thalassemia major is: (DNB 2007)
  1. Blood transfusion and iron therapy
  2. Folic acid and desferrioxamine
  3. Blood transfusion and desferrioxamine
  4. Iron, blood transfusion and desferrioxamine
Ref: Ghai 8/e p289; Nelson, 18/e p2032
37. A newborn baby presented with profuse bleeding from umbilical stump after birth. Probable diagnosis is: (DNB 2009)
  1. Factor XIII deficiency
  2. VWF deficiency
  3. Factor XII deficiency
  4. Glanzmann thrombosthenia
Ref: 8/e p613; J.b. Henty, Clinical diagnosis and management by laboratory Methods, 20/e p650
38. Non immune Hydrops is seen in: (DNB 2010)
  1. Hb Barts Thalessemia
  2. Rh Incompatibility
  3. Syphilis
  4. EBV infection
Ref: Ghai 8/e p680; Nelson 18/e p2375, 2379
39. In children with idiopathic thropmbocytopenic purpura (ITP), the thrombocytopenia is considered as severe and unsafe at a level below: (MP PGI 2008)
  1. 10,000/m2
  2. 30,000/mm3
  3. 20,000/mm3
  4. 40,000/mm
Ref: Ghai 8/e p351; Nelson’s 18/e p2082-2084; IAP’s emergencies 2/e p273
49
40. Splenectomy eliminates most of the hemolysis associated with: (MP PGI 2008)
  1. Sickle cell anemia
  2. Thallassemia
  3. Hereditary spherocytosis
  4. Glucose–6 phosphate dehydrogenase (G-6-PD) deficiency
Ref: Ghai 8/e p347; Bailey and Love’s 25/e p1106
41. Optimal oral dose of elemental iron to treat iron deficiency anemia is: (MP PGI 2008)
  1. 1-2 mg/kg/day
  2. 3-6 mg/kg/day
  3. 7-10 mg/kg/day
  4. 11-14 mg/kg/day
Ref: Ghai 8/e p334; Nelson’s 18/e p2016 and KDT’s 6/e p586
42. Drug which may lead to hemolysis in a child with G6PD deficiency is: (Karnataka PGI 2011)
  1. Penicillin
  2. Primaquine
  3. Ceftriaxone
  4. Erythromycin
Ref: Ghai 8/e p339; Harrison’s 17/e p657
43. Vital pneumonia is caused by all except: (UP PGI 09)
  1. Mumps
  2. Measles
  3. Herpes
  4. Cytomegala virus
Ref: Ghai 8/e p577; 7/e p352, 355; (UP-03)
44. Uveo-parotitis is seen in: (UP PGI 09)
  1. Mumps
  2. Measles
  3. Sarcoidosis
  4. Cystic fibrosis
Ref: Ghai 8/e p630; Basic pathology Robbins 8/e p738
45. Constitutional pancytopenia can be seen in following except: (MHPGM-CET 2010)
  1. Fanconi’s anemia
  2. Diamond-Blackfan syndrome
  3. Dyskeratosis congenita
  4. Schwachman Diamond syndrome
Ref: Ghai 8/e p347; Nelson paediatrics 18/e p2047; Table 468-1
46. Fetal Hb is: (AP 2011)
  1. Two Alpha & two beta
  2. Two Alpha & two gamma
  3. Two Alpha & two delta
  4. Two Alpha & two epsilon
Ref: Ghai 8/e p330
47. A 3-years-old boy with a week history of fever has diffuse faint rashes, erythema of palms and soles, enlarged unilateral cervical node and strawberry tongue. The diagnosis is: (AP 2012)
  1. Scarlet fever
  2. Erythema infectiosum
  3. Infectious mononucleosis
  4. Kawasaki’s disease
Ref: Ghai 8/e p631
48. Basophilic stippling of red cells in a child with hypochromic, microcytic anaemia is indicative of: (AP 2012)
  1. Beta Thalassemia minor and major
  2. Copper deficiency induced anaemia
  3. Lead poisoning
  4. X-linked receive sideroblastic anaemia
Ref: Ghai 8/e p341
49. The coagulation profile in a 13-year old girl with menorrhagia having von willebrand’s disease is: (DP PGMEE 2010)
  1. Isolated prolonged PTT with a normal PT
  2. Isolated prolonged PT with a normal PTT
  3. Prolongation of both PT and PTT
  4. Prolongation of thrombin time
Ref: Ghai 8/e p349; Nelson 18/e p2073, Harrison 16/e p676
50. An asymptomatic growth retarded infant is born by caesarean section for maternal hypertension. He is noted to have low platelet count. What is the most likely diagnosis? (DP PGMEE 2010)
  1. Placental insufficiency
  2. Gram-negative septicemia
  3. Auto-immune thrombocytopenia
  4. Glanzman’s thromboesthenia
Ref: Ghai 8/e p352; Still searching
51. Aplastic anemia is associated with: (Raj PG 2008)
  1. CD 33
  2. CD 34
  3. CD 35
  4. CD 36
Ref: Ghai 8/e p345
52. Regarding Kawasaki disease all are true except: (Kerala PG 09)
  1. Cervical lymph node enlargement is the least common among diagnostic criteria
  2. Conjunctival ingestion is last to disappear among the diagnostic criteria
  3. Coronary artery involvement is less common in infants
  4. Atypical Kawasaki is more prone for cardiac involvement
Ref: Ghai 8/e p631; Nelson 18/e p1039
53. True statement about sickle cell anaemia: (Kerala PG 09)
  1. Hydroxyurea treatment do not decrease painful crisis
  2. Ultimate height is achieved
  3. Hemorrhagic stroke in infants seen
  4. Most common surgery is cholecystectomy
Ref: Ghai 8/e p344; Nelson 18/e p2030
54. Which among the following diagnostic criterion is most characteristic of Kawasakis disease: (Kerala PG 10)
  1. Periungual desquamation
  2. Generalised lymphadenopathy
  3. Splenomegaly
  4. Exudative conjunctivitis
Ref: Ghai 8/e p631; Nelson 18/e p1039
55. Typically Peribronchovascular and Subpleural nodules on HRCT Thorax are seen in: (J & K PG 2010)
  1. Emphysema
  2. Endobronchial spread of tuberculosis
  3. Sarcidosis
  4. Interstital Pulmonary Fibrosis
Ref: Ghai 8/e p373
50
56. The commonest blood transfusion dependant anemia in children in India is: (J & K PG 2010)
  1. Thalassemia major
  2. Thalassemia minor
  3. Iron deficiency anemia
  4. Megalo blastic anemia
Ref: OP Ghai 8/e p341
57. Pneumatoceles are characteristic X ray findings in pneumonia caused by the following organism: (J & K PG 2010)
  1. Streptococcus pneumonia
  2. Staphylococcus and Klebsiella
  3. RSV
  4. Pneumocystitis carnii
Ref: Ghai 8/e p377
58. The most common leukocytoclastic vasculitis affecting children is: (AI 05)
  1. Takayasu disease
  2. Mucocutaneous lymph node syndrome (Kawasaki disease)
  3. Henoch Schonelin purpura
  4. Polyarteritis nodosa
Ref: Ghai 8/e p632; 7/e p607; Harrison 16/e p2010; Robbin’s illustrated 7/e p539-541; Nelson 19/e p1789
 
 
Ans.
1. b. Liver
2. c. 36 weeks gestation
3. a. Hb electrophoresis
4. c. Hb Bart’s cannot...
5. c. Hb electrophoresis
6. a. Hb electrophoresis
7. d. Increased osmotic...
8. c. Hb-A2
9. b. α-thalassemia
10. b and d
11. a, b and c
12. b. Replacement...
13. e. Pointmutation
14. b and e
15. a and d
16. a. Increased...
17. a. Iron deficiency...
18. a, b, c and e
19. c. Factor XII deficiency
20. c. Hemophilia
21. b and c
22. a. Factor XIII...
23. d. Factor XIII
24. a. Factor XIII deficiency
25. a. Usually occurs...
26. a. Isolated prolonged...
27. c. Palpable splenomegaly
28. c. 4 x wt. (kg)...
29. a. Constitutional anemia
30. a. Antibody mediate
31. d. 6 weeks
32. d. Henoch-Schonlein...
33. b. Neutropenia...
34. a. ABO incompatibility
35. a. EDTA
36. d. Iron, blood...
37. a. Factor XIII deficiency
38. a. Hb Barts...
39. a. 10,000/m2
40. c. Hereditary...
41. b. 3-6 mg/kg/day
42. b. Primaquine
43. a. Mumps
44. c. Sarcoidosis
45. b. Diamond-Blackfan...
46. b. Two Alpha...
47. d. Kawasaki’s disease
48. a. Beta Thalassemia...
49. a. Isolated prolonged...
50. a. Placental insufficiency
51. a. CD 33
52. c. Coronary artery...
53. d. Most common....
54. a. Periungual...
55. c. Sarcidosis...
56. a. Thalassemia major
57. b. Staphylococcus...
58. c. Henoch Schonelin...
 
518. DISORDERS OF CARDIOVASCULAR SYSTEM
 
52DISORDERS OF CARDIOVASCULAR SYSTEM (QUESTIONS)
1. Oxygenated blood to the fetus is carried by: (PGI June 07)
  1. Umbilical artery
  2. Umbilical vein
  3. SVC
  4. Pulmonary artery
Ref: Ghai 8/e p396; Nelson 19/e p1529
2. Which of the following malformation in a newborn is specific for maternal insulin dependent diabetes mellitus? (AI 06)
  1. Transposition of great arteries
  2. Caudal regression
  3. Holoprosencephaly
  4. Meningomyelocele
Ref: Ghai 8/e p400; Current Diagnosis and treatment Obstetrics and Gynaecology 10/e p312; Dutta 6/e p287
3. Which of the following syndromes is best associated with congenital heart disease? (AIIMS May 05)
  1. Lesch-Nyhan syndrome
  2. Rasmussen syndrome
  3. Holt Oram syndrome
  4. Leopard syndrome
Ref: Ghai 8/e p402; 7/e p400; Nelson 18/e p1883, 1885 and 17/e p1501; Harrison 16/e p1382; Nelson 19/e p1531
4. ASD is seen in all except: (PGI June 01)
  1. Turner’s syndrome
  2. Ellis van creveld syndrome
  3. Down’s syndrome
  4. Halt-oram syndrome
  5. TAR syndrome
Ref: Ghai 8/e p403; 7/e p400; Nelson 8/e, p2386, 2389 and 17/e p1501; Harrison 16/e p1382; Nelson 19/e p1531
5. A child after 4 weeks of birth acyanotic, ejection systolic murmur detected causes are: (PGI June 08)
  1. VSO
  2. PDA
  3. TOF
  4. Coarctation of aorta
  5. Tricurpid stenosis
Ref: Ghai 8/e p413; 7/e p419; Nelson 19/e p1568
6. Cyanosis is seen in: (PGI Dec 01)
  1. Persistent ductus arteriosus
  2. Tricuspid atresia
  3. Ostium premium ASD
  4. Eisenmenger complex
  5. Tetralogy of Fallot
Ref: Ghai 8/e p420
7. Which one of the following does not produce cyanosis in the first year of life: (AIIMS May 03)
  1. Atrial septal defect
  2. Hypoplastic left heart syndrome
  3. Truncus arteriosus
  4. Double outlet right ventricle
Ref: Ghai 8/e p402; Myung K Park 5/e, p75; Nelson 19/e p1551
8. In which of the following conditions left atrium is not enlarged: (AI 06)
  1. Ventricular septal defect
  2. Atrial septal defect
  3. Aortopulmonary window
  4. Patent ductus arteriosus
Ref: Ghai Ghai 8/e p402-403; 7/e p402; Nelson 18/e p1888-1889 and 17/e p1514; Nelson 19/e p1554, 1555
9. All of the following are true about ASD except: (AI 01)
  1. Right atrial hypertrophy
  2. Left atrial hypertrophy
  3. Right ventricular hypertrophy
  4. Pulmonary hypertension
Ref: Ghai 8/e p402; 7/e p401-402; Nelson 18/e p1883, 1884; Nelson 19/e p1554
10. A young female presents with history of dyspnea on exertion. On examination, she has wide, fixed split S2 with ejection systolic murmur (m/VI) in left second intercostal space. Her ECG shows left axis deviation. The most probable diagnosis is: (AIIMS May 2003)
  1. Total anomalous pulmonary venous drainge
  2. Tricuspid atresia
  3. Ostium prirnum atrial septal defect
  4. Ventricular septal defect with pulmonary arterial hypertension
Ref: Ghai 8/e p402; 7/e p402; Nelson 18/e p1883, 1884
11. A patient presents with LVH. And pulmonary complications. ECG, shows left axis deviation. Most likely diagnosis is: (AI 01)
  1. TOP
  2. Tricuspid atresia
  3. TAPVC
  4. VSD
Ref: Ghai 8/e p429; 7/e p403-404; Nelson 18/e p1913, 1914; Nelson 19/e p1556, 1557
12. A child with large perimembranous VSD has congestive heart failure. What may be the cause of improvement of cardiac failure in the patient? (AIIMS Nov 01)
  1. Aortic regurgitation
  2. Vascular changes in pulmonary circulation
  3. Infective endocarditis
  4. Closure of VSD spontaneously
Ref: Ghai 8/e p396; 7/e p403-404; Nelson 8/e, p1888-1891 and 17/e p1508; Nelson 19/e p1557
13. A child with VSD presents with development of cyanosis because of Eisenmenger physiology. What is the correct sequence of events which leads to this change: (AIIMS Nov 00)
  1. Left to right shunt, pulmonary hypertension, right ventricular hypertrophy, right to left shunt
  2. 53Left to right shunt, right ventricular hypertrophy, pulmonary hypertension, right to left shunt
  3. Pulmonary hypertension, right to left shunt right ventricular hypertrophy, left to right shunt
  4. Left to right shunt, right ventricular hypertrophy, right to left shunt, pulmonary hypertension
Ref: Ghai 8/e p421; 7/e p403-404; Nelson 8/e, p1867 and 17/e p1508; Nelson 19/e p1556
14. A 29-day-old child presents with features of congestive cardiac failure and left ventricular hypertrophy. Auscultation shows a short systolic murmur. Most likely diagnosis is: (AIIMS Nov 00)
  1. Rheumatic fever
  2. Tetralogy of fallot
  3. Transposition of great arteries
  4. Ventricular septal defect
Ref: Ghai 8/e p396; 7/e p404; Nelson 8/e, p1888-1891 and 17/e p1508-1509; Nelson 19/e p1556
15. All of the following statements about Patent Ductus Arteriosus (PDA) are true, except? (AI 08)
  1. It is more common in males than females
  2. It is a common heart lesion in rubella
  3. Treatment is closure of defect by ligation and division of ductus
  4. Hypoxia and immaturity are important in maintaining the patency
Ref: Ghai 8/e p404; 7/e p405-406; Nelson 18/e p1891; Nelson 19/e p1559
16. True statement about Ductus Arteriosus is: (AI 00)
  1. It undergoes anatomic closure within 24 hours of birth
  2. Forms the ligamentum venosum in later life
  3. It is induced to close by high levels of prostaglandins
  4. May cause a machinery murmur by its patency
Ref: Ghai 8/e p402; 7/e p405-406; Nelson 18/e p1863-1864 and 17/e p580; KDT 5/e, p176; Nelson 19/e p1559
17. A premature infant is born with a patent ductus arteriosus. In closure can be stimulated by administration of: (AIIMS May 06)
  1. Prostaglandin analogue
  2. Estrogen
  3. Anti-estrogen compounds
  4. Prostaglandin inhibitors
Ref: Ghai 8/e p402; 7/e p406; Nelson 18/e p1891-1893 and 17/e p580; KDT 5/e, p176; Nelson 19/e p156
18. The most appropriate management for maintaining patency of ductus arteriosus in a neonate is: (AIIMS May 04)
  1. Prostaglandin E1
  2. Oxygen
  3. Nitric oxide
  4. Indomethacin
Ref: Ghai 8/e p402; Nelson 18/e p1856-1857 and 17/e p1580; KDT 5/e p164; Nelson 19/e p1560, 61
19. Right sided aortic arch is most strongly/commonly associated with? (AI 09)
  1. Tetralogy of Fallot (TOF)
  2. Ventricular Septal Defect (VSD) with pulmonary atresia
  3. Corrected TGA
  4. Truncus arteriosus
Ref: Ghai 8/e p419; 7/e p408; Nelson 18/e p1930; Nelson 19/e p1574
20. The following features are true for tetralogy of Fallot, except: (AI 06)
  1. Ventricular septal defect
  2. Right ventricular hypertrophy
  3. Atrial septal defect
  4. Pulmonary stenosis
Ref: Ghai 8/e p403; 7/e p401-402; Nelson 18/e p1883-1886 and 17/e p1524-1526; Nelson 19/e p1573, 1574
21. In which of the following a ‘Corn en Sabot’ shape of the heart is seen: (AI 04)
  1. Tricuspid atresia
  2. Ventricular septal defect
  3. Transposition of great arteries
  4. Tetralogy of Fallot
Ref: Ghai 8/e p405; 7/e p410; Nelson 18/e p1906-1912 and 17/e p1525; Nelson 19/e p1575
22. Potts shunt is: (AI 01)
  1. Right subclavian artery to right pulmonary artery
  2. Descending aorta to left pulmonary artery
  3. Left subclavian to left pulmonary artery
  4. Ascending aorta to right pulmonary artery waterson
Ref: Ghai 8/e p422; 7/e p409
23. Essential criteria for TOF includes all except? (AIIMS Nov 08)
  1. Valvular stenosis
  2. Infundibular stenosis
  3. Over riding of aorta
  4. RVH
Ref: Ghai 8/e p419; Pediatric Cardiology for practioners, Myung K Park, 4/e p189
24. Recurrent respiratory tract infections may occur in all of the following except: (AIIMS Nov 05)
  1. Ventricular septal defect
  2. Tetralogy of Fallot
  3. Transposition of great arteries
  4. Total anomalous venous return
Ref: Ghai 8/e p419; 7/e p408; Nelson 19/e p1576
54
25. All of the following are true regarding Tetralogy of Fallot except: (AIIMS May 05)
  1. Ejection systolic murmur in second intercostal space
  2. Single second heart sound
  3. Predominantly left to right shunt
  4. Normal jugular venous pressure
Ref: Ghai 8/E P419; 7/e p408-409; Nelson 17/e p1524-1526; Nelson 19/e p1574
26. A 6 months old child with Tetralogy of Fallot develops cyanotic spell initiated by crying. Which one of the following drugs you would like to avoid? (AIIMS Nov 04)
  1. Sodium bicarbonate
  2. Propranolol
  3. Phenylephrine
  4. Isoprenaline
Ref: Ghai 8/e p419; 7/e p408-409; Nelson 18/e p1906-1912 and 17/e p1524-1525; Canadian Journal of Anaesthesia 50, 926-929 (03); Nelson 19/e p1576, 1577
27. Fallots tetralogy manifestation are: (PGI Dec 05)
  1. Left axis deviation
  2. Left ventricular hypertrophy
  3. VSD
  4. Blalock taussig shunt is between pulmonary artery and subclavian artery
  5. Morphine is contraindicated in cyanotic spells
Ref: Ghai 8/e p419; 7/e p408-409; Nelson 8/e p1888-1891, 1900-1912 and 17/e p1524-1526
28. Components of Tetralogy of Fallot is/are: (PGI June 04)
  1. VSD
  2. Lt. Ventricular hypertrophy
  3. Lt. Axis deviation
  4. Taussig-Blalock shunt is between pulmonary and subclavian artery
  5. Morphine is given for Cyanosis
Ref: Ghai 8/e p419; 7/e p408-409
29. Not seen in TOF: (PGI Nov 2009)
  1. Squatting relieves spells
  2. Cyanosis
  3. O2, Morphine useful
  4. LVH
  5. PND
  6. Clubbed feet
Ref: OP Ghai 8/e p419
30. A five years old child presents with left ventricular hypertrophy and central cyanosis what is the most probable diagnosis? (AIIMS Nov 00)
  1. Tricuspid atresia
  2. Eisenmenger syndrome
  3. Tetralogy of Fallot
  4. Total anomalous pulmonary venous drainage
Ref: Ghai 8/e p421; 7/e p409-410; Nelson 8/e, p1913, 1914 and 17/e p1531; Nelson 19/e p1580
31. A five day old, full term male infant was severely cyanotic at birth. Prostaglandin E was administered initially and later ballooned atrial septostomy was done which showed improvement in oxygenation. The most likely diagnosis of this infant is: (AI 04)
  1. Tetralogy of Fallot
  2. Transposition of great vessels
  3. Truncus Arteriosus
  4. Tricuspid Atresia
Ref: Ghai 8/e p429; 7/e p413; Nelson 18/e p1918, 1929 and 17/e p524; Nelson 19/e p1582
32. A neonate has recurrent attacks of abdominal pain, restless irritability and diaphoresis on feeding. Cardiac auscultation reveals a nonspecific murmur. He is believed to be at risk for MI Likely diagnosis here is: (AI 01)
  1. ASD
  2. VSD
  3. TOF
  4. Anomalous coronary artery
Ref: Ghai 8/e p400; Nelson 118/e p 1922-1924and 7/e p1546; Nelson 19/e p1598
33. A neonate has central cyanosis and short systolic murmur on the 2nd day of birth. The diagnosis is: (AIIMS May 01)
  1. Tetralogy of Pallet’s
  2. Transposition of great vessels
  3. Atrial septal defect
  4. Ventricular Septal defect
Ref: Ghai 8/e p429; 7/e p408-409; Nelson 8/e, p1906-1912 and 17/e p1524-1525; Nelson 19/e p1582
34. 7 day old baby presented in the emergency department with unconscious, blue in appearance with 85 % in Oxygen saturation. The diagnosis? (PGI Dec 08)
  1. Tetralogy of Fallot
  2. TGA
  3. TAPVC
  4. PDA
Ref: Ghai 8/e p429; Nelson 19/e p1585-1587
35. The most common type of total anomalous pulmonary venous connection is: (AI 05)
  1. Supracardiac
  2. Infracardiac
  3. Mixed
  4. Cardiac
Ref: Ghai 8/e p413; Nelson 19/e p1589
36. A newborn infant presented with congestive heart failure which was not improving on treatment. On examination, there is bulging anterior fontanelle with bruit. CT showed midline hypoechoic lesion with dilated lateral ventricles. Diagnosis is? (AIIMS May 2010)
  1. Vein of Galen malformation
  2. Arachnoid cyst
  3. Teratoma
  4. Encephalocele
Ref: Ghai 8/e p419; Nelson 18/e p1988
55
37. A newborn has congenital heart failure, not improving on treatment. He has bulging anterior fontanelles with a bruit on auscultation. On trans fontanelle USG a hypoechoic midline mass is seen with dilated lateral ventricles. Most probablediagnosis is: (AIIMS Nov 06)
  1. Vein of Galen malformation
  2. Arachnoid cyst
  3. Medulloblastoma
  4. Encephalocele
Ref: Ghai 8/e p419; Various Books
38. Eisenmenger syndrome is characterized by all except:
  1. Return of left ventricle and right ventricle to normal size
  2. Pulmonary veins not distended
  3. Pruning of peripheral pulmonary arteries
  4. Dilatation of central pulmonary arteries
Ref: Ghai 8/e p428; 7/e p400-401; Nelson 18/e p1727, 1936-1937 and 17/e p1531; Nelson 19/e p1601
39. Eisenmenger complex is common in adult in: (PGI June 00)
  1. VSD
  2. ASD
  3. PDA
  4. Cushion defect
Ref: Ghai 8/e p427; 7/e p405, 409-414; Nelson 8/e, p1727, 1936, 1937 and 17/e p1531; Nelson 19/e p1601, 1602
40. All are signs of impending Eisenmenger syndrome except: (AIIMS May 2010)
  1. Increased flow murmur across tricuspid and pulmonary valve
  2. Single S2
  3. Increased intensity of P2
  4. Graham Steell murmur
Ref: Ghai 8/e p428; 1. Nelson textbook of Pediatrics, 18”1 e.; 2. Harrison’s principles of Internal Medicine, 17/e; Nelson 19/e p1601
41. A child with VSD presents with development of cyanosis because of Eisenmenger physiology. What is the correct sequence of events which leads to this change: (AIIMS Nov 2000)
  1. Left to right shunt, pulmonary hypertension, right, ventricular hypertrophy, right to left shunt
  2. Left to right shunt, right ventricular hypertrophy, pulmonary hypertension, right to left shunt
  3. Pulmonary hypertension, right to left shunt right ventricular hypertrophy, left to right shunt
  4. Left to right shunt, right ventricular hypertrophy, right to left shunt, pulmonary hypertension
Ref: Ghai8/e p428; Nelson 18/e p1936; Nelson 19/e p1601
42. Which condition is most commonly associated with coarctation of aorta? (AI 09, AI 08)
  1. PDA
  2. Bicuspid aortic valve
  3. Aortic stenosis
  4. VSD
Ref: Ghai 8/e p425; 7/e p419; Nelson 18/e p1900; Nelson 19/e p1567
43. A 1 month old boy is referred for failure to thrive. On examination, he shows feature of congestive failure. The femoral pulses are feeble as compared to branhial pulses. The most likely clinical diagnosis is: (AI 06)
  1. Congenital aortic stenosis
  2. Coarctation of aorta
  3. Patent ductus arteriosus
  4. Congenital aortoiliac disease
Ref: Ghai 8/e p423; 7/e p419; Nelson 18/e p1900-1903 and 17/e p520; Nelson 19/e p1567, 1568
44. In post ductal coarctation of the aorta, blood flow to the lower limb is maintained through which of the following arteries: (AIIMS Nov 07)
  1. Umblical artery and subcostal arteries
  2. Thoracic and pericardiophrenic arteries
  3. Intercostal arteries and superior epigastric artery
  4. Ant and post circumflex arteries
Ref: www.emedicine.com/radio/topic42.htm, http://perfline.com/student/coa.html, Grays anatomy 38/e Section 10 cardiovascular system; Nelson 19/e p1568; Ghai 8/e p432
45. All of the following causes death in coarctation of Aorta except: (PGI June 2000)
  1. Infective endocarditis
  2. CCF
  3. Intra cranial hemorrhage
  4. Anterior MI
Ref: Ghai 8/e p423
46. A 4½ years old girl always had to wear warm socks even in summer season. On physical examination, it was noticed that she had high blood pressure and her femoral pulse was weak as compared to radial and carotid pulse, a chest radiography showed remarkable notching of ribs along with their lower borders. This was due to: (AIIMS Nov 02)
  1. Femoral artery thrombosis
  2. Coarctation of aorta
  3. Raynaud’s disease
  4. Takayasa arteritis
Ref: Ghai 8/e p423
47. A child after 4 weeks of birth acyanotic, ejection systolic murmur detected causes are/is: (PGI June 08)
  1. VSD
  2. PDA
  3. TOP
  4. Coarctation of aorta
  5. Tricuspid stenosis
Ref: Ghai 8/e p423; 428
48. About carey Comb’s murmur which is false: (AIIMS Nov 06)
  1. Delayed diastole murmur
  2. Seen in rheumatic fever
  3. Can be associated with AR
  4. Low pitched murmur
Ref: Ghai 8/e p423; Nelson 19/e p1627
49. True about Rheumatic fever: (PGI Dec 03)
  1. Chorea is aggravated during pregnancy
  2. Chorea and arthritis co-existing
  3. Subcutaneous nodules are tender
  4. Erythema multiforme seen
Ref: Ghai 8/e p433-435; Harrison 16/e p1978; Nelson 18/e p1142
56
50. True about subcutaneous nodule in Rheumatic fever: (PGI Dec 07)
  1. Non tender
  2. Most common manifestation
  3. Present on extensor surfaces
  4. Associated with arthritis
Ref: Ghai 8/e p433; Nelson 18/e p1142, 1143; Nelson 19/e p1627, 1628
51. Drug of choice for Rheumatic fever prophylaxis in penicillin allergic patient: (AIIMS May 07)
  1. Erythromycin
  2. Clindamycin
  3. Vancomycin
  4. Gentamycin
Ref: Ghai 8/e p432; Nelson 19/e p1627
52. Which of the following is a minor criteria for diagnosis of Rheumatic fever (RF) according to modified Jones criteria: (AI 07)
  1. ASO titer
  2. Past history of Rheumatic fever
  3. Fever
  4. Subcutaneous nodules
Ref: Nelson 18/e p1141; Ghai 8/e p432-435
53. 8 years old child presented with altered sensorium and seizure. On examination BP was 180/120. Correct statements: (PGI Dec 04)
  1. Sodium nitroprusside strips
  2. Cause of hypertension is essential hypertension
  3. IV labetolol, hydralazine, and diaoxide are given
  4. Nifedipine is used
  5. Pheochromocytoma mimics the condition
Ref: Ghai 7/e p434-435, 6/e p436; Nelson 8/e, p 1993, 1994 and 17/e p1597, 1919; Nelson 19/e p1645
54. Which of the following statements regarding Kawasaki disease is true: (AI 08)
  1. Associated with coronary artery aneurysm into 25% of untreated cases
  2. It is the most common cause of vasculitis in children
  3. IV immunoglobulins are recommended only if coronary artery is involved
  4. Lymph node biopsy is used for diagnosis
Ref: Nelson 19/e p1638; Internet
55. Cardiac tumor in childhood includes? (PGI Dec 08)
  1. Rhabdomyoma
  2. Lymphoma
  3. Atrial myxoma
  4. Sarcoma
  5. Fibroma
Ref: Ghai 8/e p599-622; Nelson 17/e p1581; 18/e p1975; Nelson 19/e p1637
56. Bacterial endocarditis is most commonly caused by: (PGI Dec 03)
  1. Hemolytic Streptococci
  2. Hemolytic Streptococci
  3. Staphylococcus aureus
  4. Cardiobacterium
  5. Staph epidermidis
Ref: Ghai 8/e p443; Harrison 17/e p790; Nelson 18/e p1953
57. Commonest cause of enlarged cardiac shadow in X – ray of a child is: (Karnataka 00)
  1. PDA
  2. Coarctation of Aorta
  3. Pericarditis
  4. Rheumatic carditis
Ref: Ghai 8/e p433; 7/e p384
58. Uncommon finding in congestive cardiac failure in newborn: (CMC 01)
  1. Tachycardia
  2. Tachypnoea
  3. Hepatomegaly
  4. Pedal edema
Ref: O.P. Ghai 8/e p396; 7/e p375
59. In a patient of rheumatic carditis dose of steroid is given for: (Kerala 04)
  1. 3 weeks
  2. 6 weeks
  3. 9 weeks
  4. 12 weeks
Ref: Ghai 8/e p433; 7/e p383
60. The following statements are true of patent ductus arteriosus (PDA) except: (Kerala 00)
  1. Spontaneous closure occurs in some term infants
  2. Pulmonary hypertension develops
  3. Bacterial endocarditis is more frequent with small PDA
  4. Recurrent chest infection and congestive failure may develop
  5. Anatomic existence of PDA is an indication for surgery
Ref: Ghai 8/e p415; 7/e p405; Nelson 17/e p1511; Harrison 16/e p1386
61. Which one of the following is the most common cause of cyanotic heart disease: (UPSC 02)
  1. Dextrocardia
  2. Fallot’s tetralogy
  3. Atrial septal defect
  4. Coarctation of aorta
Ref: Ghai 8/e p413; 7/e p408
62. Not a feature of Fallot’s tetrology: (MAHE 05)
  1. Left ventricular hypertrophy
  2. Boot shaped heart
  3. VSD
  4. Overriding of arch of aorta
Ref: Ghai 8/e p413; 7/e p408
63. All of the following are acyanotic congenital heart disease except: (SGPGI 05)
  1. VSD
  2. PDA
  3. ASD
  4. Tetralogy of fallot
Ref: Ghai 8/e p413-415; 7/e p408
64. Children born to mothers with systemic lupus erythematosis are likely to have one of the following anomalies: (Karnataka 03)
  1. Atrial septal defect
  2. Tetralogy of fallot
  3. Transposition of great vessels
  4. Complete heart block
Ref: Ghai 8/e p457; Nelson 17/e p1563
57
65. The treatment of choice for a case of congestive failure with hypertension is: (JIPMER 04)
  1. ACE inhibitors
  2. a – blockers
  3. Cat 2 channel blockers
  4. Nitrates
Ref: Ghai 8/e p396; Harrison 16/e p1478 table (230.11)
66. Commonest cause of heart failure in infancy is: (Corned 08)
  1. Myocarditis
  2. Rheumatic fever
  3. Cardiomyopathy
  4. Congenital heart disease
Ref: OP Ghai 8/e p396; 7/e p375
67. Most common type of atrial septal defect is: (Corned 08)
  1. Ostium primum
  2. Ostium secondum
  3. Endocardial cushion defect
  4. Endocardial hypertrophy
Ref: OP Ghai 8/e p402
68. A two-year-old boy presented with episodes of becoming dusky. On examination, there was central cyanosis and dubbin. There was no pallor, edema or respiratory distress. The heart was normal sized with a parasternal heave. A systolic thrill was palpable over the left middle sternal -border. First heart sound was normal and only the aortic component was audile in the second heart sound. Liver was not enlarged: (UPSC 07)
What would be the likely diagnosisa.
  1. Congenital methemoglobinemia
  2. Eisenmenger syndrome
  3. Aortic stenosis
  4. Tetralogy of Fallot
Ref: OP Ghai 8/e p413; 7/e p408
69. All of the following are true about Kawasaki disease except: (AI 07; AIIMS May 03)
  1. Purulent conjunctivitis
  2. Swelling of limb
  3. Rash
  4. Fever
Ref: Ghai 6/e p586; Nelson 18/e p1036-1041 and 17/e p825; Nelson 19/e p862; 864
70. Treatment of kawasaki disease in children is: (AI 01)
  1. Oral steroids
  2. IV steroids
  3. IV/Ig
  4. Mycophenolate mofentil
Ref: Ghai 7/e p606, 6/e p586; Nelson 18/e, 1036-1041 and 17/e p825; Nelson 19/e p866
71. A newborn presents with congestive heart failure, on examination has bulging anterior fontanelle with a bruit on auscultation. Transfontanelle USG shows a hypoechoic midline mass with dilated lateral ventricles. Most likely diagnosis is: (AIIMS Nov 06, AI 07)
  1. Medulloblastoma
  2. Encephalocele
  3. Vein of Galen malformation
  4. Arachnoid cyst
Ref: Nelson 18/e p1988, www.emedicine.com/neuro/topic538.htm, www.veinofgalen.co.uk; Nelson 19/e p2009; Internet
72. Pentalogy of fallot has which one of following extra entities:
  1. ASD
  2. VSD
  3. RVH
  4. Pulmonary stenosis
Ref: Ghai 8/e p403; 7/e p409; Nelson 18/e p1906-1912
73. A neonate has central cyanosis and short systolic murmur on the 2nd day of birth. The diagnosis is: (DNB 2007)
  1. Tetralogy of Fallot’s
  2. Transposition of great vessels
  3. Atrial septal defect
  4. Ventricular Septal defect
Ref: Nelson, 16/e p1385: OP Ghai 8/e p624
74. Commonest cause of heart failure in infancy is: (DNB 2007)
  1. Myocarditis
  2. Rheumatic fever
  3. Cardiomyopathy
  4. Congenital heart disease
Ref: Op Ghai Pediatrics, 8/e p445
75. Essential criteria for TOF includes all, except: (DNB 2008)
  1. Valvular regurgitation
  2. Infundibular stenosis
  3. Over riding of aorta
  4. RVH
Ref: OP Ghai 6/e p430
76. What constitutes Pentalogy of Fallot: (DNB 2008)
  1. TOF + PDA
  2. TOF + ASD
  3. TOF+ COA
  4. TOF + Polysplenia
Ref: Ghai 8/e p542; Nelson Pediatrics 18/e p1906, 1911
77. Most sensitive indicator of intravascular volume in infant is: (DNB 2008)
  1. Cardiac output
  2. Heart rate
  3. Stroke volume
  4. Preload
Ref: Ghai 8/e p512; CPDT 18/e p375
78. ASD with a murmur similar to MR and LAD on ECG is having: (DNB 2009)
  1. Ostium premium defect
  2. Ostium secondum defect
  3. VSD
  4. TGA
Ref: OP Ghai 8/e p510
79. Most common cause of death in PDA is: (DNB 2009)
  1. Respiratory compromise
  2. Cardiac compromise
  3. Infective endocarditis
  4. Embolization
Ref: OP Ghai 8/e p521; 405, 406, 407
58
80. Congenital long QT syndrome is associated with neonatal: (DNB 2009)
  1. Sinus bradycardia
  2. Sinus tachycardia
  3. Supra ventricular. tachycardia
  4. Ventricular tachycardia
Ref: 8/e p524; Campbell Text book of Peds, 10/e p589
81. Drug of choice for Rheumatic fever prophylaxis in penicillin allergic patient: (DNB 2009)
  1. Erythromycin
  2. Clindamycin
  3. Vancomycin
  4. Gentamycin
Ref: Ghai 8/e p524; Essentials of paediatrics Nelson 4/e p343-344, 590
82. Essential criteria for TOF includes all, except: (DNB 2009)
  1. Valvular regurgitation
  2. Infundibular stenosis
  3. Over riding of aorta
  4. RVH
Ref: OP Ghai 8/e p532
83. Most sensitive indicator of intravascular volume in infant is: (DNB 2009)
  1. Cardiac output
  2. Heart rate
  3. Stroke volume
  4. Preload
Ref: Ghai 8/e p498; CPDT 18/e p375
84. The commonest cause of death in diphtheric child is: (DNB 2010)
  1. IIIrd nerve palsy
  2. Tonsilitis
  3. Myocarditis
  4. Septicemia
Ref: OP Ghai 8/e p400; Nelson 18/e
85. Commonest cause of heart failure in infancy is: (DNB 2010)
  1. Myocarditis
  2. Rheumatic fever
  3. Cardiomyopathy
  4. Congenital heart disease
Ref: Op Ghai Pediatrics 8/e p512
86. All of the following are component of Tetralogy of Fallot (TOF) except:
  1. Ventricular septal defect
  2. Pulmonic stenosis
  3. Overrriding or dextraposed aorta
  4. Left ventricular hppertrophy
Ref: Ghai 8/e p420; Nelson’s 18/e p1906
87. Congenital heart disease with left ventricular hypertrophy is seen in: (MP PG 2008)
  1. Tricuspid atresia
  2. Transpositio of great vessels
  3. Total anomalous pulmonary venous connection
  4. Ebstein anomaly
Ref: Ghai 8/e p422; 7/e p411
88. Constituents of Tetralogy of fallot (TOF) are all except: (MP PG 2008)
  1. Aortic stenosis
  2. Pulmonary stenosis
  3. Overrriding of aorta
  4. VSD
Ref: Ghai 8/e p420; 7/e p408
89. A new born child presented with CHD has cyanosis, become prominanat on breathing and improved in crying. The diagnosis: (MP PG 2008)
  1. Bilateral choanal atresia
  2. Diaphragmatic hernia
  3. Genitourinary defects
  4. Coloboma
Ref: Ghai 8/ep366; 7/e p336
90. Average central aortic pressure in full-term neonate is ______ mm Hg? (MHPGM-CET 2010)
  1. 75/50
  2. 60/40
  3. 40/20
  4. 20/10
Ref: Nelson paediatrics 18/e p1856; OP Ghai 6/e p393
91. “Atrilized Right Ventricle” is a feature of: (AP 2010)
  1. Tricuspid atresia
  2. Ebstein’s anomaly
  3. Pulmonary artesia
  4. Fallot’s tetralogy
Ref: Ghai 8/e p423
92. ASD with left axis deviation: (AP 2011)
  1. Ostium Primum defect
  2. Floppy mitral valve
  3. Ostium secundum
  4. Cleft mitral valve
Ref: Ghai 8/e p413
93. Not true in rheumatic fever: (AP 2011)
  1. 5-15 yrs age group
  2. Migrating arthritis
  3. Large joints involved
  4. Arthritis persists for > 6 months
Ref: Ghai 8/e p435
94. A child with tetralogy of fallot uses which of the following positions? (DP PGMEE 2010)
  1. Supine
  2. Prone
  3. Squatting
  4. Leaning forwards
Ref: Ghai 8/e p420
95. The best position for examination of cardiac murmurs in a child is: (Feb DP PGMEE 2009)
  1. Sitting
  2. Standing
  3. Right lateral
  4. Recumbent
Ref: Ghai 8/e p457
96. All are characteristic of fallot’s tetralogy, except: (Feb DP PGMEE 2009)
  1. Infundibular stenosis
  2. Ventricular septal defect
  3. Overriding of aorta
  4. Left ventricular hypertrophy
Ref: Ghai 8/e p420
97. Fallot tetralogy true is: (Raj PGI 2008)
  1. Central cyanosis
  2. Pulmonary valvular stenosis
  3. Sign symptom during newborn
  4. Clonidine
Ref: Ghai 8/e p420
59
98. A child with bullous lesion which is best test: (Raj PGI 2008)
  1. VDRL of mother & child
  2. ASLO
  3. P24
  4. HLA B27
Ref: Ghai 8/e p272
99. Causes of congestive Heart failure in newborn are all except: (WB PGI 08)
  1. ASD
  2. TGA
  3. Hypoplastic left ventricle
  4. Baby of diabetic mother
Ref: Ghai 8/e p396; DCD 6/e p287
100. Least common cause of pericarditis/pericardial effusion in children: (WB PGI 08)
  1. Rheumatic fever
  2. Sarcoidosis
  3. Rheumatoid Arthritis
  4. Systemic Lupus Erythematosus
Ref: Nelson 17/e p1579; Ghai 8/e p450
101. The commonest arrhythmia in children is: (J & K PGI 2010)
  1. Sinus tachycardia
  2. Supraventicular tachycardia
  3. Sinus bradycardia
  4. Atrial fibrillation
Ref: OP Ghai 8/e p458
 
 
Ans.
1. b. Umbilical vein
2. b. Caudal regression...
3. c. Holt Oram syndrome
4. a and e
5. a, b and d
6. b, d and e
7. a. Atrial septal defect
8. b. Atrial septal defect
9. b. Left atrial...
10. c. Ostium prirnum...
11. b. Tricuspid atresia
12. b. Vascular changes...
13. b. Left to right shunt...
14. d. Ventricular septal...
15. a. It is more common...
16. d. May cause a...
17. d. Prostaglandin...
18. a. Prostaglandin E1...
19. a. Tetralogy of Fallot (TOF)
20. c. Atrial septal defect...
21. d. Tetralogy of Fallot
22. b. Descending aorta...
23. a. Valvular stenosis
24. b. Tetralogy of Fallot
25. c. Predominantly...
26. d. Isoprenaline
27. c and d
28. a, d and e
29. d and e
30. a. Tricuspid atresia
31. b. Transposition...
32. d. Anomalous...
33. b. Transposition...
34. b and c
35. a. Supracardiac
36. a. Vein of Galen...
37. a. Vein of Galen...
38. a. Return of left...
39. a, b and c
40. a. Increased flow...
41. b. Left to right...
42. b. Bicuspid aortic valve
43. b. Coarctation of aorta
44. c. Intercostal arteries...
45. d. Anterior MI
46. b. Coarctation of aorta
47. a, b and d
48. c. Can be associated...
49. a. Chorea is...
50. a and c
51. a. Erythromycin
52. c. Fever
53. a, d and e
54. a. Associated with...
55. a, c and e
56. c. Staphylococcus...
57. c. Pericarditis
58. d. Pedal edema
59. a. 3 weeks
60. a and e
61. b. Fallot’s tetralogy
62. a. Left ventricular...
63. d. Tetralogy of fallot...
64. d. Complete heart block
65. a. ACE inhibitors
66. d. Congenital heart...
67. b. Ostium secondum
68. d. Tetralogy of Fallot
69. a. Purulent...
70. c. IV/Ig
71. c. Vein of Galen...
72. A. ASD
73. b. Transposition...
74. d. Congenital heart disease
75. a. Valvular regurgitation
76. b. TOF + ASD
77. b. Heart rate
78. a. Ostium premium...
79. b. Cardiac compromise
80. a. Sinus bradycardia
81. a. Erythromycin
82. a. Valvular regurgitation
83. b. Heart rate
84. c. Myocarditis
85. d. Congenital heart...
86. d. Left ventricular...
87. a. Tricuspid atresia
88. a. Aortic stenosis
89. a. Bilateral choanal
90. a. 75/50
91. b. Ebstein’s anomaly
92. a. Ostium Primum...
93. d. Arthritis persists...
94. c. Squatting
95. c. Right lateral
96. d. Left ventricular
97. a. Central cyanosis
98. a. VDRL of mother...
99. a. ASD
100. b. Sarcoidosis
101. a. Sinus tachycardia
 
609. DISORDERS OF RESPIRATORY SYSTEM
 
61DISORDERS OF RESPIRATORY SYSTEM (QUESTIONS)
1. Wheeze in children caused by: (PGI June 05, 06)
  1. Foreign body
  2. Gastroesophageal reflux disease
  3. Bronchial asthma
  4. Epiglottis
  5. Laryngomalacia
Ref: Ghai 8/e p382; 7/e p341, 347; Nelson 19/e p1431
2. A 3-month-old child presents with intermittent stridor. Most likely cause is: (AI 01, AIIMS Dec 95)
  1. Laryngotracheobronchitis
  2. Laryngomalacia
  3. Respiratory obstruction
  4. Foreign body aspiration
Ref: Ghai 8/e p371; 7/e p340; Nelson 18/e p1767; Nelson 19/e p1445, 1446
3. A child with three days history of upper respiratory tract infection presents with stridor, which decreases on lying down postion. What is the most probable diagnosis: (AI 07)
  1. Acute Epiglottitis
  2. Laryngotracheobronchitiff
  3. Foreign body aspiration
  4. Retropharyngeal abscess
Ref: Ghai 8/e p374; 7/e, 9 339; Nelson 18/e p1763; Nelson 19/e p1445, 1446
4. A 2-years-old child is brought to emergency at 3 AM with fever, barking cough and stridor only while crying. The child was able to drink normally. On examination respirator rate is 36/min and temperature is 39.6°c. What will be your next step: (AIIMS Nov 08)
  1. Racemic epinephrine nebulization
  2. High dose dexamethasone injection
  3. Nasal wash for influenza or RS V
  4. Antibiotics and blood culture
Ref: Nelson 18/e p1765; Ghai 8/e p375; 7/e p339, 3406/e p339, 340; Nelson 19/e p1446
5. A 4 year old child has ‘seal barking’ like croupy cough. Management includes all except: (PGI June 00)
  1. O2 inhalation
  2. Antibiotic
  3. Hydration
  4. Morphine
Ref: Ghai 8/e p374-375; 7/e p339; Nelson 19/e p1448
6. The most common etiological agent for acute bronchiolitis in infancy is: (AI 06)
  1. Influenza virus
  2. Para influenza virus
  3. Rhinovirus
  4. Respiratory syncytial virus
Ref: Ghai 8/e p374; 7/e p356; Nelson 18/e p1388-1390 and 17/e p1416; Nelson 19/e p1446
7. Bronchiolitis in children is caused by: (PGI June 05)
  1. H. influenza
  2. RSV
  3. Mycoplasma
  4. EBV
  5. Influenza virus
Ref: Ghai 8/e p374; 7/e p356-357; Nelson 18/e p1773-1777 and 17/e p1416; Nelson 19/e p144
8. In Bronchiolitis followings is/are seen: (PGI Dec 02)
  1. Seen in children 5 months to 3 years of age
  2. Caused by streptococcus pneumoniae
  3. Chest X-ray shows hyperinflation bilaterally
  4. Symptomatic treatment is given
  5. Antibiotics should be started
Ref: Ghai 8/e p374-375; 7/e p357; Nelson’s Essentials of Pediatrics 5/e, p502; Nelson 19/e p1446
9. Which of the following is/are true about bronchiolitis in children: (PGI June 01)
  1. Caused by respiratory syncytial virus
  2. Hyperinflation of the chest
  3. Pleural effusion
  4. May lead to bronchial asthma later in life
  5. Lymphopenia is seen
Ref: Ghai 8/e p375; 7/e p357; Nelson 19/e p1445
10. Which of the following is the etiological agent most often associated with Epiglottitis in children: (AIIMS May 05, Nov 04, May 94)
  1. Streptococcus influenza
  2. Hemophilus influenza type b
  3. Neisseria sp
  4. Moraxella catarrhalis
Ref: Ghai 8/e p374; CPDT 18/e p501; Nelson 19/e p1445
11. A 6 months old baby coming with H/o increasing difficulty in breathing of 2 days duration and on examination baby is afebrile and B/L wheeze and C X R shows B/L hyperinflation of the lungs with normal WBC count, the diagnosis is: (PGI Dec 03)
  1. Bronchiolitis
  2. Asthma
  3. Ch. Bronchitis
  4. Pneumonia
  5. F.b.
Ref: Ghai 8/e p374; Nelson 18/e p514; Nelson 19/e p1446
12. A 11 months old child presents with complaints of respiratory distress. On examination there is bilateral crepitation and wheezing. Which of the following is the most likely cause: (AIIMS Nov 2K)
  1. Pneumonia
  2. Adenovirus
  3. Respiratory synchytial virus
  4. Rhinovirus
Ref: Ghai 8/e p375; Nelson 17/e p1416; Nelson 19/e p1434
62
13. A month old HIV positive child following URTI developed sudden onset of breathlessness. The chest X-ray shows hyperinflation. The O2 saturation was greater than 90%. The treatment of choice is: (AIIMS May 01)
  1. Cotrimoxazole
  2. Ribavarin
  3. IV Ganciclovir
  4. Nebulized Acyclovir
Ref: Ghai 8/e p229; 7/e p358; Nelson 18/e p1773-1777 and 17/e p1415-1416; Nelson 19/e p1448
14. A child with pyoderma becomes toxic and presents with respiratory distress. His chest radiograph shows patchy areas of consolidation and multiple bilateral thin walled air containing cysts. The most likely etiological agent in this case is: (AIIMS Nov 03)
  1. Mycobacterium tuberculosis
  2. Staphylococcus aureus
  3. Mycobacterium avium intracellular
  4. Pneumocystis carinii
Ref: Ghai 8/e p376; 7/e p353; Nelson 19/e p1476
15. WHO criteria for hospital admission in pneumonia: (PGI Dec 08)
  1. High fever
  2. Nasal flaring
  3. Difficulty in breathing
  4. Difficulty in feeding
  5. Chest indrawing
Ref: Ghai 8/e p376; Park 18/e p143
16. Which one of the following is the leading cause of mortality under five years of children in developing countries? (AIIMS May 05)
  1. Malaria
  2. Acute lower respiratory tract infections
  3. Hepatitis
  4. Prematurity
Ref: Ghai 8/e p376
17. A 7½ months old child with cough, mild stridor is started on oral antibiotics. The child showed initial improvement but later developed wheeze, productive cough, and mild fever. X-ray shows hyperlucency and PFT shows an obstructive curve. The most probable diagnosis is: (AI 08)
  1. Bronchiolitis obliterans
  2. Post viral syndrome
  3. Pulmonary alveolar microlithiasis
  4. Follicular bronchitis
Ref: Ghai 8/e p374; Nelson 19/e p1463
18. A 7½-year-old girl presents with breathlessness and fever for 6-7 days. She has non-productive cough for 6 months. X-ray chest shows hyperlucency in the lungs and pulmonary function tests show obstructive pattern. The most probable diagnosis will be: (AIIMS Nov 08)
  1. Lobar emphysema
  2. Bronchiolitis obliterans
  3. Follicular bronchitis
  4. Pulmonary alveolar microlithiasis
Ref: Ghai 8/e p374; Nelson 19/e p1463
19. A 6 months old baby coming with h/o increasing difficulty in breathing of 2 days duration and on examination baby is afebrile and B/L wheeze and CXR shows B/L hyperinflation of the lungs with normal WBC count, the diagnosis is: (PGI Dec 03)
  1. Bronchiolitis
  2. Asthma
  3. Ch. Bronchitis
  4. Pneumonia
  5. FB
Ref: Ghai 8/e p375; 7/e p357; Nelson 19/e p1463
20. A child presents raised sweat chloride levels and suspicion of cystic Fibrosis, which other test would you do to exclude the diagnosis of cystic fibrosis: (AI 08)
  1. Repeat sweat chloride measurements
  2. Nasal electrode potential difference
  3. Fat in stool for next 72 hours
  4. DNA analysis for delta F-508 mutation
Ref: Ghai 8/e p393; Nelson 18/e p1808; Nelson 19/e p1482, 1485
21. Features of cystic fibrosis: (PGI June 06)
  1. Lung normal at birth
  2. Abnormal sweat chloride tests
  3. Autosomal dominant
  4. Defect in chromosome
Ref: Ghai 8/e p393; Nelson’s Essentials of Pediatrics 5/e, p648; Nelson 19/e p1487
22. For a normal husband and wife the first child was diagnosed to have cystic fibrosis. What is the percentage of chances for the second child be affected: (PGI June 06)
  1. 25
  2. 50
  3. 0
  4. 75
  5. 100
Ref: Ghai 8/e p394; Nelson’s Essentials of Pediatrics 5/e p224
23. Following are true about bronchial cyst except: (AIIMS 96)
  1. Mostly mediastinal
  2. 50-70% occur in lungs
  3. Usually multiloculated
  4. Are infected quite often
Ref: Schwartz 8/e p597; Nelson 19/e p1466; Internet
24. A 4-year-old child presents with a history of chronic left lower lobe pneumonitis. On contrast bronchography, the area involved with the pneumonitis does not fill whereas the area around it does fill. The most likely diagnosis: (AI 06)
  1. Asthma
  2. Pulmonary sequestration
  3. Cystic fibrosis
  4. Bronchopulmonary dysplasia
  5. Bronchogenic cyst
Ref: Ghai 8/e p372; Nelson 18/e p1784; Nelson 19/e p14650
63
25. Which of the statements is not true regarding Macleod’s Syndrome: (AI 07)
  1. It is not a true emphysema
  2. Occurs before 8 years of age
  3. It is unilateral emphysema
  4. The pulmonary artery on the affected side is hyperplastic
  5. It is also called Swyer - Jame’s Syndrome
Ref: Internet
26. Pneumothorax could be a complication of: (UPSC 2K)
  1. Staphylococcal pneumonia
  2. Pneumococcal pneumonia
  3. Klebsiella pneumonia
  4. Viral pneumonia
Ref: Ghai 8/e p376; 7/e p352, 6/e p349
27. Child requiring repeated short acting broncho-dilators and what could be next line of management: (Manipal 06)
  1. Methylxanthines
  2. Short acting budesonide
  3. Oral prednisolone
  4. Montelukast
Ref: Ghai 8/e p382; 7/e p361; Nelson 17/e p767, 768
28. The correct line of management in child who has swallowed a coin is: (TNPSC 2K)
  1. Fibreoptic endoscopy
  2. Rigid endoscopy
  3. Laparotomy
  4. Wait and watch
Ref: Ghai 8/e p391; Nelson 17/e p1226
29. With references to bronchiolitis, consider the following statement: (UPSC 07)
  1. Is more common among boys
  2. RSV immunoglobulin does not have any role in acute episode
  3. It is common around the age of six months
  4. Mortality is high in it Which ofthe statements given above are correct
  1. 1 and 2 only
  2. 3 and 4 only
  3. 1, 2 and 3 only
  4. 1, 2, 3 and 4
Ref: CPDT 17/e p506; O.P. Ghai 8/e p374; 7/e p357
30. A child presented with mild fever and breathlessness. He was treated and his condition improved over 4 days. Later his condition deteriorated with increase in fever and breathlessness. X-ray showed hyperlucency. What is the probable diagnosis?
  1. Bronchiolitis obliterans
  2. Alveolar proteinosis
  3. Bronchitis
  4. Bronchiolitis
Ref: Ghai 8/e p374; Nelson text book of pediatrics 18/e p1781-82
31. The most important cause of under 5 mortality is: (DNB 2005)
  1. Diarrhoea
  2. Malnutrition
  3. Respiratory infections
  4. Trauma
Ref: O.P. Ghai Peds 8/e p376
32. Commonest cause of stridor in a new born is: (DNB 2005)
  1. Laryngomalacia
  2. Foreign body
  3. Meconium aspiration
  4. Recurrent laryngeal nerve palsy due to birth
Ref: Ghai 8/e p371; Nelson Pediatrics 18/e p1767
33. In a prematurely new borne child with respiratory distress with grunting after 2 hours. Respiratory management would involve: (DNB 2007)
  1. Nasal CPAP
  2. Surfactant with mechanical ventilation
  3. Intubation
  4. Warm oxygen
Ref: Nelsons. 18/e p731-5, Manual of Neonatal Care, 6/e p326-7, OP Ghai 8/e p137; 7/e p144
34. Most common cause of Acute Epiglottitis is: (DNB 2010)
  1. S. pneumoniae
  2. Influenza
  3. H. influenza
  4. Meningococci
Ref: Nelson 18/e p1173, 1174
35. Commonest cause of stridor in a new born is: (DNB 2010)
  1. Laryngomalacia
  2. Foreign body
  3. Meconium aspiration
  4. Recurrent laryngeal nerve palsy due to birth
Ref: Ghai 8/e p371; Nelson Pediatrics 18e p1767
36. A premature baby weighing 1.5 kg, born with emergnancy Cesarean Section at 32 weeks, now develops respiratory distress with grunting the best management would be: (DNB 2010)
  1. C-pap
  2. Mechanical ventilation
  3. Moist oxygen through headbox
  4. Surfactant therapy plus mechanical ventilation
Ref: Ghai 8/e p137; Campbells Textbook of Pediatrics, 10/e p177, OP; Ghai, 6/e p167, Nelson Pediatrics
37. Lung maturity is best assessed by: (DNB 2011)
  1. L:S ratio
  2. Phosphatidyl glycerol estimation
  3. Bilirubin
  4. Lecithin
Ref: Dutta obs 5/e p324; Williams OBS, 22/e Chapter 29
38. True about Alpha 1 antitrypsin deficiency is: (DNB 2011)
  1. X linked dominant
  2. Does not cause pulmonary disease
  3. Is synthesized in liver
  4. Causes decreased sodium in sweat.
Ref: Harrison Medicine, Nelson; Internet
39. Most common cause of inspiratory stridor in a newborn is: (MP PG 2010)
  1. Subglottic stensosis
  2. Tracheo-oesophageal fistula
  3. Laryngomalacia
  4. Dysphagic Lusoria
Ref: Ghai 8/e p368-372; Nelson’s 18/e p1767
64
40. The commonest cause bronchiolitis in children is: (MP PG 2010)
  1. Adenovirus
  2. Respiratory syncytial virus
  3. Klebseilla
  4. Staphyllococci
Ref: Ghai 8/e p387; Nelson’s 18/e p1774
41. Pneumatoceles on chest radiograms in a child with pneumonia are seen in infection: (Karnataka PG 2011)
  1. Staphylococcus & Klebsiella
  2. Pneumococcus
  3. Streptococcus
  4. Hemophilus influenza
Ref: OP Ghai 8/e p377; 7/e p353; Harrison’s, 17/e p876, 1622
42. Gastrointestinal disturbance in cystic fibrosis are all except: (UP PG 09)
  1. Spleen infact
  2. Biliary cirrhosis
  3. Malabsorption
  4. Gallstones
Ref: Ghai 8/e p393; 7/e p369-70; Nelson 18/e p1803
43. Rediological features of transient tachypnea of newborn is: (UP PG 09)
  1. Irregular pulmonary vascular marking
  2. Low lung volumes
  3. Air bronchogram sign
  4. Horizontal fissure
Ref: Ghai 8/e p171; Nelson 18/e p741
44. A 4-year-old child presents with a history of hoarseness, croupy couth and aphonia, the child has dyspnoea with wheezing. The most probable diagnosis is: (DP PGMEE 2010)
  1. Asthmatic bronchitis
  2. Laryngeal foreign body
  3. Bronchopneumonia
  4. Retrophyryngeal abscess
Ref: Ghai 8/e p711; Nelson 18/e p1769-1770
45. A 3-months-old infant presents with intermittent respiratory stridor since 10 days of age. The most likely diagnosis is: (Feb DP PGMEE 2009)
  1. Laryngomalacia
  2. Tracheoesophageal fistula
  3. Laryngotracheobronchitis
  4. Neoplasm
Ref: Ghai 8/e p368
46. Most common site of extra pulmonary TB is seen in: (Raj PG 2008)
  1. Lymph node
  2. Pleura
  3. Meningitis
  4. Vertebra
Ref: Ghai 8/e p250
47. 12-year-old boy came with fever, sore throat and cervical lymph node enlargement. Throat swab positive for group A beta haemolytic streptococci and started on penicillin. But he came back with worsening of symptoms even while on treatment. Next step is: (Kerala PG 09)
  1. Start azithromycin
  2. Blood monospot test and CBC
  3. IgE for IMN
  4. Presume it as EBV and start acyclovir and prednisolone
Ref: Ghai 8/e p215; Nelson 18/e p1375
48. Streptococcal pharyngitis is treated for: (J & K PG 2011)
  1. 3 days
  2. 5 days
  3. 7 days
  4. 10 days
Ref: Ghai 8/e p374
 
 
Ans.
1. a and c
2. b. Laryngomalacia
3. b. Laryngotracheobronchitis
4. b. High dose...
5. b and d
6. d. Respiratory...
7. b, c and e
8. c and d
9. a, b and d
10. b. Hemophilus influenza...
11. a. Bronchiolitis
12. c. Respiratory...
13. b. Ribavarin
14. b. Staphylococcus aureus
15. All of the above
16. b. Acute lower...
17. a. Bronchiolitis...
18. a and b
19. a. Bronchiolitis
20. b. Nasal electrode...
21. a and b
22. c. 0
23. b. 50-70% occur in lungs
24. b. Pulmonary sequestration
25. d. The pulmonary...
26. a. Staphylococcal...
27. b. Short acting...
28. a. Fibreoptic endoscopy
29. c. 1, 2 and 3 only
30. a. Bronchiolitis obliterans
31. c. Respiratory infections
32. a. Laryngomalacia
33. b. Surfactant with...
34. a. S. pneumoniae
35. a. Laryngomalacia
36. d. Surfactant therapy...
37. a > b
38. c. Is synthesized...
39. c. Laryngomalacia
40. b. Respiratory...
41. a. Staphylococcus...
42. a. Spleen infact
43. d. Horizontal fissure
44. b. Laryngeal foreign...
45. a. Laryngomalacia...
46. a. Lymph node
47. b. Blood monospot...
48. d. 10 days
 
6510. DISORDERS OF KIDNEY AND URINARY TRACT
 
66DISORDERS OF KIDNEY AND URINARY TRACT (QUESTIONS)
1. True about infantile polycystic kidney disease include the following except: (AI 08)
  1. Autosomal dominant
  2. Hepatic cysts
  3. Renal cysts present at birth
  4. Periportal fibrosis
Ref: Ghai 8/e p464; 7/e p471; Nelson 17/e p1749; Nelson 19/e p1796
2. Which of the following is the most common renal cystic disease in infant? (AI 05)
  1. Polycystic kidney
  2. Simple renal cyst
  3. Unilateral renal dysplasia
  4. Calyceal cyst
Ref: Ghai 8/e p464-466; 7/e p470; Nelson 17/e p1784; Nelson 19/e p1827
3. Which one of the following is the most common cause of abdominal mass in neonates? (AI 03)
  1. Neuroblastoma
  2. Wilm’s tumor
  3. Distended bladder
  4. Multicystic dysplastic kidneys
Ref: Ghai 8/e p507; 7/e p470; Nelson 17/e p1784; Nelson 19/e p1827
4. A 6-years-old girl presents with recurrent e. coli infection in urine. Ultrasound of abdomen shows Hydroureter and Hydronephrosis. Micturating cystourethrogram shows filling defect in urinary bladder. The likely diagnosis is: (AI 00)
  1. Sacrococcygeal Teratoma
  2. Vesicoureteric Reflux–grade II
  3. Duplication of Ureter
  4. Ureterocele
Ref: Ghai 8/e p505; Nelson 17/e p1799-1800; Nelson 19/e p1843
5. A 6-months-old boy weighing 3.2 kg presents with recurrent vomiting and polyuria. Investigations show blood area 60 mg/dl, creatinine 0.7 mg/dL, calcium 12.8 mg/dL, phosphate 3 mg/dL, pH 7.45, bicarbonate 25 mEq/L and PTH 140 pg ml (normal <60 pg/ml). Daily urinary calcium excretion is reduced. Ultrasound abdomen show bilateral: (AIIMS Nov 05)
  1. Bartter syndrome
  2. Mutation of the calcium sensing receptor
  3. Pseudo-pseudohypoparathyroidism
  4. Parathyroid adenoma
Ref: Nelson 17/e p1895; Harrison 16/e p2256; Nelson 19/e p1816
6. A 12-years-old boy is referred for evaluation of nocturnal enuresis and short stature. The blood pressure is normal. The blood urea is 112 mg/dl, creatinine 6 mg/dl, sodium 119 mEq/L, potassium 4 mEq/L, calcium 7 mg/dl, phosphate 6 mg/dl and alkaline phosphatase 400 U/l. Urinalysis shows trace proteinuria with hyaline casts; no red and white cells are seen. Ultrasound shows bilateral small kidneys and the micturating cystourethrogram is normal. The most likely diagnosis is: [AIIMS Nov. 03]
  1. Alport’s syndrome
  2. Medullary sponge kidney
  3. Chronic glomerulonephritis
  4. Nephronophthisis
Ref: Harrison’s 18/e p1799; Robbin’s 6/e p965, 966; Nelson 19/e p1816
7. A 13-year-old boy is referred for evaluation of nocturnal enuresis and short stature. His blood pressure is normal. The hemoglobin level is 8g/dl. urea 112 mg/dl, creatinine 6 mg/dl, sodium 119 mEq/ dl, potassium 4 mEq/L, calcium 7 mg/dl, phosphate 6 mg/dl and alkaline phosphatase 300 U/l. Urinalysis shows trace proteinuria with hyaline casts; no red and white cells are seen. Ultrasound shows bilateral small kidneys and the micturating cystourethrogram is normal. The most likely diagnosis is: (AIIMS May 03)
  1. Alport’s syndrome
  2. Medullary sponge kidney
  3. Chronic glomerulonephritis
  4. Nephronophthisis
Ref: Harrison’s 18/e p1799; Robbin’s 6/e p965, 966
8. An 8-year-old child suffering from recurrent attacks of polyuria since childhood presents to the pediatrics OPd. On examination the child is short statured vitals and BP are normal. Serum Creatinine - 6 mg%, HCO3 - 16 mEq, Na- 134, IC4.2 On USG bilateral small kidneys are seen. Diagnosis is: (AIIMS May 01)
  1. Reflux nephropathy
  2. Nephronophthisis
  3. Polycystic kidney disease
  4. Medullary cystic kidney disease
Ref: Harrison’s 18/e p1799; Robbin’s 6/e p965, 966
9. A 5-year-old child suffering from nephrotic syndrome is responding well to steroid therapy. What would be the most likely finding on light microscopy: (AI 01)
  1. No finding
  2. Basement membrane thickening
  3. Hypercellular glomeruli
  4. Fusion of foot processes
Ref: Ghai 8/e p477; 7/e p451; Nelson 17/e p1754-1755; Nelson 19/e p1804
67
10. The most common gene defect in ldiopathic steroid resistant nephrotic syndrome: (AIIMS Nov 06; May 07)
  1. ACE
  2. NPHS 2
  3. HOX 11
  4. PAX
Ref: Ghai 8/e p477; Robbin’s 6/e p983, 984; Nelson 19/e p1803
11. A 9-year-old boy has steroid dependent nephrotic syndrome for the last 5 years. The patient is markedly cushingoid with blood pressure of 120/ 86 mmHg and small subcapsular cataracts. The most appropriate therapy of choice is: (AIIMS Nov 04)
  1. Long-term frusemide with enalapril
  2. Cyclophosphamide
  3. Intravenous immunoglobulin
  4. Intravenous pulse corticosteroids
Ref: Ghai 8/e p479; Nelson 17/e p1756; Nelson 19/e p1805
12. A 9-year-old child has steroid dependant nephrotic syndrome for the last 5 years. He has received corticosteroids almost continuously during this period and has cushingoid features. The blood pressure is 120/86 mmHg and there are bilateral subcapsular cataracts. The treatment of choice is: (AIIMS May 03)
  1. Levamisole
  2. Cyclophosphamide
  3. Cyclosporin A
  4. Intravenous pulse corticosteroids
Ref: Ghai 8/e p479; Nelson 17/e p1756; Nelson 19/e p1805
13. Nephrotic syndrome in children is caused by: (PGI Dec 08)
  1. Minimal change disease
  2. RPGN
  3. MPGN O. FSGS
  4. Membranous nephritis
Ref: Ghai 8/e p477; 7/e p451-453; Nelson 18/e p2191, 2192; Nelson 19/e p1803
14. True about Nephrotic syndrome in a Child: (PGI June 03)
  1. Minimal Change disease is commonest cause
  2. Proteinuria of 4 gm/m2/hour is characteristic
  3. Cyclosporin and Azathioprine is mainstay of therapy
  4. Pretreatment biopsy is done in all cases
  5. Spontaneous bacterial peritonitis is associated with it
Ref: Ghai 8/e p477-478; 7/e p451; Nelson 17/e p1753 – 1756; Harrison 16/e p1684-1686; Nelson 19/e p1803
15. Which of the following is included in definition of Nephrotic syndrome: (PGI June 04)
  1. Microalbuminuria
  2. Masisve proteinuria
  3. Microscopic hematuria
  4. Edema
  5. Hyperlipidemia
Ref: Ghai 8/e p478; Robbin’s 7/e p450; Nelson 18/e p2192; Nelson 19/e p1801, 1802
16. The neonatal kidney achieves concentrating ability equivalent to adult’s kidney by: (AI 04)
  1. One year of age
  2. Eighteen months of age
  3. Three to six months of age
  4. Just before puberty
Ref: Ghai 7/e p441-442; Nelson 17/e p1790-1795
17. Which one of the following statements is false with regard to pyuria in children: (AI 03)
  1. Presence of more than 5 WBC/hpf (high power field) for girls and more than 3 WBC/hpf for boys
  2. Infection can occur without pyuria
  3. Pyuria may be present without Urinary tract infection
  4. Isolated pyuria is neither confirmatory nor diagnostic for Urinary tract infection
Ref: Ghai 8/e p483; 7/e p456; Nelson 17/e p1787
18. A 2 month old girl has failure to thrive, polyuria and medullary nephrocalcinosis affecting both kidneys. Investigations show blood pH 7.48, bicarbonate 25 mEq/L, potassium 2 mEq/L, sodium 126 mEq/L and chloride 88 mEq/L. The most likely diagnosis is: (AIIMS Nov 04)
  1. Distal renal tubular acidosis
  2. Primary hyperaldosteronism
  3. Bartter syndrome
  4. Pseudohypoaldosteronism
Ref: Ghai 8/e p505; Nelson 17/e p1763–1764; Nelson 19/e p1813
19. A 10-month-old boy, weighing 3 kg has polyuria, polydipsia and delayed motor milestones. Investigations show blood levels of creatinine 0.5 mg/dl, potassium 3 mEq/L, sodium 125 mEq/L, chloride 88 mEq/L, calcium 8.8 mg/ dl, pH 7.46 and bicarbonate 26 mEq/L. Ultrasonography shows medullary nephrocalcinosis. The most likely diagnosis is: (AIIMS Nov 03)
  1. Renal tubular acidosis
  2. Diabetes insipidus
  3. Bartter syndrome
  4. Pseudohypoaldosteronism
Ref: Ghai 8/e p505; Nelson 17/e p1756; Nelson 19/e p1813
20. 2-year-old child presented with sudden onset of altered sensorium on examination BP was 200/100: (PGI Dec 04)
  1. Renal artery stenosis
  2. Coarctation of Aorta
  3. Glomerulonephritis
  4. Essential hypertension
  5. Pheochromocytoma
Ref: Ghai 6/e p432-434; Nelson 17/e p1592-1594; Nelson 19/e p1785
21. The finnish type of congenital nephritic syndrome occurs due to gene mutation affecting the following protein: (AI 06)
  1. Podocin
  2. Alpha-actinin
  3. Nephrin
  4. CD2 activated protein
Ref: Ghai 8/e p477; Nelson 18/e p2192, 2195; Nelson 19/e p1803
68
22. A female child was brought with complaint of generalized swelling of her body. She was passing cast in her urine. No hematuria. The true statement is: (AI 09)
  1. No IgG or C3 deposition seen on renal biopsy
  2. Her C3 levels could be low
  3. IgA nephropathy is likely
  4. Alport syndrome
Ref: Ghai 8/e p479; 7/e p451; Nelson 17/e p1737-38
23. A 3-year-old male had non blanching rashes over the shin and swelling of knee joint with hematuria +++ and protein +. Microscopic analysis of his renal biopsy specimen is most likely to show? (AIIMS Nov 08)
  1. Tubular necrosis
  2. Visceral podocyte fusion
  3. Mesangial deposits of IgA
  4. Basement membrane thickening
Ref: Ghai 8/e p477; Nelson 17/e p1737-1738; Nelson 19/e p1781
24. A child was diagnosed as a case of pauci-immune crescentic glomerulonephritis. The treatment to be given in this child is: (AIIMS May 02)
  1. Prednisolone + Cyclophosphamide
  2. Methylprednisolone
  3. Cyclophosphamide
  4. Immunoglobins
Ref: Ghai 8/e p474; Harrison 16/e p1683-1684
25. Post-streptococcal glomerulonephritis is associated with: (PGI Dec 01)
  1. Follows skin and throat infection
  2. Antibiotic treatment induces remission
  3. Is a cause of chronic renal failure in majority of children
  4. Low complement level occurs
  5. Caused by all serotypes
Ref: Ghai 8/e p474; 7/e p447, 6/e p447-448
26. Child with BP 190/110, pedal edema ++ ; facial edema ascites -absent. Gross hematuria diagnosis is: (PGI June 00)
  1. Acute GN
  2. Nephrotic syndrome
  3. Renal thrombosis
  4. Renal amyloidosis
Ref: Ghai 8/e p474-476; 7/e p447
27. A 6-year-old child presents with recurrent episodes of gross hematuria for 2 yrs. He is likely to have: (AI 08)
  1. IgA nephropathy
  2. Wilm’s tumor
  3. Henoch Schonlein Purpura (HSP)
  4. Neuroblastoma
Ref: Ghai 8/e p476; 7/e p446; Nelson 18/e p2169, 2171 Ghai, 6/e p446; Nelson 19/e p1781
28. The renal biopsy of a 6 year old boy with recurrent gross hematuria shows IgA nephropathy. The urinary protein excretion is 130 mg/day. Which of the following is the most appropriate next step in the management: (AIIMS May 04)
  1. Administer corticosteroids
  2. Give Azathioprine
  3. Start Cyclosporine
  4. Urinary bag sample
Ref: Ghai 8/e p476; 7/e p446; Nelson 17/e p1737-1738; Harrison 16/e p1690; Nelson 19/e p1782
29. 12-year-of old Shyam presented with gross hematuria with 80% dysmorphic RBC’s days after a attack of upper respiratory tract infection diagnosis is: (AIIMS Nov 01)
  1. Microangiopathic thrombotic anemia
  2. IgA Nephropathy
  3. PSGN
  4. HS purpura
Ref: Robbins 7/e p986–987; Ghai 8/e p476; Nelson 17/e p1737-1738; Nelson 19/e p1781
30. A six year old male baby presents to a hospital with recurrent gross hematuria for 2 years. There is no h/o burning micturition or pyuria. Urine routine examination demonstrated no pus cells and urine culture was sterile. Serum C3 levels were normal. What is the most probable diagnosis? (AI 08)
  1. Wilm’s tumor
  2. IgA nephropathy
  3. Post-streptococcal glomerulonephritis
  4. Urinary tract infection
Ref: Ghai 8/e p476; Other characteristics explained already
31. Henoch-Schonlein purpura is characterized by the deposition of the following immunoglobulin around the vessels: (AIIMS Nov 05)
  1. IgM
  2. IgG
  3. IgA
  4. IgE
Ref: Ghai 8/e p632; Nelson 18/e p2178, 2179; Robbin’s 7/e p650, 986, 987; Nelson 19/e p1789
32. A 8 year old male had non blanching rashes over the shin and swelling of knee joint with hematuria +++ and protein +. Microscopic analysis of his renal biopsy specimen is most likely to show: (AIIMS Nov 07)
  1. Tubular necrosis
  2. Visceral podocyte fusion
  3. Mesangial deposits of IgA
  4. Basement membrane thickening
Ref: Ghai 8/e p476, 473; Harrison 17/e, p2128; Nelson 19/e p1781
33. A picture resembling microthrombin in renal vasculature occurs in: (PGI Dec 01)
  1. Pre eclampsia
  2. Henoch Schonlein
  3. Benign hypertension
  4. Hemolytic uremic syndrome
  5. Malignant hypertension
Ref: Ghai 8/e p492; Robbin’s 6/e p1008; Dutta 6/e p223
34. Most common cause of urinary obstruction in a male infant is: (AI 01)
  1. Anterior urethral valves
  2. Posterior urethral valves
  3. Stone
  4. Stricture
Ref: Ghai 8/e p465; 7/e p470; Nelson 17/e p1802-1803; Nelson 19/e p1845
69
35. A 5-year-old child presents with a calculus of size 2 cm in the upper ureter. He also complains of hematuria. USG shows no further obstruction in the urinary tract. TT of choice for this patient would be: (AI 01)
  1. Ureterolithotomy
  2. Endoscopic removal
  3. ESWL
  4. Observation
Ref: Ghai 8/e p468; 7/e p469
36. A 3-year-old boy has bilateral renal calculi secondary to idiopathic hypercalciuria. The dietary management includes all of the following except: (AIIMS Nov 04)
  1. Increased water intake
  2. Low sodium diet
  3. Reduced calcium intake
  4. Avoidance of meat proteins
Ref: Ghai 8/e p464; 7/e p446; Nelson 17/e p1748
37. A child presents with abdominal colic and hematuria. On ultrasonography a stone 2.5 cm in diameter is seen in the renal pelvis. The next step in management of this case is: (AIIMS Nov 00)
  1. Pyelolithotomy
  2. Nephroureterostomy
  3. Conservative
  4. ESWL
Ref: Ghai 8/e p478; 7/e p469
38. A child with recurrent urinary tract infections is most likely to show: (AI 05)
  1. Posterior urethral valves
  2. Vesicoureteric reflux
  3. Neurogenic bladder
  4. Renal and ureteric calculi
Ref: Ghai 8/e p467; 7/e p457-458; Nelson 17/e p1790-1791; Nelson 19/e p1834, 1835
39. The most common cause of renal scarring in a 3 year old child is: (AI 05)
  1. Trauma
  2. Tuberculosis
  3. Vesicoureteral reflux induced pyelonephritis
  4. Interstitial nephritis
Ref: Ghai 8/e p462; Nelson 17/e p1792; Nelson 19/e p1834, 1836
40. A 6-months-old girl is having recurrent UTI. Ultrasound abdomen shows bilateral hydronephrosis. MCU (Micturating cysto urethrogram) shows bilateral Grad IV vesicoureteral reflux. The treatment of choice is: (AI 02)
  1. Endoscopic injection of polyteflon at the ureteric orifices
  2. Ureteric reimplantation
  3. Bilateral ureterostomy
  4. Prophylactic antibiotics
Ref: Ghai 8/e p462; 7/e p458; Nelson 17/e p1793
41. A 3-year-old boy presents with fever; dysuria and gross hematuria. Physical examination shows a prominent suprapubic area which is dull on percussion. Urinalysis reveals red blood cells but no proteinuria. Which of the following is the most likely diagnosis? (AIIMS May 06)
  1. Acute glomerulonephritis
  2. Urinary tract infection
  3. Posterior urethral valves
  4. Teratoma
Ref: Ghai 8/e p465; 7/e p470; Nelson 17/e p1802-1803; Nelson 19/e p1845
42. The treatment of choice for primary grade V vesicoureteric reflux involving both kidneys in a 6-month-old boy is: (AIIMS Nov 04)
  1. Antibiotic prophylaxis
  2. Ureteric reimplantation
  3. Cystoscopy followed by subureteric injection of Teflon
  4. Bilateral ureterostomies
Ref: Ghai 8/e p462; 7/e p458; Nelson 17/e p1793
43. Vesicoureteric reflux is more common in: (AIIMS May 04)
  1. Newborn females
  2. Older girls
  3. Older boys
  4. Only during pregnancy
Ref: Ghai 8/e p462; 7/e p457-458; Nelson 17/e p1790-1791
44. The most common underlying anomaly in a child with recurrent urinary tract infections is: (AIIMS Nov 03)
  1. Posterior urethral valves
  2. Vesicoureteric reflux
  3. Neurogenic bladder
  4. Renal calculi
Ref: Ghai 8/e p462; Nelson 17/e p1790–1791; Nelson 19/e p1834, 1836
45. One year old male child presented with poor urinary stream since birth. The investigation of choice for evaluation is: (AIIMS May 03)
  1. Voiding cystourethrography (VCUG)
  2. USG bladder
  3. Intravenous urography
  4. Uroflowmetry
Ref: Ghai 8/e p467; 7/e p457-458; Nelson 17/e p1802-1803; Nelson 19/e p1836
46. A boy suffering from acute pyelonephritis, most specific urinary finding will be: (AIIMS Nov 06)
  1. Leukocyte esterase test
  2. WBC casts
  3. Nitrite test
  4. Bacteria in gram stain
Ref: Ghai 8/e p473; Nelson 17/e p1787; Nelson 19/e p1829, 1830
47. Which of the following is the most appropriate method for obtaining a urine specimen for culture in an 8 month old girl: (AIIMS May 04)
  1. Suprapubic aspiration
  2. Indwelling catheter sample
  3. Clean catch void
  4. Urinary bag sample
Ref: Ghai 8/e p467; 7/e p456; Nelson 17/e p1786-1787; Nelson 19/e p1831-1832
70
48. An 8 year old boy during a routine check up is found to have E. coli 1,00,000 cc/ml on a urine culture. The urine specimen was obtained by mid-stream cleancatch void. The child is asymptomatic. Which is the most appropriate next step in the management: (AIIMS May 04)
  1. Treat as an acute episode of urinary tract infection
  2. No therapy
  3. Prophylactic antibiotics for 6 months
  4. Administer long-term urine alkalinizer
Ref: Ghai 8/e p463; 7/e p456; Nelson 17/e p1787; Harrison 16/e p1718; Nelson 19/e p1831
49. A five year old male child presents with complaints of fever and abdominal distension. He is having vomiting for the last five days. On examination there are 6-8 Pus Cell/hpf in urine. WBC count shows 78% neutrophils. What is the best line of management: (AIIMS Nov 00)
  1. Send urine for culture and sensitivity and wait for results.
  2. Send urine for culture and sensitivity and start IV antibiotics immediately
  3. Send urine for culture, do an USG and start chloroquine
  4. Radio nucleotide studies
Ref: Ghai 8/e p463; Nelson 19/e p1830, 1831
50. UTI in infant, true about: (PGI Dec 03)
  1. Common in female infants
  2. If two episodes of UTI in females of 7 years occur, then cystometric evaluation needed
  3. If two episodes of UTI in male of 5 year occur then cystometric evaluation needed
Ref: Ghai 8/e p463-465; Nelson 17/e p1785–1788; Nelson 19/e p1832, 1833
51. In Schwartz formula for calculation of creatinine clearance in a child, the constant depends on the following except: (AIIMS Nov 06)
  1. Age
  2. Method of estimation of creatinine
  3. Mass
  4. Severity of renal failure
Ref: Ghai 8/e p487, 493; Nelson 19/e p1819, 1823
52. An 8 day old breast fed baby presents with vomiting, poor feeding and loose stools. On examination the heart rate is 190/minute, blood pressure 50/30 mm Hg, respiratory rate 72 breaths/minute and capillary refill time of 4 seconds. Investigations show hemoglobin level of 15g/dl, Na 120 mEq/L, K+ 6.8 mEq/L, bicarbonate 15 mEq/L, urea 30 mg/dL and creatinine 0.6 mg/dL. The most likely diagnosis is: (AIIMS Nov 04)
  1. Congenital adrenal hyperplasia
  2. Acute tubular necrosis
  3. Congenital hypertrophic pyloric stenosis
  4. Renal tubular acidosis
Ref: Ghai 8/e p487; Nelson 19/e p1819
53. Renal causes of acute renal failure include: (PGI Dec 01)
  1. Minimal change disease
  2. Renal amyloidosis
  3. Pre-eclampsia
  4. Malignant hypertension
  5. Hemolytic uremic syndrome
Ref: Ghai 8/e p487; Nelson’s Essentials of Pediatrics 5/e, p760; Nelson 19/e p1819
54. Which one of the following statements is false with regard to Xanthogranulomatous pyelonephritis in children: (AI 03)
  1. Often affects those younger than 8 years of age
  2. It affects the kidney focally more frequently than diffusely
  3. Boys are affected more frequently
  4. Clinical presentation in children is same as in adults
Ref: Ghai 8/e p474; Internet reference
55. Malformations of the following organ system of the fetus are found to be most commonly associated with single umbilical artery: (SGPGI 05)
  1. Central nervous system
  2. Cardiovascular
  3. Genitourinary
  4. Skeletal
Ref: Ghai 8/e p465; Nelson 17/e p608
56. “Potter’s syndrome” is associated with: (MAHE 05)
  1. Renal anomalies
  2. Severe oligohydramnio’s
  3. Flattened nose
  4. All the above
Ref: Nelson 17/e p1783; Internet
57. Most common infection in’a child nephritic syndrome: (MAHE 07)
  1. Spontaneous bacterial peritonitis
  2. Pneumonia
  3. UTI
  4. Cellulitis
Ref: OP Ghai 8/e p477; 7/e p453
58. Straining and dribbling of urine in a male infant with recurrent urinary infection should lead to the suspicion of: (UPSC 07)
  1. Vesico-ureteric reflux
  2. Posterior urethral valve
  3. Pelvic ureteric junction obstruction
  4. Phimosis
Ref: OP Ghai 8/e p465; 7/e p467
59. A child presents with failure to thrive with frequent vomiting, diarrhea, hepatosplenomegaly and abdominal distension. CT shows adrenal calcification. Which of the following is probable the diagnosis? (AIIMS Nov 09)
  1. Adrenal hemorrhage
  2. Woman disease
  3. Pheochromocytoma
  4. Addison’s disease
Ref: Ghai 8/e p515; Current Diagnosis and Treatment “Pediatrics 19/e (2009)); Nelson 19/e p479, 1926
60. Child presented with decreased urination, blood urea-108 and creatinine-3.2, Ca-12.5 . Which of the following is least appropriate line of management?
  1. N/2 saline
  2. Injection furosemide
  3. Bisphosphonates
  4. Hemodialysis
Ref: Nelson, Textbook of Pediatrics, 18/e p2208-9; Ghai 8/e p487
71
61. Eight-year-old boy presented with BP 180/110, Hb 9.6 gm, urea 120, creatinine 5.3, urinalysis—RBC 1-2/ hpf, WBC ++, protein urea +, no previous history of medical illness:
  1. Postinfectious GN
  2. RPGN
  3. Accelerated HT with ARF
  4. Chronic interstitial nephritis with reflux
Ref: Ghai 8/e p474
62. PKU first line therapy based on:
  1. Replacing the deficient product
  2. Avoiding substrate for the enzyme
  3. Replacement of the enzyme
  4. Administration of the tetrahydrobiopterin
Ref: Ghai 8/e p655; Nelson. Textbook of Pediatrics, 18/e p530.56
63. A 3-month-old male child with normal genitilia presents to the emergency department with severe dehydration, hyperkalemia, and hyponatremia. Measurement of which among the following is most helpful?
  1. 17-hydroxy progesterone
  2. Renin
  3. Cortisol
  4. Aldosterone
Ref: Ghai 7/e p493; Nelson 17/e p1912; 18/e, 2360
64. Commonest nephrotic syndrome in child: (DNB 2005)
  1. Minimal change
  2. Chronic glomerulonephritis
  3. Hemolytic uremic syndrome
  4. Congenital
Ref: Ghai 8/e p720; Nelson paediatrics, 18/e p2175, 2176
65. Berger Nephropathy is due to mesangial deposition of: (DNB 2007)
  1. Fibrin and C3
  2. IgD and C3
  3. IgE and C3
  4. IgA and C3
Ref: Ghai 8/e p670; Nelson Pediatrics 18/e p2171-2174, Robbins pathology
66. Most common cause or Renal Artery Stenosis in children with raised ESR in India is: (DNB 2009)
  1. Takayasu Aortoarteritis
  2. Fibromedial hypertrophy
  3. Fibrointimal hyperplasia
  4. Polyarteritis Nodosa
Ref: Op Ghai Pediatrics 6/e p586, Nelson Pediatric
67. Berger nephropathy is due to mesangiail deposition of: (DNB 2010)
  1. Fibrin and C3
  2. IgD and C3
  3. IgE and C3
  4. IgA and C3
Ref: Ghai 8/e p376; Nelson Pediatrics 18/e p2171-2174, Robbins pathology
68. Aniridia is associated with: (DNB 2010)
  1. Hepatoblastoma
  2. Medulloblastoma
  3. Nephroblastoma
  4. Retinoblastoma
Ref: Nelson Pediatrics 18/e p2140-2143; Ghai 8/e p670
69. Which of the following bacteria is most commonly responsible for peritionitis in nephrotic syndrome: (MP PG 2008)
  1. Staphylococcus
  2. Pneumooccus
  3. Hemophillus influenza type b
  4. Salmonella
Ref: Ghai 8/e p477; Bailey and Love’s 25/e p999
70. In management of UTI (Urinary tract infection) a VCUIG (voiding cystourethrogram) is always indicated in: (MP PG 2008)
  1. Asymptomatic bacteriuria
  2. Any child with febrile UTI
  3. School age girl with single UTI
  4. All children 2-5 years of age
Ref: Ghai 8/e p483; Nelson’s 18/e p2227
71. Renal papillary necrosis occur with: (MP PG 2008)
  1. Pernicious anemia
  2. Sickel cell anemia
  3. Aplastic anemia
  4. Sideroplastic anemia
Ref: Ghai 8/e p344; Harrison’s 16/e p1825; Pediatric up 2011
72. A child has nephrotic syndrome presented with fever and sudden painful lump in the abdomen diagnosis is: (MP PG 2008)
  1. Spontaneous bacterial peritonitis
  2. Renal vein thrombosis
  3. Acute glomeruloephritis
  4. Acute renal failure
Ref: Ghai 8/e p477; 7/e p1815; 453
73. Commonest cause of nephrotic syndrome in children is: (Karnatka PG 2010)
  1. Minimal change disease
  2. Membranous GN
  3. IgA nephropathy
  4. Mesangioproliferative GN
Ref: OP Ghai 8/e p477; 7/e p451; Harrison’s, 17/e p1790
74. All of the following are predisposing Factors to Urinary Tract infections in children, except: (AP 2010)
  1. Vesicoureteric Reflux
  2. Uncircumscised males
  3. Spina bifida
  4. Antobiotic treatment
Ref: Ghai 8/e p483
75. An infant with severe dehydration secondary to diarrhea suddenly presents with proteins and blood in urine. The most probable diagnosis is: (Feb DP PGMEE 2009)
  1. Renal vein thrombosis
  2. Pyelonephritis
  3. Acute glomerulonephritis
  4. Lower nephrosis
Ref: Ghai 8/e p477
76. Wormian bones are seen in all except: (Kerala PG 08)
  1. Osteogrenesis imperfecta
  2. Pyknodysostosis
  3. Cretinism
  4. Renal osteodystrophy
Ref: OP Ghai 6/e p482; Ghai 8/e p75, 520
72
77. Which of the following is true about congenital nephrotic syndrome: (Kerala PG 09)
  1. Diuretic is primary treatment of choice for ascites
  2. Renal biopsy is mandatory
  3. Prophylactic antibiotic has rule ixn prevention of SBP
  4. Live vaccines are indicated
Ref: Ghai 8/e p482; Nelson 18/e p2195, Management of Sterioid Sensitive Nephrotic Syndrome Revised Guidelines IAP, IPNG
78. Daily iron requirement of a 10 year old boy is: (Kerala PG 10)
  1. 2 mg
  2. 4 mg
  3. 8 mg
  4. 10 mg
Ref: Ghai 8/e p334; Nelson 18/e p210
 
 
Ans.
1. a. Autosomal dominant
2. c. Unilateral renal...
3. d. Multicystic dysplastic...
4. d. Ureterocele
5. b. Mutation of the...
6. d. Nephronophthisis
7. d. Nephronophthisis
8. b. Nephronophthisis
9 a. No finding
10. b. NPHS 2
11. b. Cyclophosphamide
12. b. Cyclophosphamide
13. a, b and d
14. a and e
15. b, d and e
16. a. One year of age
17. a. Presence of more...
18. c. Bartter syndrome
19. c. Bartter syndrome
20. c and e
21. c. Nephrin
22. a. No IgG or C3...
23. c. Mesangial deposits...
24. a. Prednisolone...
25. a and d
26. a. Acute GN
27. a. IgA nephropathy
28. a. Administer...
29. b. IgA Nephropathy
30. b. IgA nephropathy
31. c. IgA
32. c. Mesangial deposits...
33. a, d and e
34. b. Posterior urethral...
35. c. ESWL
36. c. Reduced calcium...
37. d. ESWL
38. b. Vesicoureteric reflux
39. c. Vesicoureteral...
40. d. Prophylactic...
41. c. Posterior urethral...
42. a. Antibiotic prophylaxis
43. a. Newborn females
44. b. Vesicoureteric reflux
45. a. Voiding...
46. d. Bacteria in gram...
47. a. Suprapubic aspiration
48. a. Treat as an acute...
49. b. Send urine...
50. All of the above
51. d. Severity of renal...
52. b. Acute tubular...
53. e. Hemolytic uremic...
54. a. Often affects those...
55. c. Genitourinary
56. d. All the above
57. a. Spontaneous...
58. b. Posterior urethral...
59. b. Woman disease
60. c. Bisphosphonates
61. d. Chronic interstitial...
62. b. Avoiding substrate...
63. d. Aldosterone
64. a. Minimal change
65. d. IgA and C3
66. a. Takayasu Aortoarteritis
67. d. IgA and C3
68. c. Nephroblastoma
69. b. Pneumooccus
70. b. Any child with...
71. b. Sickel cell anemia
72. b. Renal vein...
73. a. Minimal change...
74. d. Antobiotic treatment
75. a. Renal vein...
76. d. Renal osteodystrophy
77. b. Renal biopsy is...
78. c. 8 mg
 
7311. DISORDERS OF ENDOCRINE SYSTEM
  • A. Precocious and Delayed Puberty
  • B. Thyroid Disorders
  • C. Pituitary Disorders
  • D. Congenital Adrenal Hyperplasia
  • E. Diabetes Mellitus
  • F. Miscellaneous
 
74DISORDERS OF ENDOCRINE SYSTEM (QUESTIONS)
 
A. PRECOCIOUS AND DELAYED PUBERTY
1. First sign of puberty in girls: (PGI Dec 99)
  1. Puberche
  2. Thelarche
  3. Growth spurt
  4. Menarche
Ref: Ghai 8/e p531; Nelson 18/e p61 and 17/e p54; Nelson 19/e p651
2. A 13-year-old boy has bilateral gynecomastia. His height is 148 cm, weight 58 kg; the sexual maturity rating is stage 2. The gynecomastia is most likely due to: (AIIMS Nov 04)
  1. Prolactinoma
  2. Testicular tumor
  3. Pubertal gynecomastia
  4. Chronic liver disease
Ref: Nelson 18/e p63-65 and 17/e p54-55, 1859, 1930-1931; Nelson 19/e p651
3. Infantile proportion in adult is seen in: (PGI Nov 07)
  1. Moroquid’s disease
  2. Achondroplasia
  3. Hypothyroidism
  4. Malnutrition
  5. Constitutional dwarfism
Ref: Ghai 8/e p512; 7/e p18
4. Precocious puberty is seen in: (PGI Dec 02)
  1. Hypothyroidism
  2. CNS irradiation
  3. MC cune-Albright syndrome
  4. Turner’s syndrome
  5. Congenital adrenal hypoplasia
Ref: Ghai 8/e p532-534; Nelson’s Essentials of Pediatrics 5/e, p801; Nelson 19/e p1886, 1887, 1892
5. Delayed puberty in children is associated with: (PGI June 00)
  1. Poliomyelitis
  2. Hypothyroidism
  3. Hypopituitarism
  4. Anorexia nervosa
Ref: Ghai 8/e p535; Nelson’s Essentials of Pediatrics 5/e, p796; 18/e p63-65; Nelson 19/e p649
6. Delayed puberty seen in: (PGI Dec 06)
  1. Chronic disease
  2. Hypothyroidism
  3. Turner’s syndrome
  4. Malabsorption syndrome
Ref: Ghai 8/e p535; Nelson’s Essentials of Pediatrics 5/e, p796; 18/e p63-65
7. 6-years-old girl ith vaginal spotting. Diagnosis is: (DNB 2001)
  1. Ovarian cancer
  2. Foreign body
  3. Sexual abuse
  4. PID
Ref: Nelson 18/e p2279
8. Precocious puberty is seen in: (DNB 2007)
  1. Hyperthyroidism
  2. Addison’s disease
  3. McCune Albright syndrome
  4. Neuroblastoma
Ref: Ghai 8/e p542; Nelson 18/e p2379-2382
9. Precocious puberty is seen in: (DNB 2010)
  1. Hyperthyroidism
  2. Addison’s disease
  3. McCune Albright syndrome
  4. Neuroblastoma
Ref: Ghai 8/e p594; Campbell Text book of Peds, 10/e p1028, 1029
10. Precocious puberty is seen in: (Raj PGI 2009)
  1. Hypothyroidism
  2. CNS irradiation
  3. Me cune-albright synd
  4. All
Ref: Ghai 8/e p531
11. Precocious puberty in girls is suggestive when pubertal onset is before: (J & K PG 2011)
  1. 7 years
  2. 8 years
  3. 9 years
  4. 10 years
Ref: OP Ghai 8/e p531
 
B. THYROID DISORDERS
12. Blood specimen for neonatal thyroid screening is obtained on: (AI 05; AIIMS May 03)
  1. Cord blood
  2. 24 hours after birth
  3. 48 hours after birth
  4. 72 hours after birth
Ref: Ghai 8/e p518; Harrison 16/e p2109: Nelson 19/e p1899
13. In neonatal screening program for detection of congenital hypothyroidism, the ideal place and time to collect the blood sample for TSH estimation is: (AIIMS May 03)
  1. Cord blood at time of birth
  2. Heal pad blood at the time of birth
  3. Heal pad blood on 4 day of birth
  4. Peripheral venous blood on 28 day
Ref: Ghai 8/e p516; Harrison 16/e p2109; Nelson 19/e p1899
14. Infantile proportion in adult is seen in: (PGI Dec 01)
  1. Morquoi’s disease
  2. Achondroplasia
  3. Hypothyroidism
  4. Malnutrition
  5. Constitutional dwarfism
Ref: Ghai 8/e p512
75
15. 8 years old child presents with lethargy multiple epiphyseal breaks, wormian bones with growth retardation and mental retardation Diagnosis is? (AIIMS Nov 01)
  1. Rickets
  2. Hypothyroidism
  3. Scurvy
  4. Hypoparathyroidism
Ref: Ghai 8/e p517; 7/e p480-483; Nelson 18/e p2319-2327 and 17/e p1876; Nelson 19/e p1896, 1897
16. Common presentations of Juvenile Hypothyroidism: (PGI June 06)
  1. Growth retardation
  2. Mental retardation within 2 years
  3. Delayed puberty
  4. Umbilical Hernia
  5. Moist skin
Ref: Ghai 8/e p517; 7/e p480-483; Nelson 18/e p2319-2327 and 17/e p1876; Nelson 19/e p1898
17. True about Pendred syndrome: (PGI Dec 04)
  1. Diffuse colloid goiter
  2. Nodular goiter
  3. Mental retardation
  4. BIL sensory neural deafness
  5. Normal cochle
Ref: Harrison 16/e p181, 2104; Ghai 8/e p516; http://www.medicinenet.com/pendred_syndrome/article.htm; Nelson 19/e p1896, 1906
18. Pendred’s syndrome is: (PGI June 02)
  1. Consistently associated with deafness
  2. Hypothyroidism is seen
  3. Mutation in connection coding gene
  4. Mutation in chromosome 21 causing receptor defect
Ref: Ghai 8/e p516; 7/e p480-483; Harrison 16/e p181, 2104; http://www.medicinenet.com/pendred_syndrome/article.htm
19. Which of the following is true in cretinism: (PGI Dec 01)
  1. Goiter present at birth
  2. Can be diagnosed by serum T4 levels
  3. Prolonged physiological jaundice present
  4. Common in iodine deficiency endemic areas
  5. Delayed skeletal development
Ref: Ghai 8/ep517; 7/e p480-483; Harrison 16/e p181, 2104; http://www.medicinenet.com/pendred_syndrome/article.htm
20. Clinical features of “hypothyroidism” in a newborn are all except: (MAHE 05)
  1. Sluggishness +++
  2. Large tongue
  3. Large posterior fontanel
  4. Mental retardation
Ref: Ghai 8/e p517; 7/e p481
21. Features of hypothyroidism in infancy include the following except: (UPSC 06)
  1. Premature closure of posterior fontanel
  2. Coarse facies
  3. Umbilical hernia
  4. Constipation
Ref: Ghai 8/e p517; 7/e p481
22. Blood specimen for neonatal thyroid screening is obtained on: (DNB 2006, 2007)
  1. Cord blood
  2. 24 hours after birth
  3. 48 hours after birth
  4. 72 hours after birth
Ref: OP Ghai 8/e p546
23. Epiphyseal dysgenesis is a pathognomonic feature of: (DNB 2007) (DNB 2010)
  1. Hypoparathyroidism
  2. Hyperparathyroidism
  3. Hypothyroidism
  4. Hyperthyroidism
Ref: Ghai 8/e p524; Nelson Pediatrics 18/e p2319
24. Most common cancer of thyroid in children is: (DNB 2011)
  1. Papillary
  2. Follicular
  3. Medullary
  4. Anaplastic
Ref: Esssentials of Pediatrics, Nelson 4/e p739
25. Hypothyroidism in infancy is characterized by all except: (UP PG 09)
  1. Constipation
  2. Coarse facies
  3. Wide open cranial sutures
  4. Hyperthermia
Ref: Ghai 8/e p520; 7/e p481
26. The features of neonatal hyperthyrodidism include all except: (DP PGMEE 2009)
  1. Triangular facies with craniosynostosis
  2. Congestive cardian failure
  3. Advanced osseous maturation
  4. Goiter is rare
Ref: Ghai 8/e p75, 520; Nelson 18/e p2319-2322
27. Clinical features of “hypothyroidism’ in an new born are all, except: (Feb DP PGMEE 2009)
  1. Sluggishness +++
  2. Large tongue
  3. Large posterior fontanel
  4. Mental retardation
Ref: Ghai 8/e p516
28. Human fetus secretes thyroxine from which gestation age? (J & K PG 2011)
  1. 8 weeks
  2. 12 weeks
  3. 24 weeks
  4. 30 weeks
Ref: Ghai 8/e p516
 
C. PITUITARY DISORDERS
29. Which of the following is true regarding cretinism: (AI 01)
  1. Short limbs compared to trunk
  2. Proportionate shortening
  3. Short limbs and short stature
  4. Short limbs and long stature
Ref: Ghai 8/e p512; 7/e p481; Nelson 18/e p1885; Nelson 19/e p1896
30. Manifestations of endemic cretinism include: (AI 01)
  1. Deafness and facial nerve involvement
  2. Blindness and hypothyroidism
  3. Strabismus and spastic diplegia
  4. Multinodular goitre and mental retardation
Ref: Ghai 8/e p519; 7/e p475; Nelson 18/e p1885
76
31. A 9-year-old boy presents with growth retardation and propensity to hypoglycemia. Physical examination reveals short stature, micropenis, increased fat and high-pitched voice. The skeletal survey reveals bone age of 5 years. Which of the following is most appropriate diagnosis: (AIIMS Nov 04)
  1. Malabsorption
  2. Growth hormone deficiency
  3. Adrenal tumor
  4. Thyroxin deficiency
Ref: Ghai 8/e p511; 7/e p474-475
32. ‘Weak giants’ are produced by: (AIIMS May 04)
  1. Thyroid adenomas
  2. Thyroid carcinomas
  3. Parathyroid adenomas
  4. Pituitary adenomas
Ref: Harrison 16/e p2090; Nelson 18/e p2303 and 17/e p1859; Nelson 19/e p1878
 
D. CONGENITAL ADRENAL HYPERPLASIA
33. Which one of the following drugs is used for fetal therapy of congenital adrenal hyperplasia? (AI 05)
  1. Hydrocortisone
  2. Prednisolone
  3. Flurocortisone
  4. Dexamethasone
Ref: Ghai 7/e p493, 6/e p490; Nelson 18/e, 2360-2367 and 17/e p1912; Nelson 19/e p1934
34. A 1-month-old baby presents with frequent vomiting and failure to thrive. There are features of moderate dehydration. Blood sodium is 122 mEq/L and potassium is 6.1 mEq/L. The most likely diagnosis is: (AI 03, 02)
  1. Gitelman syndrome
  2. Bartter syndrome
  3. 21-hydroxylase deficiency
  4. 11-b hydroxylase deficiency
Ref: Ghai 8/e p527; 7/e p493; Nelson 17/e p1912; 18/e, 2360; Nelson 19/e p1930
35. A 10-day-old male pseudohermaphrodite child with 46 XY karyotype presents with BP of 110/80 mmHg. Most likely enzyme deficiency is: (AI 01)
  1. 21 hydroxylase
  2. 17 hydroxylase
  3. 11 hydroxylase
  4. 3-beta hydroxylase
Ref: Ghai 7/e p489-490, 6/e p487-489; 18/e p2360 and Nelson 17/e p1911, 280; Nelson 19/e p1936
36. Congenital adrenal hyperplasis is associated with: (PGI Dec 00)
  1. Hypoglycemia
  2. Hyponatremia
  3. Hypokalemia
  4. Hyperkalemia
Ref: Nelson 18/e p2360, 2367 and 17/e p1913; Nelson 19/e p193
37. Male pseudothermaphroditism is seen in: (PGI Dec 07)
  1. 5 reductase deficiency
  2. 21 hydroxylase deficiency
  3. 17 hydroxylase deficiency
  4. Gonadal dysgenesis
Ref: Ghai 6/e p468; Nelson 19/e p1932
38. A 8-day-old breast fed baby presents with vomiting, poor feeding and loose stooled. On examination the heart rate is 190/minute, blood pressure 50/30 mm Hg, respiratory rate 72 breaths/minute and capillary refill time of 4 seconds. Investigations show hemoglobin level of 15g/DL. Na-12 mEq/L, K-6.8 mEq/L, Cl-81 mEq/L, bicarbonate 15 mEq/L, Urea 30 mg/dL, creatinine 0.6 mg/dL. The most likely diagnosis is: (AIIMS Nov 03)
  1. Congenital adrenal hyperplasia
  2. Acute tubular necrosis
  3. Congenital hypertrophic pyloric stenosis
  4. Galactosemia
Ref: Nelson 18/e p2362; Nelson 19/e p1930
39. A 5-year-old girl presents with hypertension and virilization. There is also finding of hypokalemia what is the diagnosis: (AIIMS May 07)
  1. 21-hydroxylase deficiency
  2. 3-hydroxy seroid deficiency
  3. 11-hydroxylase deficiency
  4. Cohn’s disease
Ref: See previous explanations; Nelson 19/e p1935, 1936
40. A five-years-old boy has precocious puberty BP 130/80. Estimation of which of the following will help in diagnosis? (AI 09)
  1. 17-Hydroxyprogesterone
  2. 11-Deoxycortisol
  3. Aldosterone
  4. DOCA
Ref: Ghai 7/e p489; Nelson 18/e, Ch 563, 571, p2360, 2367 Nelson 19/e p1937
41. Adrenal hyperplasia due to 21 hydroxylase deficiency is treated with low dose: (Kerala 04)
  1. Androgen
  2. Estrogen
  3. Cortisone
  4. Anti – androgen
Ref: Ghai 8/e p514; 7/e p492; Nelson 17/e p1912
42. A 6-week-old male infant was brought in a state of dehydration and shock. Examination revealed hyper pigmentation over the body with normal external genitalia. Blood tests revealed hypoglycemia, Na - 124 mEq/L and K - 7 mEq/L. What is the probable diagnosis?
  1. Congenital adrenal hyperplasia
  2. Adrenal hemorrhage and shock
  3. Acute gastroenteritis with dehydration
  4. Septicemia with ARF
Ref: Ghai 8/e p517; Nelson 18/e p2361-64
43. A 6 week old male infant was brought in a state of dehydration and shock. Examination revealed hyper pigmentation over the body with normal external genitalia. Blood tests revealed hypoglycemia, Na – 124 mEq/L and K - 7 mEq/L. Most probable diagnosis is: (DNB 2009)
  1. Congenital adrenal hyperplasia
  2. Adrenal haemorrhage and shock
  3. Acute gastroenteritis with dehydration
  4. Septicemia with ARF
(Ref: Ghai 8/e p638; Nelson 18/e p2361-64
77
44. Most common cause of CAH is deficency of hydroxylase: (DNB 2010)
  1. 21 Alpha
  2. 11 beta
  3. 3 beta
  4. 3 alpha
Ref: Ghai 8/e p594; Nelson 18/e p2361-64
45. A female child with Hypertension, Hyperpigmentation, Virilization has deficiency of: (DNB 2011)
  1. 11 beta hydroxylase
  2. 17 alpha hydroxylase
  3. 3 beta hydroxylase
  4. 21 alpha hydroxylase
Ref: Ghai 8/e p498; Nelson Ped, 18/e p2366
46. Most common type of Congenital adrenal hyperplasia (CAH)? (MHPGM-CET 2010)
  1. 21-hydroxylase deficiency
  2. 11-hydroxylase deficiency
  3. 3-hydroxylase deficiency
  4. 17-hydroxylase deficiency
Ref: OP Ghai 8/e p526; 7/e p492; Harrison’s 17/e p2267
47. In congenital adrenal hyperplasia most common deficiency: (Kerala PGI 09)
  1. 21 – alpha hydroxylase deficiency
  2. 11 – beta hydroxylase deficiency
  3. 17 – hydroxylase deficiency
  4. 3 – beta hydroxylase deficiency
Ref: Ghai 8/e p526; Nelson 18/e p2360
 
E. DIABETES MELLITUS
48. In India the commonest cause of Juvenile Onset of Diabetes mellitus: (AIIMS May 01)
  1. IDDM
  2. Fibrocalcific pancreaticopathy
  3. Mody
  4. Gall stones
Ref: Nelson 18/e p2404 and 17/e p1947; Nelson 19/e p1970
49. In children with type I DM when is ophthalmologic evaluation indicated: (PGI June 06)
  1. At the time of diagnosis
  2. After 1 year
  3. After 2 years
  4. After 5 years
  5. After 10 years
Ref: Ghai 8/e p666; API Textbook of Medicine, 7/e p1136; Nelson 19/e p1970, 1971
50. Which one of the following medical disorders leads to delayed fetal lung maturity: (UPSC07)
  1. Heart disease
  2. Diabetes
  3. Thalassemia minor
  4. Epilepsy
Ref: Ghai 8/e p541; Nelson 17/e, 575
 
F. MISCELLANEOUS
51. A 10-year-old boy has a fracture of femur. Biochemical evaluation revealed Hb 11.5 gm/dl and ESR 18 mm 1st hour. Serum calcium 12.8 mg/dL, serum phosphorus 2.3 mg/dL, alkaline phosphate 28 Ka units and blood urea32 mg/dL. Which of the following is the most probable diagnosis in his case: (AI 04)
  1. Nutritional rickets
  2. Renal rickets
  3. Hyperparathyroidism
  4. Skeletal dysplasia
Ref: Nelson 17/e p2345; Ghai 8/e p518
52. Which of the following statements about 21 alpha hydroxylase deficiency is false: (AI 08)
  1. Most common cause of Congenital Adrenal Hyperplasia (CAH) in children
  2. Affected females present with a ambiguous genitalia
  3. Affected males present with precarious puberty
  4. Hypokalemic alkalosis is seen
Ref: Ghai 8/e p526; 7/e p493; Nelson 18/e p2360; Nelson 19/e p1930, 1961
53. 8 year old boy with undescended testis, your concern to ask for operation is due to: (PGI June 01)
  1. Cosmetic reasons
  2. Infertility
  3. Risk of malignancy
  4. Impotence
Ref: Ghai 8/e p540; Nelson’s Essentials of Pediatrics 5/e, p769; 18/e p2144; Nelson 19/e p1858-1860
54. Orchidopex is done in cases of undescended testes at the age of: (AIIMS Nov 06)
  1. Neonate
  2. 1-2 years
  3. 5 years
  4. Puberty
Ref: Ghai 8/e p540; Nelson 18/e p2261; Nelson 19/e p1859
55. A boy with undescended testis, your concern to ask for operation is due to: (PGI 01)
  1. Cosmetic reasons
  2. Infertility
  3. Risk of malignancy
  4. Impotence
Ref: Ghai 8/e p540; Nelson 18/e p2260; Nelson 19/e p1859
56. Surgical treatment of cryptorchidism of right testis with normally descended left testis can be done: (All India 99)
  1. Immediately
  2. At 2 years of age
  3. At 4 years before going to school
  4. At puberty
  5. None
Ref: Nelson 18/e p2260; Nelson 19/e p1860
57. Unilateral undescended testes is ideally operated around: (AI 04)
  1. 2 months of age
  2. 6 months of age
  3. 12 months of age
  4. 24 months of age
Ref: Nelson 18/e p2261; Nelson 19/e p1859, 1860
78
58. A 10-years-old boy has a fracture of femur. Biochemical evaluation revealed Hb 11.5 gm/dL and ESR 18 mm first hour. Serum calcium 12.8 mg/dL, serum phosphorus 2.3 mg/dL, alkaline phosphate 28 KA units and blood urea 32 mg/dL. Which of the following is the most probable diagnosis in his case? (AI 04)
  1. Nutritional rickets
  2. Renal rickets
  3. Hyperparathyroidism
  4. Skeletal dysplasia
Ref: Ghai 7/e p486-487, 6/e p129-130; Nelson 18/e p2345-2348; Nelson 19/e p1922
59. A 10-years-old boy is having polyuria, polydypsia, laboratory data showed (in mEq/lit) Na-154, K-4.5, HCO3-22, Serum osmolality - 295, Blood urea-50, Urine specific gravity -1.005. The likely diagnosis is: (AIIMS May 02)
  1. Diabetes insipidus
  2. Renal tubular acidosis
  3. Bartter’s syndrome
  4. Recurrent UTI
Ref: Ghai 8/e p514; 7/e p477; API 6/e p994; Neslon 18/e p2299-2301
60. ACTH secretion is highest during: (PGI Dec 99)
  1. Noon
  2. Evening
  3. Morning
  4. Night
Ref: Ghai 7/e p490, 6/e p497; Nelson 18/e p2291, 2292 and 17/e p1917; Nelson 19/e p1940
61. Which one of the following is not a feature of Phenylketonuria? (UPSC 04)
  1. Severe mental retardation
  2. Reduced tendon reflexes
  3. Enamel hypoplasia
  4. Vomiting in early infancy
Ref: Ghai 8/e p653; Nelson 17/e p399, 400
62. Consider the following diseases: (UPSC 07)
  1. Addison’s disease
  2. Hypervitaminosis A
  3. Obesity
  4. Hyperparathyroidism
Which of the above are associated with pseudotumor cerebri
  1. 1, 2 and 3 only
  2. 1 and 3 only
  3. 2 and 4 only
  4. 1, 2, 3 and 4
Ref: O.P. Ghai 8/e p549; Harrison 16/e, 1698, 1701; Nelson 17/e, 2048
63. With reference to classical phenylketonuria, consider the following statements: (UPSC 07)
  1. Mental retardation is usually mild
  2. Vomiting is usually severe
  3. Hypotonia and reduced tendon reflexes are present
  4. Convulsions may occur in 25% of cases
Which of the above are the features of classical phenylketonuria: (UPSC 07)
  1. 1, 2 and 3 only
  2. 1 and 3 only
  3. 1 and 4 only
  4. 1, 2, 3 and 4
Ref: OP Ghai 8/e p655; Nelson 17/e, 399-400
64. A child of phenyl ketonuric mother may develop: (AIIMS June 99)
  1. Microcephaly, mental retardation, congenital heart disease
  2. Mental retardation, cataract, congenital heart disease
  3. Hydrocephalus, cataract
  4. Microcephaly, cataract, renal dysplasia
Ref: Ghai 8/e p650-652; Nelson’s Essentials of Pediatrics 5/e p256
65. A child has microcephaly, blue eyes, fair skin, and mental retardation, ferric chloride test is positive. What is the likely diagnosis: (AI 07)
  1. Phenylketonuria (PKU)
  2. Homocystinuria
  3. Tyrosinosis
  4. Alkaptonuria
Ref: Ghai 8/e p650; 7/e p628, Nelson 18/e p529; Ghai 6/e p609
66. Which one of the following is not a feature of Phenylketonuria? (UPSC 04)
  1. Severe mental retradation
  2. Reduced tendon reflexes
  3. Enamel hypoplasia
  4. Vomiting in early infancy
Ref: Ghai 8/e p654; 7/e p628; Nelson 17/e p399, 400
67. A 15-day-old baby came with history of seizures. Blood tests revealed Ca 5 mg/dl, PO4 – – – 9 mg/dl, PTH 30pg/ml (n = 10-60). What is the most probable diagnosis?
  1. Pseudohypoparathyroidism
  2. Vitamin D deficiency
  3. Hyperparathyroidism
  4. HIE
Ref: Ghai 8/e p517; Manual of neonatal care, Cloherty p550, Nelson 18/e p2343
68. Earliest indication of sexual maturation in a girl is: (DNB 2007)
  1. Menarche
  2. Pubarche
  3. Menopause
  4. Maturation of breasts
Ref: Ghai 8/e p324; Nelson 18/e p2374, 2379, 2389
69. Not a cause of hirusitism is: (DNB 2010)
  1. Testicular feminization
  2. Cushings syndrome
  3. CAH
  4. PCOD
Ref: OP Ghai 8/e p610; Nelson 18/e p2375, 2379
70. Earliest indication of sexual maturation in a girl is: (DNB 2010)
  1. Menarche
  2. Pubarche
  3. Thelarche
  4. Maturation of breasts
Ref: Ghai 8/e p519; Nelson Pediatrics 18/e p2374, 2390, 16/e p53
 
Ans.
1. b. Thelarche
2. c. Pubertal gynecomastia
3. a, b and c
4. a, b and c
5. b, c and d
6. All of the above
7. b. Foreign body
8. c. McCune Albright...
9. c. McCune Albright...
10. d. All
11. b. 8 years
12. d > a
13. c. Heal pad blood...
14. a, b and c
15. b. Hypothyroidism
16. a, b and d
17. a, c and d
18. a, b and c
19. c, d and e
20. d. Mental retardation
21. a. Premature closure...
22. d. 72 hours after birth
23. c. Hypothyroidism
24. a. Papillary
25. d. Hyperthermia
26. d. Goiter is rare
27. d. Mental retardation
28. b. 12 weeks
29. a and c
30. c. Strabismus...
31. b. Growth hormone...
32. d. Pituitary adenomas
33. d. Dexamethasone
34. c. 21 -hydroxylase...
35. b. 17 hydroxylase
36. a, b and d
37. a, c and d
38. a. Congenital adrenal...
39. c. 11-hydroxylase...
40. b. 11-Deoxycortisol
41. c. Cortisone
42. a. Congenital adrenal...
43. a. Congenital adrenal...
44. a. 21 Alpha
45. a. 11 beta hydroxylase
46. a. 21- hydroxylase...
47. a. 21 – alpha hydroxylase...
48. b. Fibrocalcific ...
49. d. After 5 years
50. b. Diabetes
51. c. Hyperparathyroidism
52. d. Hypokalemic...
53. b and c
54. b. 1-2 years
55. b and c
56. e. None
57. b. 6 months...
58. c. Hyperparathyroidism
59. a. Diabetes insipidus
60. c. Morning
61. b. Reduced...
62. d. 1, 2, 3 and 4
63. c. 1 and 4 only
64. a. Microcephaly...
65. a. Phenylketonuria (PKU)
66. b. Reduced tendon...
67. a. Pseudohypoparathyroidism
68. d. Maturation of...
69. a. Testicular feminization
70. c. Thelarche
 
7912. DISORDERS OF NEUROMUSCULAR SYSTEM
 
80DISORDERS OF NEUROMUSCULAR SYSTEM (QUESTIONS)
1 A baby presents with tetany. First thing to be done is administration of: (AIIMS June 00)
  1. Diazepam
  2. Vitamin D
  3. Calcium gluconate
  4. Calcitonin
Ref: Nelson 18/e p2864 and 17/e p1892; Ghai 8/e p118
2. With reference to Polyarticular juvenile rheumatoid arthritis, consider the following statements: (UPSC 07)
  1. It is more common in girls
  2. Five or more joints are affected within the first six months of onset
  3. Uveitis occurs in 95% patients
  4. Rheumatoid factor may be negative
Ref: Ghai 8/e p589
Which of the statements given above are correct ?
  1. 1 and 4 only
  2. 1, 2 and 4
  3. 2, 3 and 4
  4. 2 and 3 only
Ref: O.P. Ghai 7/e p599, 6/e p52; IAP textbook of Pediatrics 3/e p868
3. A 2-years-old child is brought to emergency at 3 AM with fever, barking cough and stridor only while crying. The child was able to drink normally. On examination respirator rate is 36/min and temperature is 39.6°c. What will be your next step: (AIIMS Nov 08)
  1. Racemic epinephrine nebulization
  2. High dose dexamethasone injection
  3. Nasal wash for influenza or RS V
  4. Antibiotics and blood culture
Ref: Nelson 18/e p1765; Ghai 8/e p375; 7/e p339, 3406/e p339, 340; Nelson 19/e p1446
4. All are seen in metachromatic leukodystrophy except: (TN 03)
  1. Mental retardation
  2. Optic atrophy
  3. Decerebrate posture
  4. Exaggerated tendon reflexes
Ref: Ghai 8/e p647; 7/e p561; Nelson 17/e p2032
5. Macrocephaly is seen in which of the following syndromes? (AIIMS May 08)
  1. Metachromatic leukodjystrophy
  2. Adrenoleukodystrophy
  3. Canavan’s disease
  4. Krabbe’s disease
Ref: Internet reference; Nelson 19/e p455, 456
6. Deep white matter lesion with bilateral deep bright thalamic appearance is suggestive of: (AIIMS Nov 07)
  1. Alexander disease
  2. Canavan’s diseases
  3. Krabbe’s disease
  4. Metachromatic leukodystrophyz
Ref: Ghai 7/e p637; Nelson 18/e p2502; Nelson 19/e p48
7. All are seen in metahromatic leukodystrophy except: (TN 03)
  1. Mental retardation
  2. Optic atrophy
  3. Decrebrate posture
  4. Exaggerated tendon reflexes
Ref: OP Ghai 8/e p724; 7/e p7/e, p639; Nelson 17/e p2032
8. Ataxia telangiectasia is characterized by all of the following except: (AI 04)
  1. Chronic sinopulmonary disease
  2. Decreased level of a-fetoprotein
  3. Chromosomal breakage
  4. IgA deficiency
Ref: Ghai 8/e p580, 546; Nelson 18/e p891, 2489, 2674 and 17/e p699, 2020
9. True regarding febrile convulsion: (PGI Dec 00)
  1. Carbamazepine is good drug to treat it
  2. Patients with family h/o FC have increased incidence of recurrence
  3. Long-term neurological deficits are common
  4. Usually last for short while
Ref: Ghai 8/e p556; Nelson’s Essentials of Pediatrics 5/e, p838; 18/e p2457; Nelson 19/e p2013, 2017
10. 4-years-old male child had febrile seizures, best prophylaxis: (PGI June 00)
  1. Paracetamol 6 hourly
  2. Paracetamol and diazepam
  3. Diazepam
  4. Phenobarbitone
Ref: Ghai 8/e p556, 557; 7/e p528; Nelson 18/e p2457 and 17/e p1994-1995 Nelson 19/e p2018
11. Which of the following is NOT associated with increase in the risk of seizures in future in a child with febrile seizures? (AI 2010)
  1. Developmental delay
  2. Early age of onset
  3. Complex partial seizures
  4. Family history positive
Ref: OP Ghai 8/e p556; 7/e p528; Nelson Paediatrics IT11’/1994; Nelson 19/e p2017
12. Which one of the following is the characteristic feature of juvenile myoclortic epilepsy? (AIIMS May 06)
  1. Myoclonic seizures frequently occur in morning
  2. Complete remission is common
  3. Response to anticonvulsants is poor
  4. Associated absence seizures are present in majority of patients
Ref: Ghai 8/e p557; Nelson 18/e p2457-2475 and 17/e p1998; Harrison 16/e p2359; Nelson 19/e p2015, 2016
13. True about juvenile myoclonic epilepsy: (PGI June 08)
  1. DOC is Sodium Valproate
  2. Mental retardation
  3. 81Seizure can develop
  4. Neurological examination abnormal
  5. Life long treatment needed
Ref: Ghai 8/e p557; Nelson 18/e p2457-2475 and 17/e p1998; Nelson 19/e p2015
14. Most common cause of seizure in newborn is: (AIIMS Nov 08)
  1. Hypoxia induced ischemic encephalopathy
  2. Hypocalcemia
  3. Metabolic abnormality
  4. Sepsis
Ref: Ghai 8/ep166; Nelson 18/e p718-720; Nelson 19/e p2033, 2034
15. Seizure and Jitteriness can be differentiated by all except: (AI 07)
  1. Gaze
  2. Autonomic disturbances
  3. Sensitivity to stimulus
  4. Frequency of movements
Ref: Ghai 8/e p561; Nelson 18/e p718-720; Nelson 19/e p2035
16. Commonest type of seizure in newborn: (AIIMS June 08)
  1. Clonic
  2. Tonic
  3. Subtle
  4. Myoclonic
Ref: Ghai 8/e p558; 7/e p142; Care of the New born, Meharban Singh, 6/e p329; Nelson 19/e p2034
17. Commonest type of seizure in newborn: (AI 09)
  1. Atonic
  2. Tonic
  3. Subtle
  4. Myoclonic
Ref: Ghai, 8/e p558; Nelson 18/e p1340; Nelson 19/e p2034
18. All are features of absence seizures except:
  1. Usually seen in childhood [AI 04]
  2. 3-Hz spike wave in EEG
  3. Postictal confusion
  4. Precipitation by hyperventilation
Ref: Ghai 8/e p557; 7/e p530; Nelson 18/e p2032 and 17/e p1997; Nelson 19/e p2023
19. A school going boy was noted with vacant stare several times a day. There was no history of fever, seizures and neurological deterioration. What is the diagnosis? (AI 2010)
  1. Atonic seizures
  2. Absence seizures
  3. Myoclonic seizures
  4. School phobia
Ref: Ghai 8/e p557; 7/e p530; Nelson 19/e p2023
20. A 6 years old child with acute onset of fever of 104° F developed febrile seizures and was treated. To avoid future recurrence of seizure attacks what should be given: (AIIMS May 01)
  1. Paracetamol 400 mg + Phenobarbitone daily
  2. Oral Diazepam 6 hourly
  3. Paracetamol 400 mg 6 hourly
  4. IV diazepam infusion over 12 hours
Ref: Ghai 8/e p560; 7/e p528; Nelson 18/e p2457-2475 and 17/e p1994-1995; Nelson 19/e p2018
21. The commonest organism responsible for meningitis in neouate child is: (AI 07, PGI June 2007)
  1. Listeria
  2. E.coli
  3. H. Influenza
  4. Type B streptococcus
Ref: Ghai 8/e p563; Nelson 18/e p2514, 2517 and 17/e p2038-2039; Nelson 19/e p2087
22. Bacterial meningitis in children (2 months-12 years of age) is usually due to the following organisms except: (AI 04)
  1. Streptococcus pneumoniae
  2. Neisseria meningitidis
  3. Hemophilus influenzae type B
  4. Listeria monocytogenes
Ref: Ghai 8/e p563; Nelson 18/e p1157-1159 and 17/e p2038; Nelson 19/e p2088
23. Hemophilus influenzae has been isolated from the CSF of a two year old boy suffering from meningitis. The strain is beta-lactamase producing and resistant to chloramphenicol. The most appropriate antimicrobial in such a situation is: (AI 04)
  1. Trimethoprim-sulfamethoxazole combination
  2. Ciprofloxacin
  3. Third-generation cephalosporin
  4. Vancomycin
Ref: Ghai 8/e p565; Nelson 18/e p1119-1120 and 17/e p2042; Nelson 19/e p209
24. Most common cause of pyogenic meningitis in 6 month to 2 years of age is: (AI 98)
  1. Staphylococcus aureus
  2. Pneumococcus
  3. Streptococcus pneumonia
  4. H. influeuenzae
Ref: Nelson 18/e p2514-2517 and 17/e p2038; Ghai 8/e p563
25. Neurological complications of meningitis include all of the following except: (PGI June 08)
  1. Seizures
  2. Increased intracranial pressure
  3. Cerebral hamartoma
  4. Subdural effusions
  5. Brain abscess
Ref: Ghai 8/e p563; Nelson 19/e p2089
26. CSF examination of a patient shows high protein, markedly low sugar, low chloride and increased neutrophils. The diagnosis is: (AIIMS Nov 99)
  1. Viral meningitis
  2. Meningococcal meningitis
  3. Tuberculous meningitis
  4. Fungal meningitis
Ref: Ghai 8/e p563, 564; Nelson 18/e p2513-2531 and 17/e p2039; Nelson 19/e p2088
27. Aseptic meningitis is caused by all except: (AIIMS June 98)
  1. Mumps
  2. Polio
  3. Measles
  4. Coxsackie virus
Ref: Ghai 8/e p562; Nelson’s Essentials of Pediatrics 5/e, p481-482; 18/e p2513-2531
82
28. 5-year-old comes to pediatric ICU with RR 42, BP 110/80 and E1V1M3 on Glasgow coma scale, next step (tachycardia, unconscious with 2 days h/o fever): (PGI Nov 2009)
  1. Intubatc and ventilate
  2. IV Manitol
  3. Give IV dopamine 10µg/kg
  4. Give both furoseimde and dopamine
  5. Start antibiotics and watch CT Scan
Ref: Ghai 8/e p708
29. A 25 years old woman had premature rupture of membranes and delivered a male child who became lethargic and apneic on the 1st day of birth and went into shock. The mother had a previous history of abortion 1 year back. On vaginal swab culture growth of b-hemolytic colonies on blood agar was found. On staining these were found to be gram-positive cocci. Which of the following is the most likely etiological agent? (AI 04)
  1. Streptococcus pyogenes
  2. Streptococcus agalactiae
  3. Peptostreptococci
  4. Enterococcus faecium
Ref: Ghai 8/e p162; Nelson 18/e p1145-1149 and 17/e p627
30. Most common cause of meningitis in children between 6 months to 2 years of age is: (AI 01, AI 98)
  1. Pneumococcus
  2. Staphylococcus
  3. H influenza
  4. E. coli
Ref: Ghai 8/e p162; Nelson 18/e p2541-2517 and 17/e p2038; Ghai 6/e p517; Nelson 19/e p2087
31. A newborn of 7 days old presented with meningitis. Most common cause: (PGI Nov 08)
  1. E. Coli
  2. Streptococcal pneumonia
  3. N. meningitides
  4. Streptococcal agalactiae
  5. H.influenzae
Ref: Ghai 8/e p162
32. Neonatal meningitis is caused by: (PGI Nov 07)
  1. Group ‘A’ streptococcus
  2. Group ‘B’ streptococcus
  3. E. coli
  4. H. influenza
  5. Klebsiella
Ref: Ghai 8/e p162, 163
33. The following bacteria are most often associated with acute neonatal meningitis except: (AI 05)
  1. Escherichia coli
  2. Streptococcus agalactiae
  3. Neisseria meningitidis
  4. Listeria monocytogenes
Ref: Ghai 8/e p162; Nelson 18/e p2513-2531 and 17/e p627, 2038; Nelson 19/e p2095
34. A neonate develops signs of meningitis at seven days of birth. The presence of which of the following infectious agent in the maternal genital tract can be the causative agent of this disease: (AIIMS May 04)
  1. Neisseria gonorrhea
  2. Chlamydia trachomatis
  3. Streptococcus agalactiae
  4. Haemophilous ducreyi
Ref: Ghai 8/e p163, 162; Nelson 18/e p2513-2531 and 17/e p2038; Harrison 16/e p829
35. Most common cause of neonatal meningitis: (PGI Dec 99)
  1. Staphylococcus
  2. E. coli
  3. H. influenza
  4. Pneumococcus
Ref: Ghai 8/e p162; Nelson 17/e p2038; Harrison 16/e p829
36. CSF picture in tubercular meningitis is: (AI 07)
  1. Increased protein, increased sugar, increased lymphocyte
  2. Increased protein, decreased sugar, increased lymphocyte
  3. Decreased protein, increased sugar, increased lymphocyte
  4. Decreased protein, decreased sugar, increased lymphocyte
Ref: Ghai 8/e p566; Nelson 18/e p2513-2531 and 17/e p2039; Nelson 19/e p2088
37. A 10 month old child presents with two weeks history of fever, vomiting and alteration of sensorium Cranial CT scan reveals basal exudates and hydrocephalus, the most likely etiological agent is: (AI 04, 95)
  1. Mycobacterium tuberculosis
  2. Cryptococcus neoformans
  3. Listera monocytogenes
  4. Streptococcus pneumonia
Ref: Ghai 8/e p566; Nelson 18/e p1248, 1249; Nelson 19/e p1066
38. The CSF findings in TB meningitis include: (AI 94, 07)
  1. High sugar + low protein
  2. Low sugar + high protein and Lymphocytosis
  3. High sugar + high chloride
  4. Low sugar + high protein and Lymphopenia
Ref: Ghai 8/e p566; Nelson 19/e p2088
39. All of the following are neural tube defects except: (AI 04)
  1. Myelomeningocoele
  2. Anencephaly
  3. Encephalocele
  4. Holoprosencephaly
Ref: Ghai 8/e p575; 7/e p550-551; Nelson 18/e p2449, 2450 and 17/e p1983-1987; Nelson 19/e p1998, 1999
40. Neural tube defects are prevented by: (AI 99)
  1. Pyridoxin
  2. Folic acid
  3. Thiamine
  4. Iron
Ref: Ghai 8/e p575; 576; 7/e p550-551; Nelson 19/e p2001-2002
83
41. Increase acetylcholinestrase in amniotic fluid indicates: (AI 99)
  1. Open neural tube defects
  2. Esophageal atresia
  3. Down’s syndrome
  4. Edward’s syndrome
Ref: Ghai 8/e p575; Nelson 18/e p2449, 2450 and 17/e p1983; Nelson 19/e p2001
42. Which one of the following is the common cause of congenital hydrocephalus? (AI 05)
  1. Craniosynostosis
  2. Intra uterine meningitis
  3. Aqueductal stenosis
  4. Malformations of great vein of Galen
Ref: Ghai 8/e p574; 7/e p549; Nelson 18/e p2452-2455 and 17/e p1990; Nelson 19/e p2008, 2009
43. Porencephaly refers to: (AI 99)
  1. Fetal alcohol syndrome
  2. Dandy-walker-syndrome
  3. Vascular lesion due to degenerative vessel disease and head injury
  4. Neural tube defects
Ref: Ghai 8/e p575; Nelson 18/e p2449 and 17/e p1987; Nelson 19/e p2010, 2011
44. Most common cause of hydrocephalus in children is: (AI 98)
  1. Post inflammatory obstruction
  2. Buddchiary syndrome
  3. Brain tumor
  4. Perinatal injury
Ref: Ghai 8/e p574; Nelson 18/e p2452-2455 and 17/e p1990, 2040
45. Macrocephaly is seen in which of the following syndromes? (AIIMS Nov 08)
  1. Metachromatic leukodystrophy
  2. Adrenoleukodystrophy
  3. Canavan’s disease
Ref: Ghai 8/e p35; CDTP 18/e (2007), Robbins 7/e p1398
46. Mild MR – Feature are: (PGI June 06)
  1. Present in 5-10% population
  2. Incidence in low socioeconomic group
  3. Present in 2 years
  4. Genetic background present
Ref: Nelson 18/e p191, 192, Ghai 8/e p39; Nelson 19/e p124
47. A male child of 15 years with a mental age of 9 years has an 1Q of: (AI 03)
  1. 50
  2. 60
  3. 70
  4. 80
Ref: Ghai 8/e p38
48. Condition associated with MR: (PGI 87)
  1. Trisomy 21
  2. Fragile - X
  3. Homocystinemia
  4. Phenylketonuria
  5. Tuberous sclerosis
  6. All
Ref: Ghai 8/e p39; Has been explained
49. Reye’s syndrome is characterized by: (PGI Dec 03)
  1. Viral infection is seen
  2. Present as deep jaundice
  3. Cerebral edema
  4. Microvesicular fatty infiltration
50. True about Reye’s syndrome: (PGI June 05)
  1. Microvesicular fatty infiltration
  2. Hepatic encephalopathy
  3. Brain edema
  4. Hypoglycemia
  5. All
Ref: Ghai 8/e p569; See previous explanations
51. 10 years old Ramu has increasing muscle weakness and raised CPK levels. The most likely defect is in plasma membrane of: (AIIMS Nov 01)
  1. Nerves
  2. Muscle fibers
  3. Basement membrane
  4. All body cells
Ref: Ghai 8/e p594; Nelson 18/e p2095 and 17/e p2053-2054; Harrison 16/e p1450; Nelson 19/e p2119, 2120
52. All of the following are associated with proximal muscle weakness except: (AIIMS May 02)
  1. Spinomuscular atrophy
  2. Duchenne muscular dystrophy
  3. Polymyositis
  4. Myotonic dystrophy
Ref: Ghai 8/e p589; Nelson 18/e p2544-2547 and 17/e p2065; Nelson 19/e p2123, 2125
53. All of the following are features of systemic Juvenile Rheumatoid Arthritis except: (AIIMS May 01)
  1. Uveitis
  2. Rash
  3. Fever
  4. Hepatosplenomegaly
Ref: Ghai 8/e p624; Nelson 18/e p1001-1010 and 17/e p799-801; Nelson 19/e p2173
54. Childhood osteopetrosis is characterized by: (PGI June 04)
  1. B/L frontal bossing
  2. Multiple fracture
  3. Hepatosplenomegaly
  4. Cataract
  5. Mental retardation
Ref: Ghai 8/e p587; Nelson 18/e p2882 and 17/e p2333
55. Duchenne Muscular Dystrophy is a disease of: (AI 04)
  1. Neuromuscular junction
  2. Sarcolemmal proteins
  3. Muscle contractile proteins
  4. Disuse atrophy due to muscle weakness
Ref: Ghai 8/e p594; 7/e p566-569; Nelson 18/e p2540 and 17/e p2060-2063; Harrison 16/e p2528-2529; Nelson 19/e p851
84
56. All of the following are essential features of attention deficit hyperactivity disease (ADHD) except: (AI 04)
  1. Lack of concentration
  2. Impulsivity
  3. Mental retardation
  4. Hyperactivity
Ref: Ghai 8/e p584; 7/e p37-38; Nelson 18/e p68-69 and 17/e p107-108
57. Duchenne’s dystrophy; investigation of choice: (AIIMS Nov 08)
  1. Serum creatine
  2. Neure conduct
  3. ESR
  4. CPK
Ref: Ghai 8/e p594; Nelson 18/e p2540; Nelson 19/e p2123
58. An infant presents with hypotonia and hyporeflexia. During his intrauterine period there was polyhydramnios and decreased fetal movements. Most probable diagnosis is: (AIIMS May 02)
  1. Spinal muscular atrophy
  2. Congenital myasthenia
  3. Congenital myotonia
  4. Muscular dystrophy
Ref: Ghai 8/e p593; 7/e p574-575; Nelson 18/e p2557-2559 and 17/e p2066, 2075, 534; Harrison 16/e p2520; Nelson 19/e p2136-2138
59. Give the most probable diagnosis of a 1-year-old child of normal intelligence with features of hypotonia. On examination there are tongue fasc iculations and he keeps his body in a frog like position: (AIIMS Nov 00)
  1. Guillian Barre Syndrome
  2. Limb girdle atrophy
  3. Down’s syndrome
  4. Spinal muscular atrophy
Ref: Ghai 8/e p595; 7/e p574-575; Nelson 18/e p2557-2559 and 17/e p2075; Nelson 19/e p2138
60. A 3-year-old child comes with complaint of limp diagnosis is: (PGI Nov 08)
  1. Septic arthritis
  2. Slipped capital femoral epiphysis
  3. Perthes’ disease
  4. DOH
Ref: Ghai 8/e p584; 7/e p578
61. Features of cerebral palsy: (PGI June 05)
  1. Athetosis
  2. Spasticity
  3. Saturday night palsy
  4. Mixed palsy
  5. Rigidity
Ref: Ghai 8/e p581; 7/e p575
62. A Child is suffering from fever, sinusitis, proptosis and extraocular muscle palsy. Possible diagnosis: (PGI June 03)
  1. Cavernous sinus thrombosis
  2. Langerhans cell histiocytosis
  3. Meningitis
  4. Secondary metastasis
  5. Frontal abscess
Ref: Ghai 8/e p570; Harrison 16/e p2490
63. Not found in cerebral palsy: (PGI June 98)
  1. Hypotonicity
  2. Microcephaly
  3. Ataxia
  4. Flaccid paralysis
Ref: Ghai 8/e p581; 7/e p575; Nelson’s Essentials of Pediatrics 5/e, p58; 18/e p2494, 2495; Nelson 19/e p2007, 2008
64. A 2 year old child is brought by parents with history of seizures and developmental delay. He has multiple hypopigmented macules over the back. What tis the most probab le diagnosis? (AI 2010)
  1. Neurofibromatosis type 1
  2. Tuberous sclerosis
  3. Sturge Weber syndrome
  4. Linear Sebacous Nevus Syndrome
Ref: Ghai 8/e p586; 7/e p564
65. True about infantile tremor syndrome:
  1. Hyperpigmentation of extremities
  2. Fine tremor
  3. Cortical atrophy
  4. Self-limiting disorder
  5. More common in girls
Ref: Ghai 8/e p580; 7/e p558
66. A child in status epilepticus should not be given: (JIPMER 04)
  1. Clonazepam
  2. Phosphenytoin
  3. Lamotrigine
  4. Diazepam
Ref: Ghai 8/e p558; 7/e p524; Nelson 17/e p2008
67. A previously well 2 year old boy presents with a brief generalized seizure. There is no past or family history of seizures. On examination, child is alert, active, febrile with axillary temperature of 39 – 40°c. Except for bilateral tonsillar enlargement and congestion, systemic examination is normal. Regarding the management at this time, consider the following: (UPSC 06)
  1. Immediate reduction of body temperature
  2. Intrevenous diazepam and phenytoin, followed by a lumbar puncture
  3. Institution of phenytoin for maintenance therapy for at least one year
  4. Rectal diazepam every 8 hourly for 2 – 3 days to reduce risk of recurrence.
Which of the above should be included in the management at this time?
  1. 1 and 2 only
  2. 1 and 4 only
  3. 3 and 4
  4. 1, 2 and 4
Ref: Ghai 7/e p528, 6/e p508-509
68. 4-year-old male child has febrile seizures. Best prophylaxis is: (UPSC 00)
  1. Paracetamol 6 hourly
  2. Paracetamol and diazepam
  3. Diazepam
  4. Phenobarbitone
Ref: Ghai 8/e p556; 7/e p528; CPDT. 15/e p645
85
69. Management of typical seizures include except: (Karnataka 05)
  1. Tepid sponging
  2. Paracetamol ibuprofen
  3. Intermittent diazepam
  4. Prophylactic phenobarbitone
Ref: Ghai 8/e p557; 7/e p528
70. Rapid antigen test for meningitis organisms in a child can be done on the following specimens except: (COMED 06)
  1. Blood
  2. CSF
  3. Urine
  4. Throat swab
Ref: Ghai 8/e p563; 7/e p537
71. Most common cause of Hydrocephlus in children is: (SGPGI 04)
  1. Post inflammatory obstruction
  2. Budd chiari syndrome
  3. Brain tumor
  4. Perinatal injury
  5. None
Ref: Ghai 8/e p574; Nelson 17/e p1990
72. Acquired extra–carnial infection that causes Aqueductal Stenosis is: (UPSC 04)
  1. Bacterial endocarditis
  2. Mumps
  3. Measles
  4. Staphylococcal septicemia
Ref: Ghai 8/e p209; Harrison 16/e p734
73. Which of the following has the worst prognosis: (JIPMER 01)
  1. Rolandic epilepsy
  2. Versive epilepsy
  3. Absence epilepsy
  4. Infantile spasm
Ref: Ghai 8/e p558; CPDT 16/e p730
74. Commonest cause of obstructive hydrocephalus in children: (UP 07, 05)
  1. Acqueductal stenosis
  2. Aqueductal gliosis
  3. Subarachnoid hemorrhage
  4. Tubercular meningitis
Ref: Ghai 8/e p574; Nelson 17/e, 1990
75. Child with generalized petechiae. CSF shows gram negative diplococci. Treatment: (PGI June 07)
  1. IV Ceftriaxone
  2. IV Penicillin G
  3. IV Cefotaxime
Ref: Ghai 8/e p566; Harrison 16/e, 2476
76. The behavior therapeutic falls in management of enuresis, The pharmacological drug of choice for this case is: (UP 07)
  1. Phenytoin
  2. Diazepam
  3. Imipramine
  4. Alprax
Ref: OP Ghai 8/e p504
77. Most common cause of meningoencephalitis in children?
  1. HSV
  2. Enterovirus
  3. Mumps
  4. Listeria
Ref: Ghai 8/e p563; Nelson 18/e p2521
78. Most common cause of meningoencephalitis in children is: (DNB 2007)
  1. HSVI > Enterovirus > HSV2
  2. Enterovirus
  3. Mumps
  4. Listeria
Ref: Ghai 8/e p721; Nelson 18/e p2521
79. Generalised 3-4 Hz spike and slow wave complexes in EEG are seen in: (DNB 2009)
  1. GTC
  2. Absensce seizures
  3. Myoclonic epilepsy
  4. Temporal lobe epilepsy
Ref: OP Ghai 8/e p720
80. Proximal muscle weakness is caused by all except: (DNB 2010)
  1. Erb duchenee paralysis
  2. Beckers paralysis
  3. Myotonic dystrophy
  4. Polymyositis
Ref: Ghai 8/e p580; Nelson 18/e p2544
81. Which of the following is a non-progressive neuro-motor deficit of central nervous system? (MP PG 2008)
  1. Metachromatic leucodystrophy
  2. Cerebral palsy
  3. Friederich’s ataxis
  4. Wilson disease
Ref: Ghai 8/e p581; Nelson’s 18/e p2494
82. In intraosseous canulation of tibia, the needle is inserted into the bone at an angle of: (MP PG 2008)
  1. 45°
  2. 60°
  3. 75°
  4. 90°
Ref: Ghai 8/e p731-732; Pediatric emergency procedures by Christoher King and Fred. M. Henreting, p69
83. Residual auditory defect is a common complication in meningitis caused by: (Karnataka PG 2011)
  1. Staphylococcus
  2. Meningococcus
  3. Escherichia coli
  4. Hemophilus influenza
Ref: OP Ghai 8/e p564; 7/e p536; Harrison’s 17/e p924
84. Sign of increased intracranial tension are all except: (UP PG 09)
  1. Papilledema
  2. Eighth nerve palsy
  3. Dilated scalp vein
  4. Diplopia
Ref: Ghai 8/e p570; 7/e p544
85. Drug of choice in status epilepticus: (UP PG 09)
  1. IV phenytoin
  2. Phenolbarbitone
  3. Ethosuximide
  4. Diazipam
Ref: Ghai 8/e p555; 7/e p526
86. Increase intracranial tension causes all except: (AP 2011)
  1. Tachycardia
  2. Papilloedma
  3. Headach
  4. Convulsions
Ref: Ghai 8/e p570
86
87. Management of typical febrile seizures include all the following except: (DP PGMEE 2010)
  1. Tepid sponging
  2. Paracetamol ibuprofen
  3. Intermittent diazepam
  4. Prophylactic phenobarbitone
Ref: Ghai 8/e p556; Nelson 18/e p2457
88. A 10-month-old child presents with 2 weeks’ history of fever, vomiting and alteration of sensorium. Cranial CT scan reveals basal exudates and hydrocephalus. The most likely etiological agent is: (Feb DP PGMEE 2009)
  1. Mycobacterium tuberculosis
  2. Cryptococcus neoformans
  3. Listeria monocytogenes
  4. Streptococcus pneumoniae
Ref: Ghai 8/e p566
89. In a pregnant woman with raised phenylalanine in blood, offspring is most likely to have: (Kerala PGI 08)
  1. Mental retardation
  2. Lighter complexion
  3. Microcephaly
  4. Reduction of limb growth
Ref: Ghai 8/e p652; Nelson 17/e p400; 18/e p531
90. Meningitis in a 4 year old child; microsopy of the Gram stained CSF sample will reveal: (WB PGI 08)
  1. Gram positive cocci
  2. Gram positive bacilli
  3. Gram negative diplococci
  4. Gram negative bacilli
Ref: Ghai 8/e p563; Harrison 17/e p2622
91. Hydrocephalus is associated with all except: (WB PG 08)
  1. Increased head size
  2. Bulging tense fofntanelle
  3. Diplopia
  4. Papilledema
Ref: Nelson 17/e p1990; Ghai 8/e p574
92. In Arnold Chiari malformation type II there is: (WB PG 08)
  1. Herniation of medulla oblongata
  2. Hermiation of medulla with cerebellum
  3. Meningomyelocele
  4. Aqueductal Stenosis
Ref: Ghai 8/e p574; Nelson 17/e p1990
93. All are congenital myopathies except: (WB PG 08)
  1. Nemaline
  2. Central core
  3. Centro nuclear/myotubular
  4. Mitochondrial
Ref: Ghai 8/e p594; Harrison 17/e p2688-89
94. Hypsarrythmia in EEG is seen in which of the following conditions: (J & K PGI 2011)
  1. Infantile spasms
  2. Absence attacks
  3. Partial seizure
  4. Generalised seizure
Ref: OP Ghai 8/e p559
95. Macewen’s sign indicates: (J & K PGI 2011)
  1. Hepatic failure
  2. Raised intracranial tension Increased Intracranial tension
  3. Meningitis
  4. Rickets
Ref: OP Ghai 8/e p570
96. Sub acute Sclerosing Panencephalitis follows an attack of: (J & K PGI 2011)
  1. Measles
  2. Varicella
  3. Mumps
  4. Rubella
Ref: Ghai 8/e p584
97. Midline intra- cranial cyst is: (J & K PGI 2010)
  1. Choroid plexus cyst
  2. Dandy-Walker cyst
  3. Unilateral hydrocephalus
  4. Porencephalic cyst
Ref: Ghai 8/e p575
 
 
Ans.
1. c. Calcium gluconate
2. b. 1, 2 and 4
3. b. High dose...
4. d. Exaggerated tendon...
5. c. Canavan’s disease
6. c. Krabbe’s disease
7. d. Exaggerated tendon...
8. b. Decreased level...
9. b and d
10. b. Paracetamol and...
11. b. Early age of onset
12. a. Myoclonic seizures...
13. a, c and e
14. a. Hypoxia induced...
15. d. Frequency of...
16. c. Subtle
17. c. Subtle
18. c. Postictal confusion
19. b. Absence seizures
20. b. Oral Diazepam...
21. d. Type B streptococcus
22. d. Listeria...
23. c. Third-generation...
24. b and c
25. c. Cerebral hamartoma
26. b. Meningococcal...
27. c. Measles
28. a, b and e
29. b. Streptococcus...
30. a. Pneumococcus
31. d. Streptococcal agalactiae
32. b, c and e
33. c. Neisseria meningitidis
34. c. Streptococcus...
35. b. E. coli
36. b. Increased protein...
37. a. Mycobacterium...
38. b. Low sugar + high...
39. d. Holoprosencephaly
40. b. Folic acid
41. a. Open neural...
42. c. Aqueductal stenosis
43. c. Vascular lesion...
44. a. Post inflammatory...
45. c. Canavan’s disease
46. b and d
47. b. 60
48. f. All
49. a, c and d
50. e. All
51. b. Muscle fibers
52. d. Myotonic dystrophy
53. a. Uveitis
54. b and c
55. b. Sarcolemmal proteins
56. c. Mental retardation
57. d. CPK
58. a. Spinal muscular atrophy
59. d. Spinal muscular atrophy
60. a, b and c
61. a, b, d and e
62. a. Cavernous sinus...
63. b. Microcephaly
64. b. Tuberous sclerosis
65. a, c and d
66. a > c
67. b. 1 and 4 only
68. b and c
69. d. Prophylactic...
70. d. Throat swab
71. a. Post inflammatory...
72. b. Mumps
73. d. Infantile spasm
74. a. Acqueductal stenosis
75. b. IV Penicillin G
76. c. Imipramine
77. b. Enterovirus
78. b. Enterovirus
79. b. Absensce seizures
80. c. Myotonic dystrophy
81. b. Cerebral palsy
82. d. 90°
83. d. Hemophilus influenza
84. b. Eighth nerve palsy
85. d. Diazipam
86. a. Tachycardia
87. d. Prophylactic...
88. a. Mycobacterium...
89. a. Mental retardation
90. a. Gram positive cocci
91. c. Diplopia
92. c. Meningomyelocele
93. d. Mitochondrial
94. a. Infantile spasms
95. b. Raised intracranial
96. a. Measles
97. b. Dandy-Walker cyst
 
8713. METABOLIC DISORDERS
 
88METABOLIC DISORDERS (QUESTIONS)
1. Injection of Glucagon is effective for management of persistent hypoglycemia in all except: (AIIMS May 04)
  1. Large for date baby
  2. Galactosemia
  3. Infant of diabetic mother
  4. Nesidioblastosis
Ref: Ghai 8/e p647; Nelson 17/e p505-507
2. Leigh disease is due to the accumulation of: (NIMHANS 01)
  1. Glycogen
  2. Pyrodoxine
  3. Sphingomyelin
  4. Lipofuscin
  5. None
Ref: Ghai 8/e p656; Nelson 16/e p1846-1847
3. A child presents with massive hepatomegaly and hypoglycemia. There is no improvement in blood glucose on administration of Glucagon. The probable diagnosis is: (AI 09)
  1. Von Gierke disease
  2. McArdle disease
  3. Cori’s disease
  4. Forbe’s disease
Ref: Ghai 8/e p647; 7/e p635; Nelson 18/e p602; Nelson 19/e p492
4. For which of the following diseases is enzyme replacement therapy available? (AI 04)
  1. Albinism
  2. Neimann-Pick disease
  3. Metachromatic leukodystrophy
  4. Gaucher’s disease cerezyme
Ref: Ghai 8/e p659; 7/e p635; Nelson 17/e p463-464; Harrison 16/e p2316-2317; Nelson 19/e p494
5. Enzyme replacement therapy is available for which of the following disorders? (AIIMS May 06)
  1. Gaucher’s disease
  2. Niemann-Pick disease
  3. Mucolipidosis
  4. Metachromatic leukodystrophy
Ref: Ghai 8/e p659; 7/e p635; Nelson 18/e p602 and 17/e p463-464; Harrison 16/e p2316-2317
6. Coarse facies, hepatosplenomegaly and tall QRS on ECG are characteristic feature of: (AIIMS Nov 01)
  1. Glycogen storage disease type II
  2. Hurler’s disease
  3. Hunters disease
  4. Hemochromatosis
Ref: Ghai 8/e p652; 7/e p636; Nelson 18/e p602 and 17/e p474; Harrison 16/e p2321-2322
7. True about Gauchcr’s disease: (PGI 2010)
  1. PAS positive cells
  2. Oil Red O-He cells
  3. Deficiency of acid Sphingomyelinase
  4. Def. Glucocerebrosidase
  5. Some marrow shows gaucher cells
Ref: Ghai 8/e p658-659; 7/e p638; Robbins 7/e p163, 164
8. Injection Glucagon is effective for management of persistent hypoglycemia in all, except: (AIIMS May 04)
  1. Large for date baby
  2. Galactosemia
  3. Infant of diabetic mother
  4. Nesidioblastosis
Ref: Ghai 8/e p655; 7/e p634; Dutta 6/e p287; 18/e p603, 609, 610; Nelson 19/e p503
9. Child of Vasanthi was weaned from breast milk on the 5th day and was given sugarcane juice the child developed hypoglycemia and hepatomegaly Biochemical examination showed hypophosphatemia and reducing substances in urine. The child is probably suffering from which of the following enzyme deficiencies: (AIIMS Nov 00)
  1. Fructokinase
  2. Aldolase B
  3. Glucose 6 Phosphatase
  4. Beta galactosidase
Ref: Ghai 8/e p657; 7/e p634, 635; Nelson 18/e p601, 602 and 17/e p476- 477; Nelson 19/e p503
10. A boy comes with complains of vomiting, bloated abdomen and abdominal pain. He has history of attending ice-cream eating competition last night. He also past history of similar episodes following ingestion of milk and milk products. The likely cause: (AIIMS Nov 99)
  1. Pancreatic amylase deficiency
  2. Lactase deficiency
  3. Salivary amylase deficiency
  4. Food poisoning
Ref: Ghai 8/e p657; Nelson 18/e p1554 and 17/e p1268; Nelson 19/e p502, 503
11. An 8 days old child presents with yellow sclera, whitish stool and turmeric colour urine on 3rd day of septicemia on broad spectrum antiboitics, the likely diagnosis is: (AIIMS June 98)
  1. Galactosidase deficiency
  2. Ammonia toxicity
  3. Galactose 1-4 phosphatase uridyl transferase deficiency
  4. Glucose 6 phosphatase deficiency
Ref: Ghai 8/e p656-657; 7/e p634; Nelson’s Essentials of Pediatrics 5/e, 254-255, 18/e p609, 610; Nelson 19/e p502
12. An infant presents with history of seizures and skin rashes. Investigations show metabolic acidosis increased blood ketone levels. This child is likely to be suffering from: (AI 02)
  1. Propionic aciduria
  2. Urea cyclic disorder
  3. Phenylketonuria
  4. Multiple carboxylase deficiency
Ref: Ghai 8/e p652; 7/e p626; Nelson 18/e p451 and 17/e p411; Nelson 19/e p417
89
13. True statement regarding a 3 week old child with Phenyl-ketonuria is all, except: (AI 00)
  1. Provocative protein meal tests helps in the diagnosis
  2. Tyrosine becomes an Essential amino-acid in diet
  3. Serum Phenylalanine is increased and urinary Phenyl Pyruvate level is elevated
  4. Phenylalanine should be completely stopped in diet
Ref: Ghai 8/e p652; 7/e p629; Harrison 17/e p2333–2334; Nelson 19/e p418-419
14. Deficiency of enzyme a-keto acid decarboxylase leading to a block in the metabolism of branch chain amino acids is observed in: (AIIMS May 05)
  1. Maple syrup urine disease
  2. Hartnup’s disease
  3. Alkaptonuria
  4. Phenylketonuria
Ref: Ghai 8/e p650; 7/e p632; Nelson 18/e p540-549 and 17/e p409
15. Treatment of multiple carboxylase deficiency: (AI 07)
  1. Biotin
  2. Pyridoxine
  3. Thiamine
  4. Folic acid
Ref: Ghai 8/e p653; Nelson 18/e p614
16. A 5 day old child presents with intractable seizures. He had rashes all over the body. Blood examination showed hyperammonemia and lactic acidosis. The probable diagnosis is: (AIIMS 2K)
  1. Organic aciduria
  2. Mitochondrial encephalopathy with lactic aciduria
  3. Phenylketonuria
  4. Urea cycle enzyme deficiency
Ref: Ghai 8/e p655; 7/e p632
17. Darkening of lurine on standing is associated with: (AI 07)
  1. Alkaptonuria
  2. Cystinuria
  3. Fabry’s disease
  4. Tyrosinemia
Ref: Ghai 8/e p652; 7/e p630; Nelson 18/e p534; Lippincott 3/e p272
18. In a patient, muscle cramps on exercise, positive myoglobulinemia, the disorder is: (PGI June 98)
  1. Pompe’s disease
  2. Myotonia congenital
  3. Myotonic dystrophy
  4. Mc Ardle’s disease
Ref: Ghai 8/e p659; 7/e p636, 637; Nelson’s Essentials of Pediatrics 5/e, p253; 18/e p602
19. LEIGH disease is due to the accumulation of: (NIMHANS 01)
  1. Glycogen
  2. Pyrodoxine
  3. Sphingomyelin
  4. Lipofuscin
  5. None
Ref: Nelson 16/e p1846 and 1847 also see p1884 and 1885. level of Pyruvate and lactose; Internet
20. Which glycogen storage disease does not affect muscles? (APPG 08)
  1. Type 1
  2. Type 2
  3. Type 3
  4. Type 4
Ref: Ghai 8/e p655; 7/e p637; Harrison 17/e p2458
21. Massive aminoaciduria without a corresponding increase in plasma amino acid level is characteristic of which one of the following diseases? (UPSC 108)
  1. Homocystinuria
  2. Hartnup disease
  3. Tyrosinemia
  4. Maple syrup urine disease
Ref: Nelson 18/e p539, 540; Ghai 8/e p652; 7/e p632
 
 
Ans.
1. b. Galactosemia
2. e. None
3. a. Von Gierke disease
4. d. Gaucher’s disease...
5. a. Gaucher’s disease
6. a. Glycogen storage...
7. a, d and e
8. b. Galactosemia
9. b. Aldolase B
10. b. Lactase deficiency
11. c. Galactose 1-4...
12. d. Multiple carboxylase...
13. d. Phenylalanine...
14. a. Maple syrup...
15. a. Biotin
16. a. Organic aciduria
17. a. Alkaptonuria
18. d. Mc Ardle’s disease
19. e. None
20. a. Type 1
21. b. Hartnup disease
 
9014. GENETIC DISORDERS
 
91GENETIC DISORDERS (QUESTIONS)
1. Which of the following when present is associated more than 20% of the time with a chromosomal disorder? (AIIMS Nov 94)
  1. Omphalocele
  2. Gastroschisis
  3. Spinabifida
  4. Cleft Palate
Ref: Atlas of genetic diagnosis and counseling, CPDT 19/e p1011; Nelson 19/e p1252
2. Which one of the following is NOT true regarding Noonan’s syndrome: (COMED 06)
  1. Affects males and females
  2. Short stature
  3. Chromosomal abnormality
  4. Congenital heart disease – ASD
Ref: Ghai 7/e p475, 6/e p493; Nelson 17/e p1925
3. The chromosomal disorder in patau syndrome is: (UP 07)
  1. Chromosome 21
  2. Chromosome 18
  3. Chromosome 13
  4. Chromosome 45x / 46xx
Ref: OP Ghai 8/e p643
4. Which one of the following is a distinguishing feature of Edward’s syndrome: (UPSC 07)
  1. Hypotonia
  2. Hypotelorism
  3. Holoprosencephaly
  4. Rocker bottom feet
Ref: Ghai 8/e p638; Nelson 17/e, 384
5. Only male are affected in: (AIIMS May 08)
  1. Scheie’s syndrome
  2. Hunter’s syndrome
  3. Hurler’s syndrome
  4. Gaucher’s disease
Ref: Ghai 8/e p662; 7/e p635; CPDT 18/e p1006, 1007; Nelson 19/e p511-512
6. For which of the following diseases is enzyme replacement therapy available: (AI 04, AIIMS May 06, Nov 03)
  1. Albinism
  2. Neimann-Pick disease
  3. Metachromatic leukodystrophy
  4. Gaucher’s disease
Ref: Ghai 8/e p659; Nelson 18/e p595, 596
7. A child with a small head, minor anomalies of the face including a thin upper lip, growth delay, and developmental disability can have all of the following, except: (AI 06)
  1. A chromosomal syndrome
  2. A teratogenic syndrome
  3. A mendelian syndrome
  4. A polygenic syndrome
Ref: 8/e p635; Nelson’s Essentials of Pediatrics 5/e, p217 Nelson 19/e p393, 394
8. An affected male infant born to normal parents could be an example of all of the following, except: (AI 06)
  1. An Autosomal dominant disorder
  2. An Autosomal recessive disorder
  3. A polygenic disorder
  4. A vertically transmitted disorder
Ref: Ghai 8/e p636; 7/e p617; Nelson’s Essentials of Pediatrics 5/e, p221; Nelson 19/e p383
9. Absent thumb, radial deviation of wrist, bowing of forearm, which inv. need not to be done? (AIIMS Nov 07)
  1. Echocardiography
  2. Bone marrow examination
  3. Platelet count
  4. Karyotyping
Ref: Ghai 8/e p645; 7/e p611
10. All of the following conditions have autosomal dominant inheritance except: (AI 07)
  1. Fabry disease
  2. Marfan’s syndrome
  3. Osteogenesis imperfecta
  4. Ehlers Danlos syndrome
Ref: Ghai 8/e p638; Robbin’s 7/e p151
11. The chances of having an unaffected baby, when both parents have achondroplasia, are: (AI 05, AIIMS May 04)
  1. 0%
  2. 25%
  3. 50%
  4. 100%
Ref: Ghai 8/e p642; Robbin’s 7/e p160, 161
12. In family, the father has widely spaced eyes, increased facial hair and deafness. One of the three children has deafness with similar facial features. The mother is normal. Which one of the following is least likely pattern of inheritance in this case: (AI 06)
  1. Autosomal dominant
  2. Autosomal recessive
  3. X-linked dominant
  4. X-linked recessive
Ref: Ghai 8/e p641
13. A parent is homozygous and a parent heterozygous for an autosomal recessive gene. What will be the outcome: (AIIMS May 94)
  1. 75% children affected
  2. No child affected, but all are carriers
  3. 50% children affected, rest are carriers
  4. 25% children affected, rest are carriers
Ref: Ghai 8/e p636; Robbin’s 7/e p161
14. For a normal husband and wife the first child was diagnosed to have cystic fibrosis. What is the percentage of chances for the second child be affected: (PGI June 06)
  1. 25
  2. 50
  3. 0
  4. 75
  5. 100
Ref: Robbin’s 7/e p151; Nelson 19/e p385
92
15. An albino girl gets married to a normal boy, What are the chances of their having an affected child and what are the chances of their children being carriers? (AI 03)
  1. None affected, all carriers
  2. All normal
  3. 50% carriers
  4. 50% affected, 50% carriers
Ref: Ghai 8/e p636; Robbin’s 7/e p151; Nelson 19/e p385
16. In an Autosomal Recessive (AR) disorder, one parent is normal and the other is carrier and the child is also affected. What is the reason: (AI 07)
  1. Germ line mosaicism
  2. Genomic imprinting
  3. Penetrattion
  4. Uniparental disomy
Ref: Ghai 8/e p641; Nelson 19/e p392
17. Fragile X-syndrome is characterized by all of the following features except: (AI 99)
  1. Long face
  2. Large ear
  3. Large-nose
  4. Large-testis
Ref: Nelson 18/e, 135, 499 and 17/e p388-389; Internet
18. Kinky hair desease is disorder where an affected child has peculiar white stubby hair, does not grow, brain degeneration is seen and dies by age of two years. Mrs A is hesitant about having children because her two sisters had sons who had died form kinky hair disease. Her mother’s brother also died of the same condition. Which of the following is the possible mode of inheritance in her family: (AI 04)
  1. X-linked recessive
  2. X-linked dominant
  3. Autosomal recessive
  4. Autosomal dominant
Ref: Ghai 8/e p643; Robbin’s 7/e p152; Nelson 19/e p388
19. The process underlying differences in expression of a gene, according to which parent has transmitted, is called: (AI 06)
  1. Anticipation
  2. Mosaicism
  3. Nonpenetrance
  4. Genomic imprinting
Ref: Ghai 8/e p641; Harrison 16/e p375; Robbins 7/e p186; Nelson 19/e p391
20. Differential expression of same gene depending on parent of origin is referred to as: (AI 05, 06)
  1. Genomic imprinting
  2. Mosaicism
  3. Anticipation
  4. Nonpenetrance
Ref: Ghai 8/e p641; Robbins 7/e p186; Nelson 19/e p391
21. Atavism means child resembles with his: (AIIMS Nov 99)
  1. Father
  2. Siblings
  3. Grand parents
  4. Neighbor
Ref: Taber’s Medical Dictionary
22. Single gene defect causing multiple unrelated problems: (AIIMS Nov 06)
  1. Pleotropism
  2. Pseudo dominance
  3. Penetrance
  4. Anticipation
Ref: Nelson’s Essentials of Pediatrics 5/e, p221
23. True in Klinefelter syndrome is: (AI 98)
  1. Short stature
  2. Pituitary adenoma
  3. Subnormal intelligence
  4. Breast adenoma
Ref: Ghai 7/e p615, 6/e p594; Harrison 16/e p2214-2215
24. A baby presenting with multiple deformities, cleft lip, cleft palate, microcephaly, small eyes, scalp defect and polydactyly, seen in which patau syndrome: (AIIMS Nov 06, May 08)
  1. Trisomy 13
  2. Trisomy 18
  3. Trisomy 21
  4. Monosomy 2
Ref: Robbin’s 7/e p176, 177, OP Ghai 7/e p615; 6/e p593; Nelson 19/e p404
25. Cat eye syndrome is: (AI 07)
  1. Partial trisomy 18
  2. Partial trisomy 13
  3. Partial trisomy 21
  4. Partial trisomy 22
Ref: Dorland’s 28/e, p162
26. A Down syndrome patient is posted for surgery, the necessary preoperative investigation to be done is: (AI 08)
  1. Echocardiography
  2. CT brain
  3. X-ray cervical spine
  4. USG abdomen
Ref: Ghai 8/e p638; 7/e p613; Nelson 18/e p507-509; Nelson 19/e p402
27. All of the following may occur in Down’s syndrome except: (AI 06)
  1. Hypothyroidism
  2. Undescended testis
  3. Ventricular septal defect
  4. Brushfield’s spots
Ref: Ghai 8/e 8/e p637-638; 7/e, 613, 614; Nelson 18/e, 507-509 and17/e p384; Robbin’s illustrated 7/e p176-177; Nelson 19/e p402
28. Transient myeloproliferative disorder of the newborn is seen in association with: (AI 03)
  1. Turner syndrome
  2. Down’s syndrome
  3. Neurofibromatosis
  4. Ataxia telangiectasia
Ref: Ghai 8/e p599; Nelson 18/e p507-509 and 17/e p1697; Nelson 19/e p401, 402
29. Triple test for diagnosis of down’s syndrome includes all of the following except: (AI 99)
  1. P-HCG
  2. a-Fetoprotein
  3. Serum HPL level
  4. Serum oestriol level
Ref: Ghai 8/e p638; Dutta 5/e, p112; Nelson 19/e p401, 402
93
30. All of the following are features of down’s syndrome except: (AI 05)
  1. Increased PAPPA
  2. Increased free beta HCG levels
  3. Absent nasal bone
  4. Abnormal ductus venous flow velocity
Ref: Ghai 8/e p638; Nelson 18/e p567-509 and 17/e p535; Dutta, 6/e p107; Nelson 19/e p402
31. A 35 years old lady has chromosomal translocation 21/ 21. The risk of Down’s syndrome in the child is: (AIIMS June 99)
  1. 100%
  2. 0%
  3. 10%
  4. 50%
Ref: Ghai 8/e p637; Nelson 18/e p490, 509 and 17/e p387; Nelson 19/e p402
32. Down’s syndrome most commonly occurs due to? (AI 2010)
  1. Reciprocal translocation
  2. Nondysjunction in maternal meiosis
  3. Translocation defect
  4. Nondysjunction in paternal meiosis
Ref: Ghal 8/e p637; 7/e p630; H-17/Table 63-3; Nelson 19/e p402
33. Webbing of neck, increased carrying angle, low posterior hair line and short forth metacarpal are characteristics of: (AI 04)
  1. Klinefelter syndrome
  2. Turner’s syndrome
  3. Cri-du-chat syndrome
  4. Noonan’s syndrome.
Ref: Ghai 8/e p536, 640; Nelson 18/e p2386-2389; Robbin’s illustrated 7/e p179-180; Nelson 19/e p409
34. All of the following may occur in Noonan’s syndrome except: (AI 03)
  1. Hypertrophic cardiomyopathy
  2. Cryptorchidism
  3. Infertility in females
  4. Autosomal dominant transmission
Ref: Ghai 8/e p537; Nelson 18/e p514 and 17/e p1935, 1925
35. Turner syndrome is maximally associated with: (AIIMS May 08)
  1. Horseshoe kidney
  2. Coarctation of aorta
  3. VSD
  4. ASD
Ref: Ghai 8/e p536; Nelson 17/e p2387
36. A patient with short stature, sexual infantilism and congenital anomalies with chromosomal abnormalities ‘XO’. Diagnosis is: (PGI June 04)
  1. Turner’s syndrome
  2. Klinefelter syndrome
  3. Testicular feminization slyndrome
  4. Gonadal agenesis
  5. Gonadal dysgenesis
Ref: Ghai 8/e p640; See previous question
37. Posterior iliac horns are seen in: (AIIMS May 02)
  1. Fisher’s syndrome
  2. Crouzan syndrome
  3. Nail patella syndrome
  4. Pierre Robbins syndrome
Ref: Nelson 18/e p2731-2882 and 17/e p2220, 2332; Nelson 19/e p2293, 2294; Internet
38. A 4-year-baby is having large face, large jaw, large ear and macro orchidism is: (AMU 05)
  1. Mc Cune-Albright syndrome
  2. Down’s syndrome
  3. Cri-du chat syndrome
  4. Fragile X syndrome
Ref: Ghai 8/e p634; 7/e p6
39. Down’s syndrome predisposes to....cancer: (MAHE 05)
  1. AML
  2. CML
  3. ALL
  4. CLL
Ref: Ghai 8/e p599; 7/e p613, 614
40. In ‘Down’s symdrome’ the shape of the head is: (MAHE 05)
  1. Oxycephalic
  2. Scaphocephalic
  3. Brachicephalic
  4. Plagiocephalic
Ref: Ghai 8/e p637; 7/e p613
41. The chromosomal disorder in patau syndrome is: (UP 07)
  1. Chromosome 21
  2. Chromosome 18
  3. Chromosome 13
  4. Chromosome 45x /46xx
Ref: Ghai 8/e p640; 7/e p615
42. Which one of the following is a distinguishing feature of Edward’s syndrome: (UPSC 07)
  1. Hypotonia
  2. Hypotelorism
  3. Holoprosencephaly
  4. Rocker bottom feet
Ref: Ghai 8/e p635, 636; 7/e p614
43. Which one of the following is NOT true regarding Noonan’s syndrome: (COMED 06)
  1. Affects males and females
  2. Short stature
  3. Chromosomal abnormality
  4. Congenital heart disease - ASD
Ref: Ghai 8/e p636; 7/e p616
44. A particular genetic disorder appears in three consecutive generations of a family without any sex predilection. It was also noticed that phenotypically normal family members were have helthy offspring. What is the pattern of inheritance of this disorders? (UPSC 06)
  1. Autosomal recessive
  2. Autosomal dominant
  3. Mitochondrial inheritance
  4. Uniparental disomy
Ref: Ghai 8/e p636
94
45. Maternal disomy of chromosome 15 is seen in:
  1. Prader-Willi syndrome
  2. Klinefelter’s syndrome
  3. Angelman syndrome
  4. Turner’s syndrome
Ref: Ghai 8/e p641; Nelson, Textbook of Pediatrics, 18/e p498, 513, 516
46. A child presents with faliure to thrive with frequent vomiting, diarrhea, hepatic splenomegaly and abdominal distension. CT shows adrenal calcification. Which of the following is the diagonosis?
  1. Adrenal hemorrahage
  2. Wolman’s disease
  3. Pheochromocytoma
  4. Addisons disease
Ref: Current Diagnosis and Treatment “Pediatrics 19/e p2009
47. All are true regarding Dent’s disease except?
  1. Proteinura
  2. Hypercalciuria
  3. Father will have the same disease
  4. Rickets is present
Ref: Nelson Textbook of Pediatrics, 18/e p519
48. Duchenne muscular dystrophy is transmitted as: (DNB 2005)
  1. X-linked dominant
  2. X-linked recessive
  3. Autosomal dominant
  4. Autosomal recessive
Ref: Ghai 8/e p522; Kumar Kotran, Oxford Medicine, Philip Kalra, CMDT, NMS Medicine, Harrison
49. Unconjugated hyperbilirubinemia in newborn is caused by following, except: (DNB 2006)
  1. Breast milk jaundice
  2. Dubin Johnnson Syndrome
  3. Sphereocytosis
  4. Crigler najjar syndrome
Ref: Ghai 8/e p433
50. All of the following are features of down’s syndrome, except: (DNB 2008)
  1. Increased PAPPA
  2. Increased free beta HCG levels
  3. Absent nasal bone
  4. Abnormal ductus venous flow velocity
Ref: Ghai 8/e p410; Nelson essentials of Pediatrics 4/e p13, 18, 141, 142
51. Turner syndrome is maximally associated with: (DNB 2008)
  1. Pancake kidney
  2. Coarctation of aorta
  3. VSD
  4. ASD
Ref: OP Ghai 8/e p524, Nelson essentials of Pediatrics 4/e p143, 210, 727
52. True about Turner’s Syndrome is: (DNB 2009)
  1. Normal breast
  2. Normal gonads
  3. Normal intelligence
  4. Long stature
Ref: OP Ghai 8/e p547; Nelson essentials of Pediatrics 4/e p143, 210, 727
53. Single gene defect causing multiple unrelated problems: (DNB 2009)
  1. Pleiotropism
  2. Pseudodominance
  3. Penetrance
  4. Anticipation
Ref: NMS Medicine, Genetics
54. All of the following are features of down’s syndrome, except: (DNB 2009)
  1. Increased PAPPA
  2. Increased free beta HCG levels
  3. Absent nasal bone
  4. Abnormal ductus venous flow velocity
Ref: Ghai 8/e p638; Nelson essentials of Pediatrics 4/e p13, 18, 141, 142
55. Turner syndrome is maximally associated with: (DNB 2009)
  1. Pancake kidney
  2. Coarctation of aorta
  3. VSD
  4. ASD
Ref: OP Ghai 8/e p524; Nelson essentials of Pediatrics 4/e p143, 210, 727
56. A neonate with recurrent infection and abscess was diagnosed with Kostmann syndrome. Treament is: (DNB 2010)
  1. G-CSF
  2. GM-CSF
  3. Antithymocyte globulin+cyclosporin
  4. Antithymocyte globulin+cyclosporin +GM-CSF
Ref: Campbell Text book of Peds, 10/e p1259; Internet
57. Which of the following is associated with >20% risk of chromosomal anomalies: (DNB 2010)
  1. Cleft lip
  2. Gastroschisis
  3. Omphalocele
  4. Spina bifida
Ref: Nelson Essentials of Pediatrics, 4/e p199, 211, 493, 201, Updates on associations of Omphalocele
58. Death in Klienfilters syndrome is usually because of: (DNB 2011)
  1. Respiratory causes
  2. Cardiovascular causes
  3. Cerebrovascular causes
  4. Infections
Ref: Ghai 8/e p732; Esssentials of Pediatrics, Nelson 4/e p143, 144, 647, Harrison 17/e p2340, 2341
59. The karyotype associated with the classic pattern of Klienfelter’s syndrome is: (MP PG 2010)
  1. 47, XXY
  2. 45, X
  3. 47, XX
  4. 47, XX, + 21
Ref: Ghai 8/e p640; Nelson’s 18/e p514
95
60. A two years old girl child is brought to the out patient with features of hand wringing, stereotype movements, impaired stereotype movements, impaired language and communication development, breath holding spells, poor social skills and deceleration of head growth after six months of age. The most likely diagnosis is: (MP PG 2010)
  1. Asperger syndrome
  2. REtt’s syndrome
  3. Fragile-X syndrome
  4. Cottard syndrome
Ref: Nelson’s 18/e p2504
61. Increased nuchal fold thickness and reduced length of femur in a fetus by prenatal ultrasound predicts: (MP PG 2008)
  1. Trisomy 21
  2. Turner’s syndrome
  3. Trisomy 18
  4. Trisomy 13
Ref: Ghai 8/e p637; Nelson’s 18/e p509-509
62. The gene for Wilson’s disease is on: (MHPGM-CET 2010)
  1. Long arm of Chromosome 13
  2. Long arm of Chromosome 6
  3. Short arm of Chromosome 13
  4. Short arm of Chromosome 6
Ref: OP Ghai 8/e p320; 7/e p291; Robbins pathology 7/e p910
63. Which of the following is common in the children of pregnant woman age is above 40 years of age? (AP 2010)
  1. Klinefelter’s syndrome
  2. Turner’s syndrome
  3. Marfan’s syndrome
  4. Down’s syndrome
Ref: Ghai 8/e p637
64. A 50-year-old boy with Down’s syndrome is found to have 21/21 Translocation. The chromosomal analysis of the parents revealed the father to have a balanced translocation. The mother has missed a period and the parents want to know the risk for the fetus: (AP 2012)
  1. 15%
  2. 25%
  3. 40%
  4. 100%
Ref: Ghai 8/e p637
65. Wiskott-Aldrich syndrome is characterized by all of the following, except: (AP 2012)
  1. X-linked recessive mode of inheritance
  2. Atopic dermatitis and eczema during infancy
  3. Thrombocytopenia
  4. IgM is decreased
Ref: AP–2012; WB–2006;
66. Transient myeloproliferative disorder of the newborn is seen in association with: (DP PGMEE 2010)
  1. Turner’s syndrome
  2. Down’s syndrome
  3. Neurofibromatosis
  4. Ataxia telangiectasia
Ref: Ghai 8/e p637; Nelson 18/e p2122
67. Acute leukemia in children is associated with: (Feb DP PGMEE 2009)
  1. Down’s syndrome
  2. Klinefelter’s syndrome
  3. Marfan’s syndrome
  4. Turner’s syndrome
Ref: Ghai 8/e p599
68. All are features of Di George syndrome except: (Kerala PG 08)
  1. Cardiac anomaly
  2. Facial dystrophy
  3. Cell mediated immune deficiency
  4. Hypercalcaemia
Ref: Ghai 8/e p185; Harrison 16/e p1942; 17/e p2057; Nelson 17/e p211, 694, 735; 18/e p884, 885
69. Feature of Down Syndrome: (Kerala PG 09)
  1. 18 trisomy
  2. 14/21 translocation
  3. 21 trisomy
  4. Trisomy 13
Ref: Nelson 18/e p637; 508
70. Myeloproliferative disorders (transient?) occur with increased incident in children with: (WB PG 08)
  1. Down’s syndrome
  2. Turner’s syndrome
  3. Ataxia telanglectasia
  4. Neuro fibromatosis
Ref: Ghai 8/e p637; Nelson 17/e p1697
71. Fragile X syndrome is associated with all except: (WB PG 08)
  1. Large nose
  2. Large face
  3. Large ears
  4. Large testis
Ref: Nelson 17/e p388
72. Down syndrome diagnosed by triple test with all except: (UP PG 2010)
  1. Alpha Feto Protein
  2. HCG
  3. Estriol
  4. Esteriol
Ref: Ghai 8/e p637; 7/e p614
73. The gene for ataxia telangiectasia is localized to chromosome: (J & K PG 2011)
  1. 11 q
  2. 12 p
  3. 21 p
  4. 18 q
Ref: Ghai 8/e p580
74. Karotyping of a healthy, normal male baby with monglian spots is: (J & K PG 2010)
  1. 46 XY
  2. 46XX
  3. Trisomy XYn
  4. 45 XO
Ref: Ghai 8/e p679
 
 
Ans.
1. a. Omphalocele
2. c. Chromosomal...
3. c. Chromosome 13
4. d. Rocker bottom feet
5. b. Hunter’s syndrome
6. d. Gaucher’s disease
7. d. A polygenic syndrome
8. a. An Autosomal...
9. d. Karyotyping
10. a. Fabry disease
11. b. 25%
12. a. Autosomal dominant
13. c. 50% children...
14. c. 0
15. a. None affected...
16. d. Uniparental disomy
17. c. Large-nose
18. a. X-linked recessive
19. d. Genomic imprinting
20. a. Genomic imprinting
21. c. Grand parents
22. a. Pleotropism
23. c. Subnormal…
24. a. Trisomy 13
25. d. Partial trisomy 22
26. a. Echocardiography
27. b. Undescended testis
28. b. Down’s syndrome
29. c. Serum HPL level
30. a. Increased PAPPA
31. a. 100%
32. b. Nondysjunction...
33. b. Turner’s syndrome
34. c. Infertility in females
35. b. Coarctation of aorta
36. a. Turner’s syndrome
37. c. Nail patella syndrome
38. d. Fragile X syndrome
39. a. AML
40. c. Brachicephalic
41. c. Chromosome 13
42. d. Rocker bottom feet
43. c. Chromosomal...
44. b. Autosomal dominant
45. a. Prader-Willi…
46. b. Wolman’s disease
47. c. Father will have...
48. b. X-linked recessive
49. b. Dubin Johnnson...
50. a. Increased PAPPA
51. b. Coarctation of aorta
52. c. Normal intelligence
53. a. Pleiotropism
54. a. Increased PAPPA
55. b. Coarctation of aorta
56. a. G-CSF
57. c. Omphalocele
58. b. Cardiovascular...
59. a. 47, XXY
60. b. REtt’s syndrome
61. a. Trisomy 21
62. a. Long arm of...
63. d. Down’s syndrome
64. d. 100%
65. d. IgM is decreased
66. b. Down’s syndrome
67. a. Down’s syndrome
68. d. Hypercalcaemia
69. b and c
70. a. Down’s syndrome
71. a. Large nose
72. d. Esteriol
73. a. 11 q
74. a. 46 XY
 
9615. CHILDHOOD MALIGNANCIES
 
97CHILDHOOD MALIGNANCIES (QUESTIONS)
1. Which of the following statements about cerebellar astrocytomas in pediatric age group is False: (AI 08)
  1. These are usually Low grade tumors
  2. These tumors have a good prognosis
  3. These are more commonly seen of the 1st and 2nd decades
  4. These tumors are more common in females
Ref: Ghai 8/e p612; NTBP 18/e p2130; Nelson 19/e p1747
2. True statement regarding Brain Tumor in children is: (AI 00)
  1. Mostly is infra-tentorial
  2. Papilledema is rare
  3. Is the most common tumor in children
  4. Hydrocephalus is rare
Ref: Ghai 8/e p612, 613; Nelson 18/e p2487-2492; Nelson 19/e p1746, 1747
3. Common posterior-cranial fossa tumors include all of the following except: (AI 99)
  1. Medulloblastoma
  2. Oligodendroglioma
  3. Ependydoma
  4. Cystic astrocytoma
Ref: Ghai 8/e p610; Nelson’s Essentials of Pediatrics 5/e p742; 18/e p2132; Nelson 19/e p1747
4. A 10 year old boy presents with midline swelling arising from cerebellum. The diagnosis is: (AIIMS Dec 98)
  1. Astrocytoma
  2. Glioblastoma multiforme
  3. Ependymoma
  4. Medulloblastoma
Ref: Ghai 8/e p613; Harrison 16/e p2452–2455; Nelson 19/e p1748
5. Infantile polycythemia is seen in: (PGI June 98)
  1. Cerebellar hemangioblastoma
  2. Retinoblastoma
  3. Hepatoblastoma
  4. All
Ref: Ghai 8/e p623; Robbin’s 7/e p649; Nelson 18/e p2487
6. A 6 year old boy has been complaining of headache, ignoring to see the objects on the sides for four months. On examination he is not mentally retarded, his grades at school are good, and visual acuity is diminished in both the eyes. Visual charting showed significant field defect. CT scan of the head showed suprasellar mass with calcification. Which of the following is the most probable diagnosis? (AIIMS Sep 96)
  1. Astrocytoma
  2. Craniopharyngioma
  3. Pituitary adenoma
  4. Meningioma
Ref: Ghai 8/e p613; Has been explained
7. Which one of the following statements is true of brain tumors in childhood? (SGPGI 04)
  1. Is a rare from of malignancy
  2. Most tumors are below the tentorium
  3. Hemiparesis is a frequent form of presentation
  4. Papilledema is infrequent
Ref: Ghai 8/e p610; 7/e p545-546
8. Commonest location for craniopharyngioma is: (SGPGI 05)
  1. Intrasellar
  2. Suprasellar
  3. Intraventricular
  4. Intracerebral
Ref: Ghai 8/e p612; 7/e p546
9. Commonest childhood malignancy is: (AI 07)
  1. Leukemia
  2. Lymphoma
  3. Neuroblastoma
  4. Wilm’s tumor
Ref: Ghai 8/e p599; 7/e p580; Nelson 18/e p2116-2122; Nelson 19/e p1732
10. All of the following is associated with good prognosis in childhood leukemia except: [AI 07]
  1. Common ALL subtype.
  2. Precursor B cell ALL
  3. Hyperdiploidy.
  4. Female gender
Ref: Ghai 8/e p599; 7/e p580-581; Nelson 18/e p2116-2122 and 17/e p1696;Robbins illustrated 7/e, 673; Nelson 19/e p1732, 1733
11. A 5 year old child presents with history of fever off and on for past 2 weeks and petechial spots all over the body and increasing pallor for past 1 month. Examination reveals splenomegaly of 2 cm below costal margin. The most likely diagnosis is: (AI 04)
  1. Acute leukemia
  2. Idiopathic thrombocytopenic purpura
  3. Hypersplenism
  4. Aplastic anemia
Ref: Ghai 8/e p596; 7/e p584-585; Nelson’s Essentials of Pediatrics 5/e, p737; 18/e p2116-2120; Nelson 19/e p1732
12. Most common malignancy in children is: (AI 99)
  1. Retinoblastoma
  2. Lymphoma and leukemia
  3. Wilm’s tumor
  4. Neuroblastoma
Ref: Ghai 8/e p599; 7/e p580; Nelson 18/e p2116-2120; Nelson 19/e p1732, 1733
13. Poor prognostic indicator of ALL is:
  1. Female sex [AIIMS May 02]
  2. Leukocyte count < 50,000
  3. Age greater than 1 year
  4. Hypodiploid
Ref: Ghai 8/e p599, 600; 7/e p581; Nelson 18/e p2116-2122 and 17/e p1696; Robbin’s illustrated 7/e p673
98
14. X-ray features of leukemia in a 2 year old child is/ are: (PGI Dec 00)
  1. Osteolytic lesions in flat bones
  2. Subperiosteal erosions
  3. Osteoporosis
  4. Thick line just below growth plate
  5. Metaphyseal infarcts
Ref: Ghai 8/e p599; Sutton 6/e p2001
15. All of the following are good progonostic factor for All except: (AIIMS Nov 08)
  1. Age of onset between 2-8 years
  2. Initial WBC count less than 50000
  3. Hyperdiploid
  4. t (9: 22), t (8:14), t (4:11)
Ref: Ghai 8/e p599; Sutton 7/e p1327, 6/e p2001; Nelson 19/e p1733
16. A 2 year old boy suffering from leukemia, following are the x-ray findings: (PGI June 03)
  1. Osteolytic lesion in flat bones
  2. Metaphyseal osteoporosis
  3. Periosteal new bone formation
  4. Osteosclerosis of long bones
  5. Transverse line of dark band below the growth plate
Ref: Ghai 8/e p608; See above explanation
17. All of the following syndromes are associated with AML except: (AI 07)
  1. Down’s syndrome
  2. Klinefelter’s syndrome
  3. Patau syndrome
  4. Turner’s syndrome
Ref: Ghai 8/e p606; Harrson 16/e p631; Nelson 19/e p1738
18. Transient myeloproliferative disorder of the newborn is seen in association with:
  1. Turner syndrome
  2. Down syndrome
  3. Neurofibromatosis
  4. Ataxia telangiectasia
Ref: Ghai 8/e p599; Nelson 18/e p2122; Nelson 19/e p1738
19. Which of the following childhood tumors most frequently metastasizes to the bone: (AI 02)
  1. Neuroblastoma
  2. Ganglioneuroma
  3. Wilm’s tumor
  4. Ewing’s Sarcoma
Ref: Ghai 8/e p616; Nelson 17/e p1709–1713; Nelson 19/e p1753, 1754
20. A malignant tumor of childhood, that metastasizes to bones most often, is: (AI 06)
  1. Wilm’s tumor
  2. Neuroblastoma
  3. Adrenal gland tumors
  4. Granulose cell tumor of ovary
Ref: Ghai 8/e p616; 7/e p590; Nelson 18/e p2137-2139 and 17/e p1709-1713; Nelson 19/e p1753-1755
21. The most common malignant neoplasm of infancy is: (AI 05)
  1. Malignant teratoma
  2. Neuroblastoma
  3. Wilm’s tumor
  4. Hepatoblastoma
Ref: Ghai 8/e p616; 7/e p590; Nelson 18/e p2137-2139 and 17/e p1709; Nelson 19/e p1754
22. Most common presentation of neuroblastoma is:
  1. Lytic lesion in skull with suture diasthesis
  2. Lung metastasis [AI 98]
  3. Renal invasion
  4. Secondaries in brain
Ref: Ghai 8/e p616; 7/e p590; Nelson 18/e p2137-2139 and 17/e p1709-1710; Nelson 19/e p1754
23. Neuroblastoma originates from: (PGI June 08)
  1. Adrenals
  2. Mediastinum
  3. Chest wall
  4. Peripheral nerves
  5. Neck
Ref: Ghai 8/e p616; 7/e p590-591; Robbin S 7/e p500; Nelson 19/e p1754
24. Childhood malignancy producing proptosis is/are: (PGI June 05)
  1. Neuroblastoma
  2. Hepatoma
  3. Retinoblastoma
  4. Germ cell tumor
  5. Nephroblastoma
Ref: Ghai 8/e p614-620; 7/e p590-591; Khurana 4/e, p380; Nelson 19/e p1754
25. To which of the following events is ‘good’ outcome in neuroblastoma associated: (AI 04)
  1. Diploidy
  2. N-myc amplification
  3. Chromosome Ip depletion
  4. Trk A expression
Ref: Ghai 8/e p616; Robbin’s 7/e p763
26. Staging for Wilm’s syndrome? (AI 09)
  1. Chadwick
  2. International staging international society of pediatrics (ISOP)
  3. AJCC
  4. TNM
Ref: International Society of Pediatric. Oncology (ISOP); Nelson 19/e p1757, 1758; Ghai 8/e p617
27. The most common presentation of a child with Wilms’ tumor is: (AI 05)
  1. As asymptomatic abdominal mass
  2. Hemaruria
  3. Hypertension
  4. Hemoptysis due to pulmonary secondary
Ref: Ghai 8/e p617; 7/e p591-592; Nelson 18/e p2140-2143 and 17/e p1712; Nelson 19/e p1758
99
28. Presence of Nephroblastomatosis in a biopsy specimen from Wilm’s Tumor indicates? (AIIMS Nov 09)
  1. Chance of LN involvement increased
  2. Chance of Wilm’s tumor in Left kidney increased
  3. Poor prognosis
  4. The child could have Denys-Drash syndrome
(Ref: Nelson Textbook of Paediatrics, 18th edition; American journal, Nephroblastomatosis: Clinicopathologic Significance and Imaging Characteristics; National Center for Biotechnology Information; Nelson 19/e p1758, 1759; Ghai 8/e p617
29. A 7-year old boy with left renal mass had bone pain and was detected to have bone metastatic deposits. The most likely renal tumor is: (AIIMS Nov 04)
  1. Favorable histology Wilms tumor
  2. Renal cell carcinoma
  3. Clear cell sarcoma
  4. Rhabdoid tumor
Ref: Ghai 8/e p617; Nelson 18/e p2140, 2141; Nelson 19/e p1758
30. The most important determinant of prognosis in Wilms tumor: (AI 06)
  1. Stage of disease
  2. Loss of heterozygosity of chromosome Ip
  3. Histology
  4. Age less than one year at presentation
Ref: Ghai 8/e p617; Nelson 18/e p2143; Nelson 19/e p1759
31. Which of the following is the most common inherited malignancy ? (AIIMS May 05)
  1. Infant leukemia
  2. Retinoblastoma
  3. Wilm’s tumor
  4. Neuroblastoma
Ref: Ghai 8/e p614; 7/e p594; Robbin’s illustrated 7/e p284-287; Nelson 18/e p2151, 2152; Nelson 19/e p1768, 1769
32. Most common cause of mass in posterior mediastinum in children : (PGI June 01)
  1. Rhabdosarcoma
  2. Duplication cyst of esophagus
  3. Lymphoma
  4. Neuroblastoma
  5. Thymoma
Ref: Ghai 8/e p616; Bailey and Love 24/e p790; Nelson 18/e p2137; Nelson 19/e p1753
33. Commonest tumor of face in children is: (PGI Dec. 99)
  1. Rhabdomyosarcoma
  2. Sq. cell carcinoma
  3. Basal cell carcinoma
  4. Mixed parotid tumor
Ref: Ghai 8/e p618; Nelson 18/e p2144 and 17/e p1714; Nelson 19/e p1760-1762
34. Pilocytic astrocytoma false is: (AIIMS May 09)
  1. Spreads to posterior fossa
  2. Seen at eighty years of age
  3. Best prognosis among all intracranial neoplasms.
  4. Surgery and radiation therapy used for treatment
Ref: Ghai 8/e p612; Harrison’s 17/e p2602-2603
35. Chang staging is used for: (AIIMS May 2010)
  1. Retinoblastoma
  2. Medulloblastoma
  3. Ewing’s sarcoma
  4. Rhabdomyosarcoma
Ref: Ghai 8/e p612; Nelson 19/e p1751, 1752
36. Which of the following childhood tumors show extracranial metastasis? (AIIMS May 2010)
  1. Glioblastomamultiforme
  2. Medulloblastoma
  3. Choroid plexus carcinoma
  4. Ependymoblastoma
Ref: Ghai 8/e p612
37. What are the cancer in children group: (PGI Nov 09)
  1. Wlim’s
  2. Embryonal rhabdomyosarcoma
  3. Yolk cell tumor
Ref: Ghai 6/e p561; Robbins 7/e p111; Nelson 19/e p1751, 1752
38. The most characteristic radiographic sign in a child with leukemia is: (COMED 06)
  1. Osteosclerosis of the metaphysic
  2. Metaphyseal translucencies
  3. Periosteal reaction
  4. Osteolytic lesion
Ref: Ghai 8/e p599; Sutton 7/e p1327
39. Which is the commonest childhood tumor: (JIPMER 03)
  1. ALL
  2. CLL
  3. AML
  4. CML
Ref: Ghai 8/e p599
40. Genetic risk factors for leukemia are all except: (Kerala 04)
  1. Down’s syndrome
  2. Bloom syndrome
  3. Ataxia telangiectasia
  4. Turner’s syndrome
Ref: Ghai 8/e p598
41. CML in children is associated with: (NIMH. 01)
  1. Down’s syndrome
  2. Klinefelter syndrome
  3. Marran’s syndrome
  4. Turner’s syndrome
Ref: Ghai 8/e p608
42. Retinoblastoma gene is located on: (TNPSC 00)
  1. Chromosome 5
  2. Chromosome 8
  3. Chromosome 13
  4. Chromosome 16
Ref: Ghai 8/e p614; Nelson 17/e p1722
100
43. Deletion of chromosome 11 leads to: (DNB 01)
  1. Wilm’s
  2. Neuroblastoma
  1. Retinoblastoma
  2. Osteosarcoma
Ref: Ghai 8/e p617; Nelson 17/e p1683 table (484.2)
44. All are associated with malignancy except: (MP 00)
  1. Down’s syndrome
  2. Fragile X syndrome
  3. Bloom syndrome
  4. Fanconi’s anaemia
Ref: Ghai 8/e p608-620; Nelson 17/e p1683 table (484.2)
45. Most common maligant tumour in childhood: (UP 07)
  1. Rhabdomyosarcoma
  2. Leukemia
  3. Lymphangioma
  4. Neuroblastoma
Ref: OP Ghai 8/e p599
46. Most common benign tumours during infancy is: (UP 07)
  1. Lymphangioma
  2. Hemangioma
  3. Cystic hygroma
  4. Lipoma
Ref: OP Ghai 8/e p623
47. Poor prognostic factor for ALL is?
  1. Hyperdiploidy
  2. t(9;22) t(4;11)
  3. 2-8 yrs of age
  4. TLC < 50000
Ref: Ghai 8/e p599; Nelson 18/e p2123
48. Most common inheritend childhood tumor is: (DNB 2008)
  1. Leukemia
  2. Neuroblastoma
  3. Retinoblastoma
  4. Wilms’ tumor
Ref: Ghai 8/e p638; Nelson Pediatrics 18/e p2151-2153
49. Chang staging is for: (DNB 2008)
  1. Retino blastoma
  2. Rhabdo myosarcoma
  3. Ewings sarcoma
  4. Medulloblastoma
Ref: Ghai 8/e p830; Nelson Pediatrics 18/e p2132-2134
50. Wilms tumour is associated with all except: (DNB 2008)
  1. Aniridia
  2. Hemihypertrophy
  3. Hypertension
  4. Bilateral polycystic Kidney
Ref: Ghai 8/e p729; Nelson Pediatrics 18/e p2140-2143
51. Most common inheritend childhood tumor is: (DNB 2009)
  1. Leukemia
  2. Neuroblastoma
  3. Retinoblastoma
  4. Wilm’s tumor
Ref: Ghai 8/e p719; Nelson Pediatrics 18/e p2151-2153
52. Chang staging is for: (DNB 2009)
  1. Retino blastoma
  2. Rhabdo myosarcoma
  3. Ewings sarcoma
  4. Medulloblastoma
Ref: Ghai 8/e p724; Nelson Pediatrics 18/e p2132-2134
53. Wilms tumour is associated with all except: (DNB 2009)
  1. Aniridia
  2. Hemihypertrophy
  3. Hypertension
  4. Bilateral polycystic Kidney
Ref: Ghai 8/e p670; Nelson Pediatrics 18/e p2140-2143
54. About Wilm’s tumor incorrect statement is: (DNB 2011)
  1. At age 10 years is the most common presentation
  2. Presents as a mass in abdomen
  3. Lungs are the commonest site for metastasis
  4. Haematuria is the presenting symptom
Ref: OP Ghai 8/e p638; 7/e p592
55. Second most common neoplasm seen in children is (Karnatka PG 2011)
  1. Lymphoma
  2. Brain tumour
  3. Wilm’s tumour
  4. Neuroblastoma
Ref: Op Ghai 8/e p571, 612; 7/e p545, 580
56. Increased fetal Hb is seen in :(DP PGMEE 2010)
  1. Juvenile CML
  2. Congenital red cell aplasia
  3. Hereditary spherocytosis
  4. AML
Ref: Ghai 8/e p608; Nelson 18/e p2122
57. Which is not a tumour of the first decade of life? (DP PGMEE 2010)
  1. Rhabdomyosarcoma
  2. Neuroblastoma
  3. Medulloblastoma
  4. Retinoblastoma
Ref: Ghai 8/e p612, 613
58. Best prognostic cerebral tumour in children: (Raj PG 2009)
  1. Medulloblastoma
  2. Astrocytoma
  3. Glioma
  4. Meningioma
Ref: Ghai 8/e p612
59. All of the following brain tumors spread via CSF except: (Kerala PG 09)
  1. Craniopharyngoma
  2. CNS lymphoma
  3. Germ Cell tumours
  4. Medulloblastoma
Ref: Ghai 8/e p572
60. Neuroblastoma arises form: (Kerala PG 10)
  1. Adrenal
  2. Chestwall
  3. Rib
  4. Cerebrum
Ref: Ghai 8/e p616; Nelson 18/e p2137
 
 
Ans.
1. d. These tumors ...
2. a. Mostly is ...
3. b. Oligodendroglioma
4. d. Medulloblastoma
5. a. Cerebellar...
6. b. Craniopharyngioma
7. b. Most tumors...
8. b. Suprasellar
9. a. Leukemia
10. b. Precursor B cell ALL
11. a. Acute leukemia
12. b. Lymphoma
13. d. Hypodiploid
14. b. c. and d.
15. d. t (9: 22), t (8:14), t (4:11)
16. b. c. and e
17. d. Turner’s syndrome
18. b. Down syndrome
19. a. Neuroblastoma
20. b. Neuroblastoma
21. b. Neuroblastoma
22. a. Lytic lesion in...
23. a. b. d. and e
24. a and c
25. d. Trk A expression
26. b. International...
27. a. As asymptomatic
28. b. Chance of Wilm’s...
29. c. Clear cell sarcoma
30. a. and c.
31. b. Retinoblastoma
32. d. Neuroblastoma
33. a. Rhabdomyosarcoma
34. b. Seen at eighty...
35. b. Medulloblastoma
36. b. Medulloblastoma
37. a and b
38. b. Metaphyseal
39. a. ALL
40. d. Turner’s syndrome
41. a. Down’s syndrome
42. c. Chromosome 13
43. a. Wilm’s
44. b. Fragile X syndrome
45. b. Leukemia
46. b. Hemangioma
47. b. t(9;22) t(4;11)
48. c. Retinoblastoma
49. d. Medulloblastoma
50. d. Bilateral polycystic Kidney
51. c. Retinoblastoma
52. d. Medulloblastoma
53. d. Bilateral polycystic...
54. a. At age 10 years
55. b. Brain tumour
56. a. Juvenile CML
57. a. Rhabdomyosarcoma
58. b. Astrocytoma
59. a. Craniopharyngoma
60. a. Adrenal
 
10116. MISCELLANEOUS
 
102MISCELLANEOUS (QUESTIONS)
1. At the end of 1 year of age, the number of carpal bones seen in the skiagram ofthe hand is: (Corned 08)
  1. Nil
  2. 1
  3. 2
  4. 3
Ref: Ghai 8/e p32; Nelson 17/e p. 37 table (10.7)
2. A girl on exposure to cold pallor, then cyanosis, predisposed to: (AIIMS Nov 08)
  1. Scleroderma
  2. JRA
  3. SLE
  4. None
Ref: Ghai 8/e p630; Nelson 18/e p1024; Nelson 19/e p851
3. A child presents with seborrheic dermatitis, lytic skull lesions, ear discharge and hepatosplenomegaly; likely diagnosis: (AI 01)
  1. Leukemia
  2. Lymphoma
  3. Histiocytosis X
  4. Multiple myeloma
Ref: Ghai 8/e p620; 7/e p595-596; Nelson 18/e p902; Nelson 19/e p1773-1775
4. A 2-year-old child comes with discharge, seborrheic dermatitis, polyuria and hepatosplenomegaly. Which of the following is the most likely diagnosis : (AIIMS May 04)
  1. Leukemia
  2. Lymphoma
  3. Langerhan’s cell histiocytosis
  4. Germ cell tumor
Ref: Ghai 8/e p620; 7/e p595; Nelson 18/e p902; Nelson 19/e p1775
5. A one-year-old boy presented with hepatosplenomegaly and delayed milestones. The liver biopsy and bone marrow biopsy revealed presence of histocytes with PAS positive. Electron microscopic examination of these histiocytes is most likely to reveal the presence of: (AIIMS 03)
  1. Birbeck granules in the cytoplasm
  2. Myelin figures in the cytoplasm
  3. Parallel rays of tubular structures in lysosomes
  4. Electron dense deposit in the mitochondria
Ref: Ghai 8/e p620; Nelson 18/e p2159, 2160, Robbin’s 7/e p701; Nelson 19/e p1773, 1774
6. A 2-year-old child presents with scattered lesions in the skull. Biopsy revealed Langerhans giant cells. The most commonly associated is marker with this condition will be: (AIIMS 05)
  1. CD Ia
  2. CD 57
  3. CD 3
  4. CD 68
Ref: Ghai 8/e p620; Nelson 18/e p2159, 2160, Robbin’s 7/e p701; Nelson 19/e p1773
7. True about Langerhan’s histocytosis ‘X’: (PGI June 01)
  1. Can be associated with diabetes insipidus
  2. X-ray shows pathognomonic osteosclerotic lesions
  3. Birbeck’s granules in langhan’s cell
  4. Proliferation of antigen presenting cells
  5. Associated with specific HLA DR
Ref: Ghai 8/e p620; Nelson 19/e p1774
8. Which of the following drugs is used for the treatment of refractory histiocytosis: (AIIMS Nov 08)
  1. High dose methotiocytosis
  2. High dose cytarabine
  3. Cladribine
  4. Fludarabine
Ref: Ghai 8/e p620; Nelson 18/e p2161; Nelson 19/e p1774
9. A 2-year-old girl child is brought to the out patient with features of hand wringing stereotype movements, impaired language and communication development, breath-holding spells, poor social skills and deceleration of head growth after 6 months of age. The most likely diagnosis is: (AIIMS Nov 08)
  1. Asperger’s syndrome
  2. Rett’s syndrome
  3. Fragile X syndrome
  4. Colard syndrome
Ref: Nelson 18/e p2504, 2505 and 17/e p94, 2034; Nelson 19/e p2075; Internet
10. A mother is exposed to Diethyl stilbesterol during pregnancy. All the following features may be seen in the child after birth, except: (AIIMS May 02)
  1. Clear cell carcinoma
  2. Microglandular hyperplasia
  3. Malformation of the vagina and uterus
  4. Vaginal adenosis
Ref: Gynae shows 15/e p248; Nelson 17/e p1839; Robbin’s illustrated 7/e p1071; Nelson 19/e p1873
11. Kasabach-Merritt syndrome, true about A/E: (AIIMS Nov 09)
  1. Platelet sequestration
  2. Infantile hemangioma
  3. Consumption coagulopathy
  4. Portwine hemangioma
  5. Thrombocytosis
Ref: Medscape
12. All of the followings are seen in Systemic Juvenile Arthritis, except: (AI 00)
  1. Rheumatoid factor positive
  2. Hepatosplenomegaly
  3. High fever with rash
  4. Elevated SR
Ref: Ghai 8/e p624; Nelson 19/e p829
103
13. What is the probable diagnosis for a cyst in a child which is located at and associated with vertebral defects: (AI 08)
  1. Myelocele
  2. Bronchogenic cyst
  3. Neuroenteric cyst
  4. Neuroblastoma
Ref: Ghai 8/e p619; Sabiston 18/e p1696, Internet
14. A child is brought with drowsiness, decreased deep tendon reflexes and seizures. On examination the child has a line on gums. There is history of constipation. Which will be most appropriate drug that should be used in this child: (AI 07)
  1. EDTA
  2. DMSA
  3. BAL
  4. Penicillamine
Ref: Ghai 8/e p696; KDT 6/e p645, Nelson 18/e p2916; Nelson 19/e p2452
15. A two year old boy presents with fever for 3 days which responded to administration of paracetamol. Three days later he developed acute renal failure, marked acidosis and encephalopathy. His urine showed plenty of oxalate crystals. The blood anion gap and osmolal gap were increased. Which of the following is the most likely diagnosis: (AIIMS Nov 05)
  1. Paracetamol poisoning
  2. Diethyl glycol poisoning
  3. Severe malaria
  4. Hanta virus infection
Ref: Ghai 8/e p696; Nelson 17/e p2372
16. Combinnation of retinitis pigmentosa and ichthyosis is seen in: (AI 08)
  1. Netherton syndrome
  1. Refsum’s symdrome
  2. Down’s syndrome
  3. Mob’s syndrome
Ref: Ghai 8/e p666; CPDT 18/e p759
17. Sphenoid wing dysplasia is seen in: (JIPMER 2K)
  1. Von Hippel Lindau’s disease
  2. Neurofibromatosis
  3. Sturge Weber syndrome
  4. Bournville’s diisease
Ref: Ghai 8/e p586; www.springerlink
18. A child presents with a history of scorpion sting. He is having increased sweating. What is the next best step: (JIPMER 2K)
  1. Lytic coktail
  2. Atropine
  3. Antevenom
  4. Local xylocaine infiltration
Ref: Ghai 8/e p700; www.springerlink
19. A small boy who is brought to the emergency department by his parents is found to have a spiral fracture of the femur, with a variety of ecchymoses. What is the most likely cause of the injuries: (Karnataka 03)
  1. Automobile hit - and - run accident
  2. Fall from tree
  3. Child abuse
  4. Fall from a bicycle
Ref: Ghai 8/e p708; Nelson 18/e p174
20. Cherry red spots and cloudy cornea are seen in: (AI 08)
  1. Glycogen storage disease
  2. Lysosomal storage disease
  3. Amino acidurias
  4. Fall from a bicycle
Ref: Nelson 18/e p594, Ghai 8/e p652
21. WHO 3 step ladder is for management of: (Orissa 04)
  1. Pain
  2. Dysphagia
  3. Dyspnea
  4. Dysuria
Ref: Nelson 18/e p476
22. In infants, the cause of blindness arising out of oxygen toxicity is: (AI 08)
  1. Degeneration of crystalline lens
  1. Growth of blood vessels into vitreous followed by fibrosis
  2. Damage to cornea
  3. Enzymic defect in lens
Ref: OP Ghai 8/e p665
23. Ten percent dextrose is given with NS in neonates, the sister has to:
  1. 20%D + 80%NS
  2. 40% + 60%NS
  3. 60%D + 40%NS
  4. 80%D + 20%NS
Ref: Ghai 8/e p320
24. Which of the following is the least likely cause of neonatal mortality in India?
  1. Severe infections
  2. Congenital malformation
  3. Permaturity
  4. Birth asphyxia
Ref: SPM Part 20/e p485
25. Regarding an imbecile, all are true except?
  1. IQ is 50-60
  1. Intellectual capacity equivalent to a child of 3-7 years of age
  2. Not able to take care of themselves
  3. Condition is congenital or acquired at an early age
Ref: O P Ghai 7/e p562
26. A term male baby, with birth weight 3.5 kg, developed respiratory distress at birth, not responded to administration of surfactant. Echocardiogram was normal. X-ray chest shows ground glass appearance. There is history of death of a female sibling at 1 month of age. What is the diagnosis?
  1. TAPVC
  2. Meconium aspiration
  3. Neonatal alveolar proteinosis
  4. None of above
Ref: Nelson 18/e p1820-1821
104
27. Which of the following is not true about JRA?
  1. Fever
  2. Uveitis
  3. Rheumatoid nodules
  4. Raynaud’s phenomenon
Ref: Ghai 6/e p581 - 582; Nelson 18/e p1001-1010 and 17/e p799 – 801
28. False about acrodermatitis enteropathica
  1. Triad of diarrrhea, dementia, and dermatitis
  2. Low zinc level
  3. Symptoms improve with zinc supplement.
  4. Autosomal recessive
Ref: Internet
29. Histopathological featues of fat necrosis in a newborn closely resembles?
  1. Steroid induced lipid necorsis
  2. Lupoid necrosis
  3. Erythema induratum
  4. Lipodermatosclerosis
Ref: Internet
30. A 9-year-old girl has difficulty in combing hair and climbing upstairs. Gowers sign is positive. What is the next investigation to be performed?
  1. ESR
  2. RA factor
  3. Creatine kinase
  4. EMG
Ref: Nelson 18/e p2095 and 17/e p2053-2054; Harrison 16/e p1450
31. Failure to pass meconium within 48 hrs of birth in a newborn with no obvious external abnormality should lead to the suspicion of: (DNB 2005)
  1. Anal atresia
  2. Congenital pouch colon
  3. Congenital aganglionosis
  4. Meconium ileus
Ref: Ghai 8/e p512; Nelson 16/e p1888 15/e p1784)
32. Best method of diagnosis of childhood HIV: (DNB 2007)
  1. CD4 cell counts
  2. P24 antigen
  3. ELISA
  4. Anti HIV antibody
Ref: OP Ghai peds 8/e p510, Nelson Pediatrics 18/e p1427-1435
33. Most common sequelae to periventricular leukomalacia is: (DNB 2008)
  1. Spastic diplegia
  2. Spastic quadriplegia
  3. Hypotonia
  4. Mental retardation
Ref: Ghai 8/e p633; Nelsons text book of paediatrics, Campbell Text book of Peds, 10/e p1259
34. In Hutchinsons triad not seen is: (DNB 2009)
  1. Keratitis
  2. Notched incisors
  3. Deafness
  4. Cataracts
Ref: 8/e p710
35. Not a cause of infantile tremor syndrome is: (DNB 2009)
  1. Malnutrition
  2. Magnesium deficiency
  3. Infections
  4. Vitamin b 12 deficency
  5. None
Ref: OP Ghai 8/e p639
36. Pawn ball megakaryocytes are characteristic of: (DNB 2010)
  1. Myelodysplastic syndrome
  2. Idiopathic thrombocytopenic purpura
  3. Thrombotic thrombocytopenic purpura
  4. Chloramphenicol toxicity
Ref: Ghai 8/e p230; Campbell Text book of Peds, 10/e p868, 869
37. Early Neonatal sepsis in india is most commonly due to: (DNB 2010)
  1. Escherichia coli
  2. Group-B Streptococci
  3. Staphylococci
  4. Pseudomonas
Ref: Ghai 8/e p274; Nelson Essentials of Pediatrics 4/e p375, 376, 377
38. Hypoglycemia in a new borne is glucose level of less than: (DNB 2011)
  1. 50
  2. 55
  3. 40–45 mg/day
  4. 35
Ref: Ghai 8/e p218; Essentials of Peds 4/e p672, Nelson Paeds 17/e p816
39. Not a response seen in new borne as a measure of Thermogenesis is: (DNB 2011)
  1. Flexion
  2. Vasoconstriction
  3. Brown fat breakdown
  4. Shivering
Ref: Ghai 8/e p99; Op Ghai 6/e p150, 151, 152, Campbell Peds
40. Neonatal hypoglycaemia is defined as blood glucose of less than: (MP PG 2010)
  1. 30 mg/dl
  2. 40 mg/dl
  3. 50 mg/dl
  4. 60 mg/dl
Ref: Ghai 8/e p179; Nelson’s 18/e p702; NNF Manual of neonatal care 92
41. The most common mode of transmission of HIV from the mother to the baby is: (MP PG 2009)
  1. Transplacental
  2. During delivery
  3. Through breast milk
  4. Kiss and skin contract
Ref: Ghai 8/e p183; Nelson’s 18/e p1429
42. Which channel is defective in “Malignant Hyperpyrexia”? (MP PG 2008)
  1. Sodium
  2. Potassium
  3. Calcium
  4. Magnessium
Ref: Ghai 8/e p521; Harrison’s 17/e p118
43. Major criteria for jones classification is: (MP PG 2008)
  1. Fever
  2. Arthralgia
  3. Chorea
  4. PR Interval
Ref: Ghai 8/e p433; 7/e p38
105
44. Antidote of paracetamol poisoning: (MP PG 2008)
  1. Flumazenil
  2. N-acetyle cysteine
  3. Nalexone
  4. Sodium bicarbonate
Ref: Ghai 8/e p700; 7/e p680
45. Hypoglycemia in neonates occurs in blood glucose less than (UP PG 09)
  1. 20 mg%
  2. 40 mg%
  3. 60 mg%
  4. 10 mg%
Ref: Ghai 8/e p179; 7/e p155
46. Increased ammonia level, normal pH, acidosis (high anion gap) is seen in (UP PG 09)
  1. Urea cycle
  2. Galectosemia
  3. Organic-acidemia
  4. Alkaptonuria
Ref: Ghai 8/e p655
47. What should be the internal diameter of the endotracheal tube in a child weighing 1500g ? (MHPGM-CET 2007, 2010)
  1. 2mm
  2. 2.5mm
  3. 3mm
  4. 3.5mm
Ref: OP Ghai 8/e p137
48. A 10-year-old gets drowned and stays under water for 20 minutes, what should you do? (AP 2010)
  1. Barbiturate Reflux
  2. Steroid administration
  3. Cardiac massage
  4. Wait & watch
Ref: Ghai 8/e p705, 706
49. All of the following drugs are used in the remission induction in a case of all except: (AP 2010)
  1. Vincristine
  2. Prednisone
  3. L-asparaginase
  4. Cytosine arabinoside
Ref: Ghai 8/e p599
50. Which is not an induction for bone marrow transplantation (AP 2010)
  1. Severe combined immunodeficiency disease
  2. Wiskott-Aldrich’s syndrome
  3. Ataxia telangiectasia
  4. Chronic granulomatous disease
Ref: Ghai 8/e p187
51. A positive Apt test would mean that the blood is due to
  1. Urogenital bleeding from the neonate
  2. Gastrointestinal bleeding from the neonate
  3. Neonate swallowed maternal blood
  4. Neonate aspirated maternal blood
52. Sudden ophthalmoplegia in Thai child eating raw fish and cooked rice (AP 2011)
  1. Thaimine
  2. Lead Poisoning
  3. Botulism
  4. Influenza
Ref: Ghai 8/e p696
53. Prophylaxis with cotrimoxazole is recommentded in the following situation except: (DP PGMEE 2009)
  1. All symptomatic HIV infected children>5 year of age irrespective of CD4
  2. All HIV exposed infants till HIV infection can be ruled out
  3. All HIV infected infants less than 1 year age irrespective of symptoms or CD4 counts
  4. As secondary prophylaxis after initial treatment for pneumocystis carini pneumonial
Ref: Ghai 8/e p183; 7/e p205-207
54. Indications to start ART in children under NACO all except: (DP PGMEE 2009)
  1. <11 months infant-if CD4 Count <1500 cells/mm3
  2. 12-35 months-if CD4 count <750 cells/mm3
  3. 36-59 months-if CD4 Cont <350 cells/mm3
  4. >5 year-if CD4 Cout <200 cells/mm3
Ref: Ghai 8/e p183; 7/e p204, Nelson 18/e p1430-1432
55. Factor associated with increased incidence for sudden infant death syndrome is: (DP PGMEE 2009)
  1. High socioeconomic status
  2. Female gender
  3. Supine sleep positioning
  4. Maternal smoking
Ref: Ghai 8/e p708; Reddy 22/e p374
56. An infant is crying excessively even after being given good feed. He is passing a large quantity of urine and repeatedly getting dehydrated. Urine examination shows no proteinuria and a specific gravity of 1004. The most likely diagnosis is: (DP PGMEE 2010)
  1. Diabetes mellitus
  2. Diabetes insipidus
  3. Congenital nephritic syndrome
  4. Protein losing enteropathy
Ref: Ghai 8/e p519
57. A small boy who is brought to the emergency department by his parents is found to have a spiral fracture of the femur, with a variety of ecchymoses. What is the most likely cause of the injures? (DP PGMEE 2010)
  1. Automobile hit-and-run accident
  2. Fall from a tree
  3. Child abuse
  4. Fall from a bicycle
Ref: Ghai 8/e p770; Nelson 18/e p174
58. Which of the following is the principal mode of heat exchange in an infant incubator? (DP PGMEE 2010)
  1. Radiation
  2. Evaporation
  3. Convection
  4. Conduction
Ref: Ghai 8/e p145
59. Enzyme replacement therapy is available for which of the following disorders? (Feb DP PGMEE 2009)
  1. Gaucher’s disease
  2. Niemann pick disease
  3. Mucolipidosis
  4. Metachromtic leukodystrophy
Ref: Ghai 8/e p661
106
60. Costochondral junction swelling are seen in: (Raj PG 2009)
  1. Scurvy
  2. Rickets
  3. Chondrodystrophy
  4. All of the above
Ref: Ghai 8/e p113
61. Percentage of burn in an infant if head and neck are involved: (Kerala PG 08)
  1. 9 %
  2. 18 %
  3. 27 %
  4. 36 %
Ref: Ghai 8/e p705; Nelson, 17/e p333 Fig. 62-1, 18/e p453, Fig. 74-3
62. Contraindication for bag and mask ventilation among the following is: (Kerala PG 08)
  1. Meconium aspiration
  2. Tracheo esophageal fistula
  3. Congenital lung cyst
  4. Birth asphyxia
Ref: OP Ghai 8/e p125
63. Calcification of intervertebral disc occurs in: (Kerala PG 08)
  1. Alkaptonuria
  2. Phenylketouria
  3. Gout
  4. Rickets
Ref: Ghai 8/e p653; Nelson, 17/e p403; 18/e p534
64. All are true about cephalhematoma except: (Kerala PG 08)
  1. Subperiosteal location
  2. Donot cross suture line
  3. Subsides within 24 hours
  4. Increase in jaundice
Ref: Ghai 8/e p141; Dutta 6/e p483
65. Lithium administration in pregnancy will cause which of the following abnormality in fetus: (Kerala PG 08)
  1. Ebstein anomaly
  2. Transposition of great vessels
  3. Truncus arteriosus
  4. Tetralogy of Fallot
Ref: Ahuja textbook of psychiatry, 5/e p196; Ghai 8/e p423
66. Which among the following is not true regarding artificial feeding in mothers with HIV infection: (Kerala PG 10)
  1. Acceptable if safe alternative
  2. Continue if affordable
  3. Rapid weaning from initial 6 months of breast feeding
  4. Not at all recommended
Ref: Ghai 8/e p183; Nelson 18/e p215, 1429
67. In Ehlers Danlos Syndrome there is defect In: (WB PG 08)
  1. Glycoprotein
  2. Collagen
  3. Cartilage
  4. Mucopolysaccharide
Ref: Ghai 8/e p624, 630; Harrison 6/e p2328
68. I systemic form of JRA, all are true except: (WB PG 08)
  1. RA factor positive
  2. High Fever with rash
  3. Hepato splenomegaly
  4. Elevated ESR
Ref: Ghai 8/e p624, 630
69. All of the following are indirect markers of Neonatal Sepsis except: (J & K PG 2011)
  1. Blood Culture
  2. Total Leucocyte count
  3. I/T ratio
  4. C – Reactive Protein
Ref: OP Ghai 8/e p163
70. In which of the following poisonings is gastriclavage contraindicated: (J & K PG 2010)
  1. Organophosporous compound
  2. Kerosene
  3. Opium
  4. INH
Ref: Ghai 8/e p696
71. The bones of which joint are e valuated for estimation of skeletal age in children between 3 and 9 months: (J & K PG 2010)
  1. Wrist
  2. Shoulder
  3. Elbow
  4. Hip
Ref: OP Ghai 8/e p11
 
 
Ans.
1. c. 2
2. a. Scleroderma
3. c. Histiocytosis X
4. c. Langerhan’s cell histiocytosis
5. a. Birbeck granules...
6. a. CD Ia
7. a, b, c and e
8. c. Cladribine
9. b. Rett’s syndrome
10. b. Microglandular hyperplasia
11. b, d and e
12. a. Rheumatoid factor...
13. c. Neuroenteric cyst
14. a. EDTA
15. b. Diethyl glycol...
16. b. Refsum’s symdrome
17. b. Neurofibromatosis
18. d. Local xylocaine...
19. c. Child abuse
20. b. Lysosomal...
21. a. Pain
22. b. Growth of blood ...
23. d. 80%D + 20%NS
24. b. Congenital malformation
25. a. IQ is 50-60
26. c. Neonatal alveolar proteinosis
27. d. Raynaud’s...
28. a. Triad of diarrrhea...
29. d. Lipodermatosclerosis
30. c. Creatine kinase
31. c. Congenital...
32. b. P24 antigen
33. a. Spastic diplegia
34. d. cataracts
35. e. None
36. a. Myelodysplastic
37. b. Group-B Streptococci
38. c. 40–45 mg/day
39. d. Shivering
40. b. 40 mg/dl
41. b. During delivery
42. c. Calcium
43. c. Chorea
44. b. N-acetyle cysteine
45. b. 40 mg%
46. c. Organic-acidemia
47. c. 3mm
48. c. Cardiac massage
49. d. Cytosine arabinoside
50. c. Ataxia telangiectasia
51. c. Neonate swallowed…
52. a. Thaimine
53. a. All symptomatic...
54. b. 12-35 months-if CD4...
55. d. Maternal smoking
56. b. Diabetes insipidus
57. c. Child abuse
58. c. Convection
59. a. Gaucher’s disease
60. d. All of the above
61. b. 18%
62. a. Meconium aspiration
63. a. Alkaptonuria
64. c. Subsides within...
65. a. Ebstein anomaly
66. d. Not at all recommended
67. b. Collagen...
68. a. RA factor positive...
69. a. Blood Culture
70. b. Kerosene
71. b. Shoulder